You are on page 1of 87

JURISDICTION

16. SAN MIGUEL PROPERTIES v. SEC. HERNANDO PEREZ, ET AL. (2013)


To accord with the doctrine of primary jurisdiction, the courts cannot and will not determine a controversy
involving a question within the competence of an administrative tribunal, the controversy having been so
placed within the special competence of the administrative tribunal under a regulatory scheme.
San Miguel sued BF Homes in the HLURB for specific performance to compel the latter to deliver the TCTs of
fully paid lots purchased by San Miguel. Not happy with just one suit, San Miguel filed a complaint-affidavit in the
OCP Las Pias charging respondent directors and officers of BF Homes with non-delivery of titles in violation of
PD 957. The Prosecutor dismissed the criminal complaint, stating among others that there existed a prejudicial
question necessitating the suspension of the criminal action until after the issue on the liability of the distressed BF
Homes was first determined by the SEC en banc or by the HLURB. It appears that there was an issue regarding the
authority of the receiver appointed by the SEC for BF Homes to sell the lots.
The SC held that the action for specific performance, even if pending in the HLURB, an administrative
agency, raises a prejudicial question. In other words, the administrative case for specific performance in the
HLURB posed a prejudicial question that must first be determined before the criminal case for violation of
PD 957 could be resolved. It must be noted that the disputed matters in this case are within HLURBs
competence and expertise. That the action for specific performance was an administrative case pending in
the HLURB, instead of in a court of law, was of no consequence at all. The action for specific performance,
although civil in nature, could be brought only in the HLURB. This situation conforms to the doctrine of
primary jurisdiction.
17. FIRST GAS POWER CORPORATION V. REPUBLIC OF THE PHILIPPINES (2013)
The doctrine of judicial stability states that the judgment of a court of competent jurisdiction may not be
interfered with by any court of competent jurisdiction which is founded on the concept of jurisdiction.
Petitioner filed a petition for the registration of 2 parcels of land in Batangas City before the RTC. The RTC
granted petitioners application for registration. Subsequently however, petitioner filed a Manifestation with Motion
in the same RTC, manifesting the existence of a previous LRA Report stating that the lots were already registered
through Cadastral Case No. 37 and moved that the latter case be set aside. The RTC granted the motion and issued
an amended order setting aside the earlier decision. The CA granted the petition for certiorari filed by the
respondents and found that the amended order was issued in grave abused of discretion. The RTCs Amended
Order setting aside the earlier decision in Cadastral Case No. 37 was issued in violation of the doctrine of
judicial stability. A court that acquires jurisdiction over the case and renders judgment therein has
jurisdiction over its judgment, to the exclusion of all other coordinate courts, for its execution and over all its
incidents, and to control the conduct of ministerial officers in connection with the judgment.
63. HEIRS OF CANDIDO DEL ROSARIO, ET AL. vs. MONICA DEL ROSARIO (2012) [Jurisdiction of
RTC vs. DARAB]
The jurisdiction of the PARAD and the DARAB is limited only to all agrarian disputes and matters or
incidents involving the implementation of the CARP.
Candido, Gil, and Monica del Rosario are siblings who inherited a piece of land in Bulacan from their parents.
Candido and Gil filed with the Office of the Provincial Agrarian Reform Adjudicator (PARAD) a complaint against
Monica for amendment of TCT and partition of the land alleging that Monica refused to cede to Gil the 1/3 portion
of the land pursuant to their agreement. PARAD ruled in favor of Candido and Gil. On appeal, DARAB reversed
and ruled in favor of Monica. The CA, while holding that the PARAD and the DARAB both had no jurisdiction
over the complaint, ruled that the petitioners are bound by the decision of the DARAB having participated in the
proceedings without raising any objection thereto.
Held: The PARAD and the DARAB had no jurisdiction. The jurisdiction of the PARAD and the DARAB is
limited only to all agrarian disputes and matters or incidents involving the implementation of the CARP. In
this case, the complaint for amendment and partition does not involve any agrarian dispute, nor does it involve

REM Digests ALS 2014B Justice Gesmundo


any incident arising from the implementation of agrarian laws. The petitioners and Monica have no tenurial,
leasehold, or any agrarian relations whatsoever that will bring this controversy within the jurisdiction of the
PARAD and the DARAB. While ostensibly assailing Monicas qualification as a farmer-beneficiary, the petitioners
did not seek the nullification of the emancipation patent issued to Monica and the issuance of a new one in their
names. Instead, the petitioners merely sought that the subject land be equally partitioned among the surviving heirs
of their parents, including Monica. Verily, by merely asking for the recovery of their alleged hereditary share in the
subject land, the petitioners implicitly recognized the validity of the issuance of the emancipation patent over the
subject land in favor of Monica.
The decision of the DARAB is thus null and void and not binding upon the parties. Jurisdiction over the subject
matter cannot be acquired through, or waived by, any act or omission of the parties. The active
participation of the parties in the proceedings before the DARAB does not vest jurisdiction on the DARAB,
as jurisdiction is conferred only by law.
64b. ORTIGAS AND COMPANY, LIMITED PARTNERSHIP v. COURT OF APPEALS (2012)
"not every case involving buyers and sellers of subdivision lots or condominium units can be filed with the
HLURB, cases not falling under the jurisdiction of HLURB are filed with the court of general jurisdiction
(JURISDICTION)"
This case resolves the question of jurisdiction of the RTC over a complaint filed against a subdivision owner. The
City of Pasig filed a complaint against Ortigas and Greenhills Properties, Inc. (GPI) for specific compliance with
regard to Municipal Ordinance 5, Series of 1966. The MO requires it to designate appropriate recreational and
playground facilities at its former Capitol VI Subdivision.
As held in the case of Delos Santos v. Sarmiento, not every case involving buyers and sellers of subdivision
lots or condominium units can be filed with the HLURB. Its jurisdiction is limited to those cases filed by the
buyer or owner of a subdivision lot or condominium unit and based on any of the causes of action
enumerated in Section 1 of P.D. 1344. Obviously, the City had not bought a lot in the subject area from
Ortigas which would give it a right to seek HLURB intervention in enforcing a local ordinance that regulates
the use of private land within its jurisdiction in the interest of the general welfare. It has the right to bring
such kind of action but only before a court of general jurisdiction such as the RTC.
68. SAMELCO II v. Ananias (2012)
Rule 65 - Writ of Prohibition; Doctrine of Primary Jurisdiction; Corollary - Exhaustion of Administrative Remedies
the doctrine of primary jurisdiction applies where a claim is originally cognizable in the courts and comes
into play whenever enforcement of the claim requires the resolution of issues which, under a regulatory
scheme, has been placed within the special competence of an administrative agency. In such a case, the court
in which the claim is sought to be enforced may suspend the judicial process pending referral of such issues
to the administrative body for its view or, if the parties would not be unfairly disadvantaged, dismiss the case
without prejudice.
Corollary to the doctrine of primary jurisdiction is the principle of exhaustion of administrative remedies. The
Court, in a long line of cases, has held that before a party is allowed to seek the intervention of the courts, it
is a pre-condition that he avail himself of all administrative processes afforded him. Hence, if a remedy
within the administrative machinery can be resorted to by giving the administrative officer every
opportunity to decide on a matter that comes within his jurisdiction, then such remedy must be exhausted
first before the courts power of judicial review can be sought. The premature resort to the court is fatal to
ones cause of action. Accordingly, absent any finding of waiver or estoppel, the case may be dismissed for
lack of cause of action. The doctrine of exhaustion of administrative remedies is based on practical and legal
reasons. The availment of administrative remedy entails lesser expenses and provides for a speedier
disposition of controversies. Furthermore, the courts of justice, for reasons of comity and convenience, will
shy away from a dispute until the system of administrative redress has been completed and complied with, so
as to give the administrative agency concerned every opportunity to correct its error and dispose of the case.
True, the doctrines of primary jurisdiction and exhaustion of administrative remedies are subject to certain
exceptions, to wit:

REM Digests ALS 2014B Justice Gesmundo


(a) where there is estoppel on the part of the party invoking the doctrine;
(b) where the challenged administrative act is patently illegal, amounting to lack of jurisdiction;
(c) where there is unreasonable delay or official inaction that will irretrievably prejudice the complainant;
(d) where the amount involved is relatively so small as to make the rule impractical and oppressive;
(e) where the question involved is purely legal and will ultimately have to be decided by the courts of justice;
(f) where judicial intervention is urgent;
(g) where the application of the doctrine may cause great and irreparable damage; (h) where the
controverted acts violate due process;
(i) where the issue of non-exhaustion of administrative remedies has been rendered moot;
(j) where there is no other plain, speedy and adequate remedy;
(k) where strong public interest is involved; and
(l) in quo warranto proceedings.
As members of the Board of Directors (BOD) of the petitioner Samar II Electric Cooperative, Inc. (SAMELCO II),
an electric cooperative [organized under the provisions of Presidential Decree (P.D.) No. 269, otherwise known as
the National Electrification Administration Decree, as amended by P.D. No. 1645] providing electric service to
all members-consumers in all municipalities within the Second Congressional District of the Province of Samar,
individual petitioners passed Resolution No. 5 [Series] of 2005 on January 22, 2005. The said resolution disallowed
the private respondent to attend succeeding meetings of the BOD effective February 2005 until the end of his term
as director. The same resolution also disqualified him for one (1) term to run as a candidate for director in the
upcoming district elections. Convinced that his rights as a director of petitioner SAMELCO II had been curtailed by
the subject board resolution, private respondent filed an Urgent Petition for Prohibition against petitioner
SAMELCO II, impleading individual petitioners as directors thereof, in the Regional Trial Court (RTC) in Calbiga,
Samar. RTC granted the petition. CA affirmed.
the basic issue in the present case is not whether the RTC has jurisdiction over the petition for prohibition filed
by respondent; rather, the issue is who between the RTC and the NEA has primary jurisdiction over the question of
the validity of the Board Resolution issued by SAMELCO II. P.D. No. 269 as amended provides:
Section 10. Enforcement Powers and Remedies. In the exercise of its power of supervision and control over
electric cooperatives and other borrower, supervised or controlled entities, the NEA is empowered to issue orders,
rules and regulations and motu proprio or upon petition of third parties, to conduct investigations, referenda and
other similar actions in all matters affecting said electric cooperatives and other borrower, or supervised or
controlled entities.
Section 24. Board of Directors. (a) The Management of a Cooperative shall be vested in its Board, subject to the
supervision and control of NEA which shall have the right to be represented and to participate in all Board meetings
and deliberations and to approve all policies and resolutions.
The Court, therefore, finds it erroneous on the part of the CA to rule that the doctrine of primary
jurisdiction does not apply in the present case. while the RTC has jurisdiction over the petition for
prohibition filed by respondent, the NEA, in the exercise of its power of supervision and control, has primary
jurisdiction to determine the issue of the validity of the subject resolution. In order that prohibition will lie,
the petitioner must first exhaust all administrative remedies. Thus, respondent's failure to file a complaint
before the NEA prevents him from filing a petition for prohibition before the RTC.
70. Addition Hills Mandaluyong vs Megaworld Properties Inc. (April 18, 2012. Exhaustion of Adminstrative
Remedies)
When the law provides for a remedy against a certain action of an administrative board, body, or officer,
relief to the courts can be made only after exhausting all remedies provided therein. It is settled that the non-
observance of the doctrine of exhaustion of administrative remedies results in lack of cause of action, which
is one of the grounds in the Rules of Court justifying the dismissal of the complaint.

REM Digests ALS 2014B Justice Gesmundo


Megaworld conceptualized the construction of a residential condo complex in Wack-Wack Heights. However
AHMCSO (Addition Hills Mandaluyong) filed a complaint in RTC Pasig to prohibit issuance of licenses to sell to
Megaworld. Megaworld filed a Motion to Dismiss claiming lack of cause of action as jurisdiction is with the
HLURB and not the regular courts. A complaint to annul any permit issued by the HLURB may be filed before the
Housing and Land Use Arbiter (HLA). The decision of the HLA may be brought to the Board of Commissioners by
Petition for Certiorari and the decision of the Board of Commissioners is appealable to the Office of the President.
Megaworld is right. Addition Hills unjustifiably failed to exhaust the administrative remedies available with
the Housing and Land Use Regulatory Board (HLURB) before seeking recourse with the trial court. It is
settled that the non-observance of the doctrine of exhaustion of administrative remedies results in lack of
cause of action, which is one of the grounds in the Rules of Court justifying the dismissal of the complaint.
92. LAND BANK OF THE PHILIPPINES v. CORAZON M. VILLEGAS (2010)
Jurisdiction is the courts authority to hear and determine a case. The courts jurisdiction over the nature
and subject matter of an action is conferred by law. The Comprehensive Agrarian Reform Law states that a
branch of an RTC designated as a Special Agrarian Court for a province has the original and exclusive
jurisdiction over all petitions for the determination of just compensation in that province. SC ruled that
Special Agrarian Courts have original and exclusive jurisdiction over two categories of cases: (1) all petitions
for the determination of just compensation to landowners, and (2) the prosecution of all criminal offenses
under R.A. 6657. By "special" jurisdiction, Special Agrarian Courts exercise power in addition to or over
and above the ordinary jurisdiction of the RTC, such as taking cognizance of suits involving agricultural
lands located outside their regular territorial jurisdiction, so long as they are within the province where they
sit as Special Agrarian Courts.
93. LAND BANK OF THE PHILIPPINES v. HEIRS OF CATALINO V. NOEL ET AL. (2010)
JURISDICTION: A branch of an RTC designated as a Special Agrarian Court for a province has the original
and exclusive jurisdiction over all petitions for the determination of just compensation in that province.
Petitioner Land Bank of the Philippines (Land Bank) filed cases for determination of just compensation against
respondent Corazon M. Villegas and respondent heirs of Catalino V. Noel and Procula P. Sy before the RTC of
Dumaguete City sitting as a Special Agrarian Court for the province of Negros Oriental. Respondent Villegas
property was in Hibaiyo, Guihulngan City, Negros Oriental, while respondent heirs land was in Nangca, Bayawan
City, Negros Oriental. These lands happened to be outside the regular territorial jurisdiction of RTC of Dumaguete
City.
The sole question presented in these cases is whether or not an RTC, acting as Special Agrarian Court, has
jurisdiction over just compensation cases involving agricultural lands located outside its regular jurisdiction but
within the province where it is designated as an agrarian court under the Comprehensive Agrarian Reform Law of
1998.
A branch of an RTC designated as a Special Agrarian Court for a province has the original and exclusive
jurisdiction over all petitions for the determination of just compensation in that province. In Republic v. Court of
Appeals,[9] the Supreme Court ruled that Special Agrarian Courts have original and exclusive jurisdiction over two
categories of cases: (1) all petitions for the determination of just compensation to landowners, and (2) the
prosecution of all criminal offenses under R.A. 6657. By special jurisdiction, Special Agrarian Courts exercise
power in addition to or over and above the ordinary jurisdiction of the RTC, such as taking cognizance of suits
involving agricultural lands located outside their regular territorial jurisdiction, so long as they are within the
province where they sit as Special Agrarian Courts. Since RTC, Branch 32 of Dumaguete City is the designated
Special Agrarian Court for the province of Negros Oriental, it has jurisdiction over all cases for determination of
just compensation involving agricultural lands within that province, regardless of whether or not those properties
are outside its regular territorial jurisdiction.
103. SULTAN YAHYA JERRY M. TOMAWIS vs. HON. RASAD G. BALINDONG, AMNA A.
PUMBAYA, JALILAH A. MANGOMPIA, and RAMLA A. MUSOR (2010)
Respondents filed with the Sharia District Court an action for quieting of title of a parcel of land against petitioner
Sultan Tomawis and one Mangoda Radia. In his answer, Tomawis debunked the sisters claim of ownership and
raised, as one of his affirmative defenses treated by the court as a motion to dismiss, SDCs lack of jurisdiction over
REM Digests ALS 2014B Justice Gesmundo
the subject matter of the case. As argued, the regular civil court, not SDC, had such jurisdiction pursuant to BP 129.
Petitioner contends that Art. 143 of PD 1083, insofar as it granted the SDC concurrent jurisdiction over certain real
actions, was repealed by the BP 129 provisions adverted to. Private respondents however contend that, Art.
143(2)(b) of PD 1083 is of specific applicability and, hence, cannot, be superseded by laws of general application,
absent an express repeal.
A reading of the pertinent provisions of BP 129 and PD 1083 shows that the former, a law of general application to
civil courts, has no application to, and does not repeal, the provisions found in PD 1083, a special law, which only
refers to Sharia courts. We must read and construe BP 129 and PD 1083 together, then by taking PD 1083 as an
exception to the general law to reconcile the two laws. Also, as things stood prior to the effectivity date of BP 129,
the SDC had, by virtue of PD 1083, original jurisdiction, concurrently with the RTCs and MTCs, over all personal
and real actions outside the purview of Art. 143(1)(d) of PD 1083, in which the parties involved were Muslims,
except those for ejectment.
It is important to note that Petitioner waited two years after the CA issued its denial before filing what turned out to
be his second motion to dismiss. The Court frowns on such unsporting practice. The rule is settled that a question of
jurisdiction may be raised at any time, even on appeal, provided its application does not result in a mockery of the
basic tenets of fair play.
111. SANTE v. CLARAVALL February 22, 2010, JURISDICTION
In cases where the claim for damages is the main cause of action, or one of the causes of action, the amount of such
claim shall be considered in determining the jurisdiction of the court. A case was filed a complaint for damages in
the RTC for moral damages P300,000; exemplary P50,000; attorneys fees P50,000; P20,000 cost of suit. Is this
within the exclusive jurisdiction of the MTCC since the moral damages claimed is P300,000? No you include
exemplary damages in computing for reasons given above. Also you could amend claim of damages. Also while it
is a basic jurisprudential principle that an amendment cannot be allowed when the court has no jurisdiction over the
original complaint and the purpose of the amendment is to confer jurisdiction on the court, however since the RTC
had jurisdiction in this case the RTC correctly allowed the amendment of damages claimed form P300,000 to P1M.
122.) THE CITY MAYOR OF BAGUIO VS. ATTY. BRAIN MASWENG, G.R. NO. 165003 (2010)
Jurisdiction of the NCIP to hear and decide main actions for injunction
Respondents are membets of the Ibaloi tribe. Their ancestors were grantees of a survey plan approved by the
Director of Lands in 1920. They have a pending case before the NCIP for the validation of ancestral land claims
covering a parcel of land in Baguio City and Tuba, Benguet. A portion of such land overlaps the Baguio Dairy
Farm which is a govt reservation under the supervision of the Department of Agriculture. Respondents filed a
petition for injunction (with prayer for issuance of a TRO and a writ of preliminary injunction) with the NCIP
seeking to enjoin the mayor of Baguio City and the head of the citys demolition team to implement a demolition
order. Brian Masweng, the regional hearing officer of NCIP issued a 72 hr TRO. Petitioners appealed. CA upheld
NCIP decision. In City Government of Baguio v. Atty. Masweng we have held that the NCIP may issue
temporary restraining orders and writs of injunction without any prohibition against the issuance of the writ
when the main action is for injuction. The power to issue TROs or writs of injunction allows parties to a
dispute over which the NCIP has jurisdiction to seek relief against any action which may cause them grave
or irreparable damage or injury. However the claim of the respondents on the subject land is still pending
before the NCIP. Thus their rights are mere expectations, not the present and unmistakable right required
for the grant of the provisional remedy of injunction. Moreover the structure subject of the demolition were
either being built or being constructed without the requisite permit at the time the demolition order was
issued.
173. Ridgewood Estate Inc (Camella Homes) v. Belaos (2006) Jurisdiction of HLURB
The mere relationship between the parties, i.e., that of being subdivision owner/developer and subdivision lot
buyer, does not automatically vest jurisdiction in the HLURB. For an action to fall within the exclusive
jurisdiction of the HLURB, the decisive element is the nature of the action as enumerated in Section 1 of P.D.
No. 1344. The HLURB has jurisdiction over complaints aimed at compelling the subdivision developer to
comply with its contractual and statutory obligations.

REM Digests ALS 2014B Justice Gesmundo


Belaos entered into a contract to sell with Camella Homes, a subdivision developer, for the purchase of a house and
lot in Cavite and issued several postdated checks as amortization for the property. Camella failed to construct the
house and therefore Belaos wrote a letter rescinding the contract and demanding the return of the amounts paid and
postdated checks. Camella only remitted the downpayment and six monthly amortizations but continued to encash
the other postdated checks. Belaos then filed a complaint for damages against Camella Homes. Camella filed a
MTD on the ground that it was not a real party in interest and that the complaint stated no cause of action (contract
was entered bet. Belaos and Ridgewood Estate) and that Camella was not an entity authorized by law to be a party
to a civil suit. The RTC denied Camellas MTD, applying the doctrine on corporation by estoppel. Camella filed a
petition for certiorari before the CA and in addition, raised that the subject matter of the complaint was within the
exclusive jurisdiction of the Housing and Land Use Regulatory Board (HLURB). The CA dismissed the petition.
The RTC has jurisdiction. Under PD 1344 the HLURB (then Natl Housing Authority) has exclusive
jurisdiction over cases of the following nature: a. Unsound real estate business practices; b. Claims involving
refund and any other claims filed by subdivision lot or condominium unit buyer against the project owner,
developer, dealer, broker or salesman; and c. Cases involving specific performance of contractual and
statutory obligations filed by buyers of subdivision lot or condominium unit against the owner, developer,
dealer, broker, or salesman. The complaint filed by Belaos against Camella was one for damages. It prayed for
the payment of moral, actual and exemplary damages by reason of Camellas malicious encashment of the checks
even after the rescission of the contract to sell between them. Belaos claimed that because of Camellas malicious
and fraudulent acts, he suffered humiliation and embarrassment in several banks, causing him to lose his credibility
and good standing among his colleagues. Such action falls within the jurisdiction of regular courts, not the
HLURB.
175. Smart Communications Inc (SMART) et al., v. National Telecommunciations Comm (NTC) (2003)
doctrine of exhaustion of admin remedies and doctrine of primary jurisdiction
The doctrine of exhaustion of administrative remedies and doctrine of primary jurisdiction only apply when
an admin. agency performs its quasi-judicial functions, not when the act assailed is pursuant to its rule-
making or quasi-legislative function, as in the case of questioning the validity or constitutionality of a rule or
regulation.
The NTC issued a Memo Circ No. promulgating rules and regulations on the billing of telecommunications
services. Petitioners filed an action for declaration of nullity of the billing circular on the following grounds: the
NTC has no jurisdiction to regulate the sale of consumer goods such as the prepaid call cards since such jurisdiction
belongs to the DTI under the Consumer Act of the Philippines; that the Billing Circular is oppressive, confiscatory
and violative of the constitutional prohibition against deprivation of property without due process of law; that the
Circular will result in the impairment of the viability of the prepaid cellular service by unduly prolonging the
validity and expiration of the prepaid SIM and call cards; and that the requirements of identification of prepaid card
buyers and call balance announcement are unreasonable. NTC filed a MTD for failure to exhaust admin. remedies.
In questioning the validity or constitutionality of a rule or regulation issued by an administrative agency, a
party need not exhaust administrative remedies before going to court. This principle applies only where the
act of the administrative agency concerned was performed pursuant to its quasi-judicial function, and not
when the assailed act pertained to its rule-making or quasi-legislative power. Quasi-legislative or rule-making
power is the power to make rules and regulations. Quasi-judicial or administrative adjudicatory function is
the power to hear and determine questions of fact to which the legislative policy is to apply and to decide in
accordance with the standards laid down by the law itself in enforcing and administering the same law. This
includes the authority to investigate facts or ascertain the existence of facts, hold hearings, weigh evidence, and
draw conclusions from them as basis for their official action and exercise of discretion in a judicial nature. In the
case at bar, the issuances by the NTC was pursuant to its quasi-legislative or rule-making power.
In like manner, the doctrine of primary jurisdiction applies only where the administrative agency exercises its
quasi-judicial or adjudicatory function. The courts will not determine a controversy involving a question which
is within the jurisdiction of the administrative tribunal prior to the resolution of that question by the administrative
tribunal, where the question demands the exercise of sound administrative discretion requiring the special
knowledge, experience and services of the administrative tribunal to determine technical and intricate matters of
fact, and a uniformity of ruling is essential to comply with the premises of the regulatory statute administered. It

REM Digests ALS 2014B Justice Gesmundo


applies where the claim is originally cognizable in the courts and comes into play whenever enforcement of the
claim requires the resolution of issues which, under a regulatory scheme, has been placed within the special
competence of an administrative body; in such case, the judicial process is suspended pending referral of such
issues to the administrative body for its view.
The determination of whether a specific rule or set of rules issued by an administrative agency contravenes
the law or the constitution is within the jurisdiction of the regular courts. Indeed, the Constitution vests the
power of judicial review or the power to declare a law, treaty, international or executive agreement, presidential
decree, order, instruction, ordinance, or regulation in the courts, including the regional trial courts. This is within
the scope of judicial power, which includes the authority of the courts to determine in an appropriate action the
validity of the acts of the political departments. Judicial power includes the duty of the courts of justice to settle
actual controversies involving rights which are legally demandable and enforceable, and to determine whether or
not there has been a grave abuse of discretion amounting to lack or excess of jurisdiction on the part of any branch
or instrumentality of the Government. In their complaint before the Regional Trial Court, petitioners averred
that the Circular contravened Civil Code provisions on sales and violated the constitutional prohibition
against the deprivation of property without due process of law. These are within the competence of the trial
judge. Contrary to the finding of the CA, the issues raised in the complaint do not entail highly technical
matters. Rather, what is required of the judge who will resolve this issue is a basic familiarity with the workings of
the cellular telephone service, including prepaid SIM and call cards and this is judicially known to be within the
knowledge of a good percentage of our population and expertise in fundamental principles of civil law and the
Constitution.

RULE 1

119. METROPOLITAN BANK AND TRUST CO. vs PEREZ


when insufficient filing fees were initially paid by the plaintiffs and there was no intention to defraud the
government, the Manchester ruling does not apply
TOPIC: Rule 1, Docket Fee
The filing fees Perez paid did not cover her prayer for unrealized income for the ensuing idle months since at the
time of filing and payment, the period that the building would be idle could not yet be determined. Metrobank
claims Perez deliberately concealed the insufficient payment of docket fees and that the complaint is thus rendered
dismissible for lack of jurisdiction. HELD: The ensuing months in which the leased premises would be rendered
vacant could not be determined at the time of the filing of the complaint. The building constructed on Perez leased
premises was specifically constructed to house a bank, hence, the idle period before another occupant with like
business may opt to lease would be difficult to project. While the payment of prescribed docket fee is a
jurisdictional requirement, even its non-payment at the time of filing does not automatically cause the
dismissal of the case, as long as the fee is paid within the applicable prescriptive or reglementary period,
more so when the party involved demonstrates a willingness to abide by the rules prescribing such payment.
Thus, when insufficient filing fees were initially paid by the plaintiffs and there was no intention to defraud
the government, the Manchester ruling (strict), that a pleading which does not specify in the prayer the
amount sought shall not be admitted or shall be expunged, and that a court acquires jurisdiction only upon
payment of the prescribed docket fee, does not apply. However, Metrobank is held in estoppel since it only
raised the issue of jurisdiction before the appellate court after it participated in the proceedings before the trial
court. Perez was ordered to pay the balance between actual fees paid and the correct payable filing fees to include
an assessment on the award of unrealized income.

REM Digests ALS 2014B Justice Gesmundo


RULE 2

7. UNICAPITAL INC V CONSING JR 2013:


A cause of action is defined as the act or omission by which a party violates a right of another. It is well-
settled that the existence of a cause of action is determined by the allegations in the complaint. In this
relation, a complaint is said to sufficiently assert a cause of action if, admitting what appears solely on its
face to be correct, the plaintiff would be entitled to the relief prayed for. Thus, if the allegations furnish
adequate basis by which the complaint can be maintained, then the same should not be dismissed, regardless
of the defenses that may be averred by the defendants. The resolution on this matter should stem from an
analysis on whether or not the complaint is able to convey a cause of action; and not that the complainant
has no cause of action.
Facts: The case involves a consolidation of two special actions filed in the RTC of pasig and another one by the
respondent in the pasig case in a Makati-RTC (this is the case below: Consing Jr.) This consolidated case answered
the cause of action issue in the two cases before RTC-pasig. Unilateral gave a loan in favour of Dela Cruz worth
18m secured by post-dated checks and a real mortgage on a parcel of land. PBI (real estate co.) wanted to develop
the land of De Villa so it negotiated a joint-venture agreement with Unilateral through its real estate arm, UNI.
Consing Jr was appointed by Dela Cruz as his Atty. in-fact. Consing negotiated the sale and the parties agreed to
pay Dela Cruz P42,195,397.16. After payment but before the titles could be passed, a certain Teng and Yu emerged
claiming they are the real owners of the lot sold and that they never sold it to Dela Cruz. Upon investigation PBI,
Unicapital and UNI found the title of Dela Cruz dubious so they demanded for the return of their payment. At this
time, Consing got fed up because he was the one being nagged by the buyers. He filed an action Complex Action
for Declaratory Relief, later amended to Complex Action for Injunctive Relief claiming he was being coerced by
the buyers to issue blank signed deed of sale as well as to issue post-dated checks for the amount to be recovered
knowing that he did not have enough funds making him commit a violation of B.P.22. PBI, UNI and Unilateral
moved for dismissal alleging no cause of action.
Held: Dismissal not proper. The elementary test for failure to state a cause of action is whether the complaint
alleges facts which if true would justify the relief demanded. Stated otherwise, may the court render a valid
judgment upon the facts alleged therein? The inquiry is into the sufficiency, not the veracity of the material
allegations. If the allegations in the complaint furnish sufficient basis on which it can be maintained, it should
not be dismissed regardless of the defense that may be presented by the defendants. Court found Consing, Jr.s
complaint in SCA No. 1759 properly states a cause of action since the allegations therein sufficiently bear out
a case for damages under Articles 19 and 26 of the Civil Code.
8. CONSING JR V HON. MACARAIG-GUILLEN
It is hornbook principle that when or two or more cases involve the same parties and affect closely related
subject matters, the same must be consolidated and jointly tried, in order to serve the best interest of the
parties and to settle the issues between them promptly, thus, resulting in a speedy and inexpensive
determination of cases. In addition, consolidation serves the purpose of avoiding the possibility of conflicting
decisions rendered by the courts in two or more cases, which otherwise could be disposed of in a single suit.
Facts: This is the other side of the consolidated case above mentioned. In this case, Unicapital Inc. filed a case for
recovery of sum of money plus damages and attorneys fees against Consing Jr. and Dela Cruz before the Makati
RTC. PBI also filed a case for damages and attachement against Consing and Dela Cruz. Consing filed for a
motion to dismiss. Denied. He moved for consolidation of this case with the two pending before the Pasig RTC.
Motion for consolidation was dismissed based on no identity of rights or causes of action and the reliefs sought for
by Consing, Jr. from the RTC-Pasig City will not bar Unicapital from pursuing its money claims against him.
Moreover, the RTC-Makati City noted that Consing, Jr. filed his motion only as an afterthought as it was made
after the mediation proceedings between him and Unicapital failed.
Held: Consolidation not proper.The Court observes that the subject cases, i.e., SCA No. 1759 and Civil Case
No. 99-1418, although involving the same parties and proceeding from a similar factual milieu, should
remain unconsolidated since they proceed from different sources of obligations and, hence, would not yield
conflicting dispositions. SCA No. 1759 is an injunction and damages case based on the Civil Code provisions
REM Digests ALS 2014B Justice Gesmundo
on abuse of right and defamation, while Civil Case No. 99-1418 is a collection and damages suit based on
actionable documents, i.e., the subject promissory notes. In particular, SCA No. 1759 deals with whether or
not Unicapital and PBI, et al. abused the manner in which they demanded payment from Consing, Jr., while
Civil Case No. 99-1418 deals with whether or not Unicapital may demand payment from Consing, Jr. based
on the subject promissory notes. Clearly, a resolution in one case would have no practical effect as the core
issues and reliefs sought in each case are separate and distinct from the other.
46. LILIA B. ADA ET AL v. FLORANTE BAYLON (2012)
1. Partition + rescission = misjoinder; 2. Misjoinder = not always dismissal or severance, if there is no
objection and both actions are under jurisdiction of the trying court; 3. Supplemental pleading can give rise
to new cause of action.
This case started out as an action for PARTITION filed before the RTC. In a supplemental pleading however, the
petitioners in the case assailed a donation inter vivos between the defendants in an action for RESCISSION.
Held: The actions of partition and rescission cannot be joined in a single cause of action. A joinder of causes
of action is subject to the condition that the joinder shall not include special civil actions governed by special
rules. An action for partition is a special civil action governed by Rule 69 of the Rules of Court while an
action for rescission is an ordinary civil action governed by the ordinary rules of civil procedure.
Nonetheless, a misjoinder of causes of action is not a ground for dismissal. A misjoined cause of action, if not
severed upon motion of a party or by the court sua sponte, may be adjudicated by the court together with the other
causes of action. If there is no objection to the improper joinder or the court did not motu proprio direct a
severance, then there exists no bar in the simultaneous adjudication of all the erroneously joined causes of
action, provided that the court trying the case has jurisdiction over all the causes of action even if they are
misjoined.
If the court trying the case has no jurisdiction over a misjoined cause of action, then there is no option but to
SEVER the misjoined cause of action from the other causes of action, and if not so severed, any adjudication
rendered by the court with respect to the same would be a nullity.
Note that the cause of action for rescission in this case was introduced through a SUPPLEMENTAL PLEADING.
A supplemental pleading may raise a new cause of action as long as it has some relation to the original
cause of action set forth in the original complaint. The purpose of the supplemental pleading is to bring into
the records new facts which will enlarge or change the kind of relief to which the plaintiff is entitled, even
though the new facts themselves constitute a new right of action.
104. ARTHUR DEL ROSARIO AND ALEXANDER DEL ROSARIO VS. HELLENO D. DONATO, JR.
AND RAFAEL V. GONZAGA, RESPONDENTS (2010).
Philip Morris wrote the NBI, requesting assistance in curtailing the proliferation of fake Marlboro cigarettes in
Pampanga. After doing surveillance work, respondent Donato,, succeeded in confirming the storage and sale of
such fake cigarettes at the house that belonged to petitioner Alexander del Rosario. Respondent Donato applied for
a search warrant. Their search yielded no fake Marlboro cigarettes. Subsequently, petitioners, the Del Rosarios filed
a complaint against respondents NBI agents Donato and Gonzaga and two others. Respondents NBI agents
answered the complaint with a motion to dismiss on the grounds of failure of the complaint to state a cause of
action.
The test of sufficiency of a complaint is whether or not, assuming the truth of the facts that plaintiff alleges in it, the
court can render judgment granting him the judicial assistance he seeks. And judgment would be right only if the
facts he alleges constitute a cause of action that consists of three elements: (1) the plaintiff's legal right in the
matter; (2) the defendant's corresponding obligation to honor or respect such right; and (3) the defendant's
subsequent violation of the right. Absent any of these, the complaint would have failed to state a cause of action.
However, all that the Del Rosarios allege is that respondents NBI agents used an unlawfully obtained search
warrant against them, evidenced by the fact that, contrary to the sworn statements used to get such warrant, the NBI
agents found no fake Marlboro cigarettes in petitioner Alexander del Rosario's premises. But a judicially ordered
search that fails to yield the described illicit article does not of itself render the court's order "unlawful."
The Del Rosarios did not allege that respondents NBI agents violated their right by fabricating testimonies to

REM Digests ALS 2014B Justice Gesmundo


convince the RTC of Angeles City to issue the search warrant. Their allegation that the NBI agents used an
unlawfully obtained search warrant is a mere conclusion of law. While a motion to dismiss assumes as true
the facts alleged in the complaint, such admission does not extend to conclusions of law. Statements of mere
conclusions of law expose the complaint to a motion to dismiss on ground of failure to state a cause of action.
149. DOTMATRIX TRADING as rep. by its proprietors, namely ROMY YAP CHUA ET. AL., v.
ROMMEL B. LEGASPI ET. AL. (2009) (RULE 2)
"The rule on litis pendentia does not require that the case later in time should yield to the earlier case; what
is required merely is that there be another pending action, not a prior pending action. Neither is it required
that the party be served with summons before lis pendens can apply; it is the filing of the action, not the
receipt of summons, which determines priority in date.
Respondent is the proprietor of Big J Farms and RBL Farm supplies the day-old chicks to Dotmatrix, and in one
contract they were supposed to deliver P1,360,000 worth of day-old chicks but was only able to deliver P1,136,150
worth. When neither the demand to deliver the deficiency nor the return of overpayment was complied with, both
parties went to court for judicial relief. Petitioners filed on June 11, 2002 in RTC Tarlac a complaint for sum of
money and damages and respondent filed a case on June 19, 2002 in RTC Malolos a complaint for sum of money
and damages stating that P1,368,100 worth of day-old chicks was delivered but petitioners only paid P1,150,000.
Upon receipt of summons and complaint for the case filed by petitioner, the respondent filed a motion to dismiss on
the ground of litis pendentia before RTC Tarlac because it is merely anticipatory and defensive to the case filed in
RTC Malolos.
The Court held that YES, the case filed in RTC Tarlac which was filed first, should not be dismissed.
Although the elements of litis pendentia are present, there are different ways to determine which case should
be dismissed. Under this established jurisprudence on litis pendentia, the following considerations
predominate in the ascending order of importance in determining which action should prevail: (1) the date
of filing, with preference generally given to the first action filed to be retained (priority-in-time rule); (2)
whether the action sought to be dismissed was filed merely to preempt the later action or to anticipate its
filing and lay the basis for its dismissal (anticipatory test); and (3) whether the action is the appropriate
vehicle for litigating the issues between the parties the more appropriate action being the one where the real
issues raised can be fully and completely settled (more appropriate action test). In this case it was seen that
the respondent initiated the preparatory moves that led to the present litigation when he sent the petitioners
a demand letter for the payment of delivered day-old chicks 5 months after the end of the supply contract
thus the case filed by petitioners was merely anticipatory and that the real issue between the parties which is
whether or not the correct payment had been made on the delivered day-old chicks is better asserted in the
case filed by the respondents.

RULE 3

5. Remulla v. Maliksi (2013)


Courts will not hesitate to give standing to taxpayers in cases where serious legal issues are raised or where
public expenditures of millions of pesos are involved. A taxpayer need not be a party to the contract in order
to challenge its validity, or to seek the annulment of the same on the ground of extrinsic fraud. For as long as
taxes are involved, the people have a right to question contracts entered into by the government.
Facts: De villa as administratix of her husbands estate donated to the province of Cavite 134,957 sq m. of their
396,622 where the province erected its govt buildings. Subsequent thereto the province wanted to expropriate the
remaining lots. De villa opposed. Pending expropriation proceedings, the lot was sold to Goldenrod. While this was
happening, respondent (gov. of Cavite) issued an order that created a committee to be in charge of recommending
the terms and conditions of the expropriation. A compromise agreement was entered into and was ratified by the
RTC. Petitioner, vice-mayor and presiding officer of sanggunian panlalawigan of Cavite, thereafter filed a petition
of annulment of judgment in his capacity as tax payer and vice-mayor alleging that the sale of lots were overpriced.
CA denied petition holding he has no locus standing.

REM Digests ALS 2014B Justice Gesmundo


Held:Petitioner has locus standing.
10. SC MEGAWORLD CONSTRUCTION AND DEVELOPMENT CORPORATION v. ENGR.
LEONARDO A. PARADA OF GENLITE INDUSTRIES (2013)
"FIRST DOCTRINE: the verification and certification of non-forum shopping in the complaint is not a
jurisdictional but a formal requirement, and any objection to non-compliance therewith should be raised in
the proceedings below and not for the first time on appeal (FORUM SHOPPING)"
Megaworld bought electrical lighting materials from Genlite Industries, a sole proprietorship owned by Engr.
Parada. Megaworld was unable to pay on due date, but blamed it on its failure to collect under its sub-
contract. Megaworld was able to collect partial payments and was able to partially pay Genlite Industries. Parada
sued Megaworld for the balance, but Megaworld denied liability claiming that it was released from its indebtedness
by reason of the novation of their contract. He claims that the sub-contractor is now liable to Genlite Industries.
Any objection as to the compliance with the requirement of verification in the complaint should have been
raised in the proceedings in the trial court, and not in the appellate court for the first time. As held in the
case of Kilusan-Olalia v. CA: We have emphasized, time and again, that verification is a formal, not a
jurisdictional requisite, as it is mainly intended to secure an assurance that the allegations therein made are
done in good faith or are true and correct and not mere speculation. The Court may order the correction of
the pleading, if not verified, or act on the unverified pleading if the attending circumstances are such that a
strict compliance with the rule may be dispensed with in order that the ends of justice may be served.
"SECOND DOCTRINE: a sole proprietorship has no juridical personality separate and distinct from that of
its owner, and need not be impleaded as a party-plaintiff in a civil case (PARTIES)" same facts.
Genlite Industries is merely the DTI-registered trade name or style of the Engr. Parada by which he
conducted his business. As such, it does not exist as a separate entity apart from its owner, and therefore it
has no separate juridical personality to sue or be sued. As the sole proprietor of Genlite Industries, there is
no question that the Engr. Parada is the real party in interest who stood to be directly benefited or injured
by the judgment in the complaint below. There is then no necessity for Genlite Industries to be impleaded as
a party-plaintiff, since the complaint was already filed in the name of its proprietor.
21. B. ST. RITA, &CO. and KANAPI v. GUECO (2013)
"the complaint-in-intervention essentially latches on the complaint for its legal efficacy so much so that the
dismissal of the complaint leads to its concomitant dismissal. Furthermore, It is a standing rule that no
person shall be adversely affected by the outcome of a civil action or proceeding in which he is not a party."
Gueco purchased four parcels of land from B. Sta. Rita (Company) through its then President, Ben Sta. Rita (Ben),
situated at Barangay San Juan de Mata, Tarlac City. Gueco then filed a petition for the surrender of the subject titles
against B. Sta. Rita, its corporate secretary Edgardo Kanapi (Edgardo). The heirs of Ben intervened. Furthermore,
while the surrender of titles case was pending, the heirs of Ben, and as shareholders of Company, by way of a
derivative suit, filed a complaint for reformation and rescission of contract and quieting of title against Gueco. The
Heirs of Edgardo intervened in the reformation case. The cases were consolidated and both were eventually
dismissed.
This course of action is impelled by the fact that Arlene and the Heirs of Edgardo do not have any legal personality
to appeal the CA Decision before the Court since: first, they were only intervenors in the reformation case which
had already been dismissed by the Court with finality; and second, they were not parties in the surrender of titles
case. Arlenes and the Heirs of Edgardos complaint-in-intervention in the dismissed reformation case had been
effectively discharged since the principal complaint therein had already been terminated with finality. Clearly, their
complaint-in-intervention cannot be treated as an independent action as it is merely an ancillary to and a
supplement of the principal action. Anent the second incident, records disclose that Arlene or the Heirs of Edgardo
were not parties either as defendants or intervenors in the surrender of titles case nor did they, in any manner,
participate in the proceedings of the same. It is a standing rule that no person shall be adversely affected by the
outcome of a civil action or proceeding in which he is not a party.

REM Digests ALS 2014B Justice Gesmundo


23. ABDULRAHMAN v. OMBUDSMAN (2013)
"Section 11, Rule 3 of the Rules of Court, states that neither the misjoinder nor the non-joinder of parties is
a ground for the dismissal of an action AND that the general rule is that a motion for reconsideration is a
condition precedent to the filing of a petition for certiorari under Rule 65 of the Rules of Court."
Private respondent reported the alleged illegal activities of petitioner and Sayutin. According to private respondent,
petitioner solicited from him the total amount of PP5,450 as consideration for the titling in private respondents
name of lands located in South Upi, Maguindanao, and covered by the homestead applications of Unos Pacutin and
Ting Midtimbang. On the other hand, Sayutin received documents belonging to private respondent from Ellen
Alcoriza (Alcoriza), records officer of CENRO Salimbao, Sultan Kudarat, without authority therefor. The
Ombudsman recommended the dismissal of petitioner, Sayutin, and Alcoriza from service. Petitioner filed a motion
for reconsideration but it was denied. He then filed a Motion for New Trial or Second Motion for Reconsideration,
attaching thereto the Affidavit of private respondent, as well as the Joint-Affidavit of Mama Sangeban, Jr.
(Sangeban) and Mario Tuhok (Tuhok), both dated 16 August 1999. In an Order dated 23 August 1999, the
Ombudsman denied the motion for being a second motion for reconsideration. Petitioner filed a petition for review
before the CA docketed as CA-G.R. SP No. 55737 assailing the Ombudsmans Resolution recommending his
dismissal. On 31 March 2004, the Ombudsman issued an Order of Implementation directing DENR XII RED
officer-in-charge Jim Sampulna to implement the dismissal from service of petitioner and to show proof of
compliance within 10 days from receipt. The petitioner filed a Petition for Certiorari and Prohibition with Prayer
for a Status Quo Order but the CA dismissed the petition for the following reasons: (1) failure to implead private
respondent; and (2) failure to attach copies of the pleadings and documents relevant to the petition.
In this case, it was an error for the CA to dismiss the petition for failure to comply with Section 5, Rule 65 of the
Rules of Court. Section 11, Rule 3 of the Rules of Court, states that neither the misjoinder nor the non-joinder of
parties is a ground for the dismissal of an action.
As to the issue of a motion for reconsideration, there are well-settled exceptions to the general rule that a motion for
reconsideration is a condition precedent to the filing of a petition for certiorari under Rule 65 of the Rules of Court.
However, none of them finds application in this case, especially since questions raised in the certiorari proceeding
before the CA were different from those passed upon by the Ombudsman.
While petitioner initially questioned the Order of Implementation because it became a direct order to dismiss
allegedly beyond the authority of the Ombudsman, empowered as it is, only to recommend the removal of erring
public employees his main argument was that the Order of Implementation should have been addressed to the
Secretary of Environment and Natural Resources as the head of office who had the power to appoint and dismiss
him. In this case there is no GALDEJ. Petitioner himself manifested that at the time that private respondent filed the
complaint, the former was employed at DENR Xll on a contractual basis. Thus, when the recommendation to
dismiss petitioner from service was issued by the Ombudsman through the Resolution dated 14 March 1995, the
recommendation was coursed through then DENR XII RED Macorro Macumbal. Later, due to the query of the
DENR XII RED officer-in-charge regarding the status of the case of petitioner, the Order of Implementation dated
31 March 2004 was directed to the former to effect petitioner's dismissal.
35. Atty. Palmiano-Salvador v. Angeles (2012)
Effect of complaint filed by one without authority
If a complaint is filed for or on behalf of the plaintiff [by one] who is not authorized to do so, the complaint
is deemed not filed. An unauthorized complaint does not produce any legal effect. Hence, the court should
dismiss the complaint on the ground that it has no jurisdiction over the complaint of the plaintiff.
[The complaint was filed by an unauthorized person SPA executed 1 month after the complaint was filed.
The Court ruled that the trial court never had jurisdiction over the case and all the proceedings before it were
thus null and void.]

REM Digests ALS 2014B Justice Gesmundo


44. THEODORE AND NANCY ANG, REP. BY ELDRIDGE MARVIN B. ACERON VS. SPOUSES ALAN
AND E, ANG (2012)
Doctrine (Rules 3 and 4):
1. The proper venue is the court of the place where respondents reside. A personal action is involved as it
primarily seeks the enforcement of a contract. According to the ROC, the plaintiff has the option of
choosing where to file his complaint, (1) where he himself or any of them resides, or (2) where the
defendant or any of the defendants resides or may be found. The plaintiff or the defendant must be
residents of the place where the action has been instituted at the time the action is commenced. If the
plaintiff does not reside in the Philippines, the complaint may be filed in the court of the place where the
defendant resides.
2. Attorneys-in-fact are not real parties in interest because they do not stand to be benefited or injured by
any judgment therein
Spouses Alan and Em Ang (respondents) borrowed money from Theodore and Nancy Ang
(petitioners). Respondents executed a promissory note in favor of petitioners promising to pay amount, with 10%
interest per annum, upon demand. Respondents failed to pay, despite demands and receipt of a demand letter.
Petitioners executed an SPA in favor of Atty. Aceron to file an action against respondents for collection of sums of
money in RTC QC. Respondents filed an MR on the grounds of improper venue and prescription. They insisted
that the proper venue is the place where either they (Bacolod) or petitioners reside (LA, USA).
The proper venue should have been filed in RTC of Bacolod City, the court of the place where respondents
reside. A personal action is involved as it primarily seeks the enforcement of a contract. According to the ROC,
the plaintiff has the option of choosing where to file his complaint, (1) where he himself or any of them resides, or
(2) where the defendant or any of the defendants resides or may be found. The plaintiff or the defendant must be
residents of the place where the action has been instituted at the time the action is commenced. If the plaintiff does
not reside in the Philippines, the complaint may be filed in the court of the place where the defendant resides. The
petitioners, being residents of LA, USA, are not given a choice as to the venue of filing their complaint. The
complaint may only be filed in Bacolod City the court of the place where the respondents reside.
Despite being an attorney-in-fact, Atty. Aceron is not a real party in interest because he does not stand to be
benefited or injured by any judgment therein. Interest means material interest or an interest in issue to be affected
by the decree or judgment of the case, as distinguished from mere curiosity about the question
involved. Appointment as attorney-in-fact for a limited purpose does not mean that he is subrogated into the rights
of petitioners and ought to be considered a real party in interest. He is merely a representative. The real parties in
interest are the petitioners, the beneficiaries.
76. (b) AQUILINO PIMENTEL vs. SENATE COMMITTEE, G.R. No. 187714, March 8, 2011;
Indispensable Party; Section 7, Rule 3
An indispensable party is a party who has an interest in the controversy or subject matter that a final
adjudication cannot be made, in his absence, without injuring or affecting that interest.
Senator Lacson delivered a privilege speech on Septmeber 15, 2008Senator Lacson called attention to the
congressional insertion in the 2008 General Appropriations Act, particularly the P200 million appropriated for the
construction of the President Carlos P. Garcia Avenue Extension from Sucat Luzon Expressway to Sucat Road.
Lacson inquired from DBM Secretary Rolando Andaya, Jr. about the double entry and was informed that it was on
account of a congressional insertion. Senator Lacson further stated that when he followed the narrow trail leading to
the double entry, it led to Senator Villar, then the Senate President. Senator Villar delivered a privilege speech10
where he stated that he would answer the accusations against him on the floor and not before the Ethics Committee.
The motion was approved with ten members voting in favor, none against, and five abstentions.The preliminary
conference was set on 26 May 2009.
In its Comment, respondent argues that:
1. The instant petition should be dismissed for failure to join or implead an indispensable party. In the alternative,
the instant petition should be archived until such time that the said indispensable party has been joined or
impleaded and afforded the opportunity to be heard;

REM Digests ALS 2014B Justice Gesmundo


1. There was no grave abuse of discretion on the part of respondent Committee;
2. The instant petition must be dismissed for being premature. Petitioners failed to observe the doctrine or primary
jurisdiction or prior resort;
Indispensable Party
Section 7, Rule 3 of the 1997 Rules of Civil Procedure provides:
SEC. 7 Compulsory joinder of indispensable parties. - Parties in interest without whom no final determination
can be had of an action shall be joined as plaintiffs or defendants.
The test to determine if a party is an indispensable party is as follows:
An indispensable party is a party who has an interest in the controversy or subject matter that a final adjudication
cannot be made, in his absence, without injuring or affecting that interest, a party who has not only an interest in the
subject matter of the controversy, but also has an interest of such nature that a final decree cannot be made without
affecting his interest or leaving the controversy in such a condition that its final determination may be wholly
inconsistent with equity and good conscience. It has also been considered that an indispensable party is a person in
whose absence there cannot be a determination between the parties already before the court which is effective,
complete or equitable. Further, an indispensable party is one who must be included in an action before it may
properly go forward.
A person who is not an indispensable party, however, if his interest in the controversy or subject matter is separable
from the interest of the other parties, so that it will not necessarily be directly or injuriously affected by a decree
which does complete justice between them. Also, a person is not an indispensable party if his presence would
merely permit a complete relief between him and those already parties to the action, or if he has no interest in the
subject matter of the action. It is not a sufficient reason to declare a person to be an indispensable party that his
presence will avoid multiple litigation.15
In this case, Senator Madrigal is not an indispensable party to the petition before the Court. While it may be true
that she has an interest in the outcome of this case as the author of P.S. Resolution 706, the issues in this case are
matters of jurisdiction and procedure on the part of the Senate Committee of the Whole which can be resolved
without affecting Senator Madrigals interest. The nature of Senator Madrigals interest in this case is not of the
nature that this case could not be resolved without her participation.
84) Republic of the Philippines v Coalbrine International Philippines, Inc., G.R. 161838 (2010) (Rule 3-
Parties to Civil Action)
A real-party in interest must have a material interest on the actions done or outcome of the case. He must have
the right sought to be enforced. Mere incidental interest is not enough. The case springs from a contract between
PEZA and Coalbrine International regarding the rehabilitation and lease of the Bataan Hilltop Hotel. PEZA
rescinded the contract after which several actions were filed between the two opposing parties. Neri and Coalbrine
filed a complaint for damages against Administrator Quindoza and then RP as represented by Quindoza (in his
capacity as Admin of Bataan Ecozone) also filed a petition for certiorari under Rule 65 regarding the lower courts
decision on the preveious complaint for damages.
The case involves to questions regarding Rule 3 (Parties to Civil Action) :
1. The Interest of NERI
SC: The verification and certification against forum shopping signed by Neri was defective. First, because Neri is
not a real party in interest because based on the complaint for damages, the harassment solely affected the hotels
operation. Neris interest was only incidental since he was at that time the Hotels Managing Director.
Second, Neri failed to show proof of authority to institute the complaint and sign the verification and certification
against non-forum shopping on behalf of the corporation. The certification itself did not contain a statement that she
was authorized by the Corp Sec. Thus certif is defective and shall cause for the complaint to be dismissed.
2. RP was not a party to the civil case for damages hence it has no personality to file petition for review.

REM Digests ALS 2014B Justice Gesmundo


SC: Administrator Quindoza was sued for damages for certain acts he allegedly committed while he was the Zone
Admin of the Bataan EPZA. The complaint is the nature of suit against the State. Republic therefore has the
personality to file the petition.
86. Nemesio Goco, et. al v. Hon. Court of Appeals, et. al., G.R. No 157449 (2010) Rule 3
One having no material interest to protect cannot invoke the jurisdiction of the court as the plaintiff in an
action, in which case, it is dismissible on the ground of lack of cause of action.
In 1952, the Municipality of Calapan (now a City) acquired a one-half interest over Lot No. 2042 in satisfaction
of a judgment award in its favor against Alveyra. Upon registration, however, the entire Lot No. 2042 was included
in the TCT; the OCT was accordingly cancelled. To determine the extent of Alveyra and the Municipality of
Calapans interest over Lot No. 2042, an action to quiet title was instituted. The CA subdivided Lot No. 2042 into
two lots: one-half or Lot No. 2042-A (northern portion) was declared as the property of the heirs of Alveyra
who had since died, while the other half, Lot No. 2042-B (southern portion), was declared owned by the
Municipality of Calapan.
Meanwhile, while the heirs of Alveyra and the Municipality of Calapan were litigating their conflicting rights over
Lot No. 2042, the heirs sold their one-half interest over the land (Lot No. 2042-A) to respondent spouses Hicoblino
and Lourdes Catly (respondent Catlys). In 1999, respondent Catlys alleged that a portion of their Lot No. 2042-A
was being occupied by the petitioners and sought to recover possession of the lot, initially, by instituting an
ejectment case against the petitioners. Said case was dismissed, so respondent Catlys then filed a complaint for
recovery of possession. Allegedly to defend themselves against the cases filed by respondent Catlys and to protect
their vested rights as lawful occupants of the land, the petitioners filed a complaint for declaration of nullity of
the four certificates of title issued in respondent Caltys names. Respondent Catlys, in turn, moved for the
dismissal of the complaint asserting that it failed to state a cause of action and that the petitioners (plaintiffs below)
were not the real parties in interest.
In this case, the Court ruled that an action for annulment of title, like any other civil action, must be instituted
by the real party in interest. Section 2, Rule 3 of the Rules of Court states: A real party in interest is the party
who stands to be benefited or injured by the judgment in the suit, or the party entitled to the avails of the
suit. Unless otherwise authorized by law or these Rules, every action must be prosecuted or defended in the
name of the real party in interest.
This provision has two requirements: 1) to institute an action, the plaintiff must be the real party in interest;
and 2) the action must be prosecuted in the name of the real party in interest. Interest within the meaning of
the Rules of Court means material interest or an interest in issue to be affected by the decree or judgment of
the case, as distinguished from mere curiosity about the question involved. One having no material interest
to protect cannot invoke the jurisdiction of the court as the plaintiff in an action. When the plaintiff is not
the real party in interest, the case is dismissible on the ground of lack of cause of action.
128. LITTIE SARAH A. AGDEPPA ET. AL. VS. HEIRS OF IGNACIO BONETE ET AL. (2010) [RULE 3]
Misjoinder or nonjoinder of parties is not a cause of dismissal. Court could drop or implead any party to
correct the misjoinder or nonjoinder of parties.
Respondent obtained a loan from the DBP to purchase farm implements, a parcel of agricultural land was used a
collateral for the loan. Upon demand for payment, petitioner expressed deep sympathy and obligated herself to
pay the loan. Thereafter, Dorotea was allegedly made to sign a document as Littie Sarahs security for the amount
which the latter paid to DBP in connection with the said loan. Since 1982, Littie Sarah and her representatives had
been gradually easing them out of the subject property until they were eventually ejected from the
subject property. It turns out that the form she made Dorotea sign was really a deed of sale instead of a security for
loan. So they filed an action to annul the sale. Littie moved to dismiss since the lot was in the name of Dorotea
instead of her late husband hence she was not a real party in interest. While it is true that respondents committed
a procedural infraction before the RTC, such infraction does not justify the dismissal of the case. Misjoinder
of parties does not warrant the dismissal of the action. Rule 3, Section 11 of the Rules of Court clearly
provides: Neither misjoinder nor non-joinder of parties is ground for dismissal of an action. Parties may be
dropped or added by order of the court on motion of any party or on its own initiative at any stage of the action
and on such terms as are just. Any claim against a misjoined party may be severed and proceeded with
REM Digests ALS 2014B Justice Gesmundo
separately. It bears stressing that TCT No. T-56923, covering the subject property, was issued in the name of
Dorotea. This is established by the record, and petitioners themselves admit this fact. However, because TCT
No. T-75454, allegedly issued in favor of Littie Sarah, and the purported deed of sale, allegedly executed by
Dorotea in favor of Littie Sarah, are not on record. Considering the allegations in the pleadings, it is best
that a trial on the merits be conducted.
139. Navarro v. Hon. Escobido (2009)
A party in interest is one who will directly benefit from, or be injured from the filing of the suit.
Karen Go filed two complaints before the RTC for replevin and or/sum of money with damages against Navarro.
Navarro alleges that the complaints state no cause of action because the lease agreements were in the name of
Kargo Enterprises, an entity without the requisite juridical personality to sue. Moreover, since the actual parties to
the agreement are Navarro and Glenn Go (Karens husband) Karen was not a real party in interest. W/N Karen was
a real party-in-interest? YES. It is incorrect to say that her complaint does not state a cause of action because
her name did not appear in the Lease Agreement. As registered owner of the enterprise, Karen Go is the
party who will directly benefit from or be injured in this case and is the party-in-interest. Moreover, Glenn
Go need not be joined as he is not strictly an indispensable party in the action. He only needs to be impleaded
as a pro-forma party to the suit. (Husband and wife shall sue or be sued jointly, except as provided by law.)
Lastly, being co-owners of Kargo Enterprises, both have an equal right to bring an action with respect to the
co-owned property.
140. Marmo v. Anacay (2009)
where the suit is brought by a co-owner, without repudiating the co-ownership, then the suit is presumed to
be filed for the benefit of the other co-owners and may proceed without impleading the other co-owners.
However, where the co-owner repudiates the co-ownership by claiming sole ownership of the property or
where the suit is brought against a co-owner, his co- owners are indispensable parties and must be impleaded
as party-defendants, as the suit affects the rights and interests of these other co-owners.
Anacay owned a 50-sqm parcel of land with a house built thereon in Cavite. He authorized Josephine Marmo to sell
the subject property, which was subsequently purchased by Danilo for PhP 520,000.00, payable in monthly
installments of PhP8,667.00 from May 2001 to June 2006. Danilo defaulted in his payments. Anacay discovered
that the TCT in his own name has been cancelled and another was issued in the name of Josephine of a falsified
deed of sale. Such Deed of Sale was subsequently transferred to Danilo. Anacay now seeks the annulment of the
Deed of Absolute Sale and the cancellation of the TCTs. Josephine, in her answer, claims that Anacays children, as
co-owners of the subject property, should have been included as plaintiffs because they are indispensable parties.
W/N Anacays children are indispensable parties? NO. Section 7, Rule 3 of the Revised Rules of Court defines
indispensable parties as parties-in-interest without whom there can be no final determination of an action and if his
interest is such that a final decree cannot be made without affecting this interest or without placing the controversy
in a situation where the final determination may be wholly inconsistent with equity and good conscience.
In cases of co-ownership, where the suit is brought by a co-owner, without repudiating the co-ownership, then
the suit is presumed to be filed for the benefit of the other co-owners and may proceed without impleading
the other co-owners. However, where the co-owner repudiates the co-ownership by claiming sole ownership
of the property or where the suit is brought against a co-owner, his co- owners are indispensable parties and
must be impleaded as party-defendants, as the suit affects the rights and interests of these other co-owners.
In the present case, Anacay never disputed the existence of a co-ownership nor claimed to be the sole or exclusive
owner of the litigated lot. Moreover and more importantly, the respondents claim in his complaint in Civil Case
No. 2919-03 is personal to him and his wife, i.e., that his and his wifes signatures in the Deed of Absolute Sale in
favor of petitioner Josephine were falsified. The issue therefore is falsification, an issue which does not require the
participation of the co-owners at the trial; it can be determined without their presence because they are not parties to
the document; their signatures do not appear therein. Their rights and interests as co-owners are adequately
protected by their co-owner and father, Mr. Anacay, since the complaint was made precisely to recover ownership
and possession of the properties owned in common.

REM Digests ALS 2014B Justice Gesmundo


146. VALDEZ-TALLORIN v. HEIRS OF TIRONA (2009) (RULE 3)
(1) Indispensable parties are those with such an interest in the controversy that a final decree would
necessarily affect their rights, so that the courts cannot proceed without their presence; (2) Non-joinder of
indispensable parties is not a ground for dismissal. Sec 11, Rule 3 prohibits the dismissal of a suit on the
ground of non-joinder or misjoinder of parties and allows the amendment of the complaint at any stage of
the proceedings, through motion or on order of the court on its own initiative. Only if plaintiff refuses to
implead an indispensable party, despite the order of the court, may it dismiss the action.
The Heirs of Tirona filed a case against Tallorin for the cancellation of a tax declaration over a parcel of land under
the name of Tallorin and her co-owners (Valdez and Vda de. Valdez) who were not impleaded in the case. The
heirs alleged that the tax declarations were originally under their fathers name but was illegally issued in favor of
the three women. The RTC annulled the tax declaration in favor of Tallorin and the two other women. On appeal,
Tallorin contended that the RTC should have dismissed the complaint for failure to implead Valdez and Vda. De
Valdez who were indispensable parties, their interests in the tax declaration having been affected by the RTC
judgment.
Joining indispensable parties into an action is mandatory, being a requirement of due process. Without their
presence, the judgment of the court cannot attain real finality. Judgments do not bind strangers to the
suit. The absence of an indispensable party renders all subsequent actions of the court null and void. It is
inevitable that any decision granting what the Taronas wanted would necessarily affect the rights of such
persons to the property covered by the tax declaration. But the Taronas action cannot be dismissed outright
(See doctrine 2 above). The case was remanded to the RTC with an order to implead Vda. de Valdez and
Valdez so that they may be heard.
183. RIOFERIO V. CA (2004)
Heirs may bring suit to recover property of the estate pending the appointment of an administrator.
Orfinada died. He left behind a legitimate family, and a family with his mistress. The common law family executed
an Extrajudicial Settlement of Estate. Some months later, Alfonso Orfinada (son of Orfinada in the legitimate
family) prayed for letters of administration be issued to him. The legitimate family then filed a Complaint for the
Annulment of the Extra Judicial Settlement of Estate executed by the common law family. The common law family
now seeks the dismissal of this complaint stating that the members of the legitimate family are not the proper
parties to bring the action because it is not the heirs, but the estate, through the administrator, which should bring
the case.
The Court held that the heirs (the legitimate family) are allowed to bring the case for recovery of property of the
estate because no administrator has yet been appointed. Pending the filing of administration proceedings, the
heirs without doubt have legal personality to bring suit in behalf of the estate of the decedent. Heirs cannot be
expected to wait for the appointment of an administrator; then wait further to see if the administrator appointed
would care enough to file a suit to protect the rights and the interests of the deceased.
188. Regner v. Cynthia Logarta and Teresa Tormis (October 19, 2007) (Rule 3, Section 7; Parties)
Victoria Regner, the petitioner, is the second wife of Luis. The respondents are Luiss children with his first wife.
When he was alive, Luis acquired shares at Cebu Country Club. He later executed a Deed of Donation in favor of
the respondents. After Luis died, Victoria filed a complaint for declaration of nullity of the deed of donation.
Victoria claimed that Luis had been very ill at the time the donation was made and that the respondents had
fraudulently caused the donation to made. Teresa and Cynthia lived in the US, but occasionally visited the PHL.
Teresa was served summons when she went to the Philippines. Teresa filed her rejoinder against the motion to set
the case for pre-trial on the ground that Cynthia, an indispensable party, had not yet been served summons. Thus,
Teresa prayed for the dismissal of the case, as it would not proceed without Cynthia. The RTC dismissed. A co-
donee is an indispensable party in an action to declare the nullity of the deed of donation. As a rule, if a
defendant has not been summoned, the court acquires no jurisdiction over his person and a personal
judgment rendered against such defendant is null and void. Rule 3, Section 7 defines indispensable parties as
parties-in-interest without whom there can be no final determination of an action. As such, they must be
joined either as plaintiffs or as defendants. The absence of an indispensable party renders all subsequent
actions of the court null and void for want of authority to act, not only as to the absent parties but even as to
REM Digests ALS 2014B Justice Gesmundo
those present. Cynthia and Teresa are indispensable parties to the case, at hand. A judgment in favor of the
defendants would not be conclusive as against the other co-owners not parties to the suit, and thus the
defendant in possession of the property might be harassed by as many succeeding actions of ejectment, as
there might be co-owners of the title asserted against him. The purpose of this provision was to prevent
multiplicity of suits by requiring the person asserting a right against the defendant to include with him,
either as co-plaintiffs or as co-defendants, all persons standing in the same position, so that the whole matter
in dispute may be determined once and for all in one litigation.
As to the summons, there are only four instances wherein a defendant who is a non-resident and is not found
in the country may be served a summons by extraterritorial service: (1) when the action affects the personal
status of the plaintiff; (2) when the action relates to, or the subject of which is property within the
Philippines, on which the defendant claims a lien or an interest, actual or contingent; (3) when the relief
demanded in such action consists, wholly or in part, in excluding the defendant from any interest in property
located in the Philippines; and (4) when the defendant non-residents property has been attached within the
Philippines. In these instances, service of summons may be effected by (a) personal service out of the country,
with leave of court; (b) publication, also with leave of court; or (c) any other manner the court may deem
sufficient. Since this case involves an action in personam, the general rule requires the personal service of
summons on Cynthia within the Philippines. But since Cynthia is a non-resident, not found in the PHL, service of
summons to her must be made either (1) by personal service; (2) by publication in a newspaper of general
circulation in such places and for such time as the court may order, in which case a copy of the summons and order
of the court should be sent by registered mail to the last known address of the defendant; or (3) in any other manner
which the court may deem sufficient. The third mode, like the first two, must be made outside the Philippines,
such as through the Philippine Embassy in the foreign country where Cynthia resides. Service of summons
upon Cynthia was not done by any of the authorized modes

RULE 4

67. Paglaum Devt v. Union Bank (2012)


Rule 4 - Venue
Rule 4, Section 3(b) - When Rule not applicable. This Rule shall not apply (b) Where the parties have
validly agreed in writing before the filing of the action on the exclusive venue thereof. (Emphasis supplied.)
The Real Estate Mortgages were executed by PAGLAUM in favor of Union Bank to secure the credit line extended
by the latter to HealthTech. [The REM had a provision on venue, i.e., Section 9. Venue. The venue of all suits
and actions arising out of or in connection with this Mortgage shall be in Cebu City Metro Manila or in the place
where any of the Mortgaged Properties is located, at the absolute option of the Mortgagee, the xxxxxxxxxxxxx any
other venue.] Subsequently, due to the 1997 Asian Financial Crisis, Healthtech was unable to pay. A restructuring
agreement was entered into, which had a provision 20. Venue Venue of any action or proceeding arising out of
or connected with this Restructuring Agreement, the Note, the Collateral and any and all related documents shall be
in Makati City, [HealthTech] and [Union Bank] hereby waiving any other venue. The property was extrajudicially
foreclosed and sold to Union Bank as the sole bidder. PAGLAUM and Healthtech filed a Complaint for Annulment
of Sale and Titles with Damages and Application for Temporary Restraining Order and Writ of Injunction before
the RTC of NCR, which case was raffled to Br. 134 of Makati. The RTC granted Union Banks motion to dismiss
on the ground of [among others] improper venue.
The only dispute is whether the venue that should be followed is that contained in the Real Estate Mortgages, as
contended by Union Bank, or that in the Restructuring Agreement, as posited by PAGLAUM and HealthTech.
The restructuring agreement controls. It is restrictive and exclusive in accordance with the doctrine in Sps.
Lantin v. Lantion. Makati is the proper venue. Even if of the Court were to consider the venue stipulations
under the Real Estate Mortgages, it must be underscored that those provisions did not contain words
showing exclusivity or restrictiveness. The case is reinstated.

REM Digests ALS 2014B Justice Gesmundo


90. GENEROSA ALMEDA LATORRE v. LUIS ESTEBAN LATORRE (2010)
The nature of an action is determined by the allegations in the Complaint itself, rather than by its title or
heading. It is also a settled rule that what determines the venue of a case is the primary objective for the
filing of the case.
3 modes of appeal from decisions of the RTC, namely: (1) ordinary appeal or appeal by writ of error, where
judgment was rendered in a civil or criminal action by the RTC in the exercise of its original jurisdiction; (2)
petition for review, where judgment was rendered by the RTC in the exercise of its appellate jurisdiction;
and (3) petition for review to the Supreme Court.
A case for the Collection and Declaration of Nullity of Deed of Absolute Sale was filed in RTC Muntinlupa. The
subject property is located in Makati City. Respondent Luis filed a Motion to Dismiss on the ground of improper
venue. Luis contends that while the complaint was denominated as one for Collection and Declaration of Nullity of
Deed of Absolute Sale with application for Injunction, in truth the case was a real action affecting title to and
interest over the subject property. Since the subject property is located in Makati City, respondent argued that
petitioner should have filed the case before the RTC of Makati City and not of Muntinlupa City. RTC of
Muntinlupa dismissed the case for lack of Jurisdiction. Petitioner Generosa filed a Petition for Review on Certiorari
under Rule 45, in relation to Rule 41 to the SC.
According to Sections 1 and 2, Rule 4 of the ROC, actions affecting title to or possession of real property or
an interest therein (real actions) shall be commenced and tried in the proper court that has territorial
jurisdiction over the area where the real property is situated. On the other hand, all other actions (personal
actions) shall be commenced and tried in the proper courts where the plaintiff or any of the principal
plaintiffs resides or where the defendant or any of the principal defendants resides. The action in the RTC,
other than for Collection, was for the Declaration of Nullity of the Deed of Absolute Sale involving the subject
property, which is located at Dasmarias Village, Makati City. The venue for such action is unquestionably the
proper court of Makati City, where the real property or part thereof lies, not the RTC of Muntinlupa City.
The nature of an action is determined by the allegations in the Complaint itself, rather than by its title or
heading. It is also a settled rule that what determines the venue of a case is the primary objective for the
filing of the case. Petitioner's complaint is a real action involving the recovery of the subject property on the basis
of her co-ownership thereof.
The RTC insisted that trial on the merits be conducted even when it was awfully glaring that the venue was
improperly laid, as pointed out by respondent in his motion to dismiss. After trial, the RTC eventually dismissed the
case on the ground of lack of jurisdiction, even as it invoked, as justification, the rules and jurisprudence on venue.
Despite the conduct of trial, the RTC failed to adjudicate this case on the merits.
Finally, in Murillo v. Consul, SC laid down a doctrine that was later adopted by the 1997 Revised Rules of Civil
Procedure. In that case, this Court had the occasion to clarify the three (3) modes of appeal from decisions of the
RTC, namely: (1) ordinary appeal or appeal by writ of error, where judgment was rendered in a civil or
criminal action by the RTC in the exercise of its original jurisdiction; (2) petition for review, where
judgment was rendered by the RTC in the exercise of its appellate jurisdiction; and (3) petition for review to
the Supreme Court.
The first mode of appeal, governed by Rule 41, is brought to the Court of Appeals (CA) on questions of fact or
mixed questions of fact and law. The second mode of appeal, covered by Rule 42, is brought to the CA on
questions of fact, of law, or mixed questions of fact and law. The third mode of appeal, provided in Rule 45, is filed
with the Supreme Court only on questions of law.
A question of law arises when there is doubt as to what the law is on a certain state of facts, while there is a
question of fact when the doubt arises as to the truth or falsity of the alleged facts. For a question to be one of
law, the same must not involve an examination of the probative value of the evidence presented by the
litigants or any of them. The resolution of the issue must rest solely on what the law provides on the given set of
circumstances. Once it is clear that the issue invites a review of the evidence presented, the question posed is one of
fact. Thus, the test of whether a question is one of law or of fact is not the appellation given to such question
by the party raising the same; rather, it is whether the appellate court can determine the issue raised without
reviewing or evaluating the evidence, in which case, it is a question of law; otherwise it is a question of fact.
REM Digests ALS 2014B Justice Gesmundo
Generosa prayed that the SC decide the case on the merits. To do so, however, would require the examination
by the SC of the probative value of the evidence presented, taking into account the fact that the RTC failed to
adjudicate this controversy on the merits. It thus becomes exceedingly clear that the filing of the case directly with
the SC ran afoul of the doctrine of hierarchy of courts. Pursuant to this doctrine, direct resort from the lower
courts to the Supreme Court will not be entertained unless the appropriate remedy sought cannot be
obtained in the lower tribunals. This Court is a court of last resort, and must so remain if it is to
satisfactorily perform the functions assigned to it by the Constitution and by immemorial tradition.
145. SPS. SAMONTE V. CENTURY SAVINGS BANK (2009)
(Rule 4 - no rule 4 related doctrine in the case except that they filed the ejectment case in the MeTC) an
ejectment suit cannot be abated or suspended by the mere filing of another action raising ownership of the
property as an issue. Exception: Equity Considerations
Sps. Samonte obtained a loan from Century secured by a mortgage on their property. Mortgage was extrajudicially
foreclosed and was sold to Century as highest bidder. Sps. Samonte entered into a lease agreement with Century
and retained possession of the property. Sps. Samonte failed to pay rent. Century filed an ejectment case with the
MeTC. The Sps. insisted that the foreclosure proceedings were. They filed a case for the nullification of the
foreclosure proceedings. The MeTC ordered the spouses to vacate the premises. On appeal the Sps. argued that the
ejectment case should have awaited the result of the nullification proceedings.
As a general rule, an ejectment suit cannot be abated or suspended by the mere filing of another action
raising ownership of the property as an issue. The exception is equity considerations, example: demolition of
house and would create confusion, disturbance, inconvenience, and expense. The court would be wasting
much time and effort by proceeding to a stage wherein the outcome would at best be temporary but the
result of enforcement would be permanent, unjust and probably irreparable. Sps. in this case failed to show
that they fall under the exception.
201. INFANTE v. ARAN BUILDERS (2007)
"the proper venue depends on the determination of whether the present action for revival of judgment is a
real action or a personal action (VENUE)"
Aran Builders, Inc. filed before the RTC of Muntinlupa an action for revival of judgment against Infante. The
judgment sought to be revived was based on an action for specific performance and damages which was rendered
by the RTC of Makati. The judgment by RTC of Makati became final and executory. Infante filed a motion to
dismiss the action for revival of judgment on the grounds that the venue was improperly laid.
The proper venue depends on the determination of whether the present action for revival of judgment is a
real action or a personal action. If it is a real action (i.e., action affects title to or possession of real property
or interest therein), then it must be filed with the court of the place where the real property is located. On the
other hand, if it is a personal action, then it may be filed with the court of the place where the plaintiff or
defendant resides.
The RTC of Muntinlupa is the proper venue for the present action. The case at bar involves the enforcement
of the private respondents rights over a piece of real property located at Ayala Alabang Subdivision.
Because it involves a real action, the complaint should indeed be filed with the RTC of the place where the
realty is located (which in this case is Muntinlupa City). Originally, Muntinlupa City was under the
territorial jurisdiction of the Makati courts. However, Section 4 of RA 7154 which amended Section 14 of BP
129 provided for the creation of a branch of the RTC in Muntinlupa. Thus, it is now the RTC in Muntinlupa
City which has territorial jurisdiction or authority to validly issue orders and processes concerning real
property within Muntinlupa City.

RULE 5

REM Digests ALS 2014B Justice Gesmundo


RULE 6

53. MENDIOLA v. CA & SHELL (2012)


Four tests to determine whether counterclaim is compulsory: (1) are the issues of fact or law raised by the
claim and counterclaim largely the same? (2) would res judicata bar subsequent suit on defendants claims,
absent the compulsory counterclaim rule? (3) will substantially the same evidence support or refute
plaintiffs claim as well as defendants counterclaim? (4) is there any logical relation between the claim and
the counterclaim, such that the conduct of separate trials of the respective claims of the parties would entail
a substantial duplication of effort and time by the parties and the court? Of the four, the fourth is the most
compelling test.
For Rules 4, 6, 37
Ramon Mendiola agreed to be the distributor of Shells products, and his obligation was secured by a real estate
mortgage. He defaulted on his payments, so Shell commenced extrajudicial foreclosure of the property. Mendiola
went to the scheduled date of auction, but the auction was held somewhere else. Foreclosure became final, but the
proceeds werent enough to settle the debt, so Shell sued in Manila (Manila case) to collect the deficiency.
Mendiola countered that he had no notice of the auction so the foreclosure was invalid. The trial court sided with
Shell and ordered the payment of the deficiency. Mendiola appealed, and at the same time filed for annulment of
the foreclosure for being invalid in the Makati RTC (Makati case). The Manila case became final and executory.
Shell moved to dismiss the Makati case for being barred by litis pendentia, and later res judicata. Makati RTC ruled
in favor of Mendiola, stating that Manila RTC had no jurisdiction because the property foreclosed was located in
Makati, and the causes of action were not identical. Shell filed for reconsideration, which was denied, so Shell
appealed. Mendiola moved to dismiss the appeal, because (under the old Rules) an order denying a motion for
reconsideration cannot be appealed.
The Court held that the appeal was proper, even under the old rules, because what is proscribed is an appeal from
an order denying an MR of an interlocutory order, not of a final order. The order denying the MR of a final order is
considered a final order subject to an appeal, as clarified by Quelnan v. VHF and Neypes v. CA. What is proscribed
is appealing an order denying an MR of an interlocutory order. (Note that under the 2007 amendments to Rule 41,
an order denying a motion for new trial or reconsideration is no longer part of the list of orders which cannot be
subject of an appeal.)
The Court also held that the Makati case should have been dismissed for res judicata (using the elements of res
judicata). The causes of action were deemed identical, even if one was for annulment and one was for collection,
because of the logical connection between them and the identity of the evidence required. The annulment of the
foreclosure is to be deemed a compulsory counterclaim in the action for collection (using elements of compulsory
counterclaim). Four tests to determine whether counterclaim is compulsory: (1) are the issues of fact or law
raised by the claim and counterclaim largely the same? (2) would res judicata bar subsequent suit on
defendants claims, absent the compulsory counterclaim rule? (3) will substantially the same evidence
support or refute plaintiffs claim as well as defendants counterclaim? (4) is there any logical relation
between the claim and the counterclaim, such that the conduct of separate trials of the respective claims of
the parties would entail a substantial duplication of effort and time by the parties and the court? Of the four,
the fourth is the most compelling test.
Also, Manila RTC had jurisdiction, as the location of the property pertained only to venue, not jurisdiction.
Although venue was improperly laid, there was waiver when the parties did not object.
187. Maranan v. Manila Banking (Section 2 and 3, Rule 10; Section 6 and 14, Rule 6; Pleadings)
Mandarin had obtained a loan from Manila Bank. Maranan was one of the sureties. Manila Banking filed a case
against Maranan + sureties/guarantors for payment, which included Chua and Pacific Enamel. Chua and Pacific
Enamel had the case dismissed against them on the ground that it stated no cause of action against them. Maranan
claimed that in her Answer that the surety agreement did not express the true intent of the surety because Chua was
REM Digests ALS 2014B Justice Gesmundo
the real borrower and Mandarin was merely a conduit. 2 years later, Maranan filed an amended answer impleading
Chua and Pacific Enamel as defendants in her counterclaim. The RTC denied the amended answer because it had
already dismissed the complaint against Chua and Pacific Enamel for lack of cause of action. The issues to be
resolved in this case were whether the amended answer, filed without prior leave of court, should have been
admitted and whether petitioners resort to a counterclaim instead of a third-party complaint was proper.
Maranans Amended Answer contained substantial amendments not found in her original Answer and involved a
significant shift in the theory of the defense. Such Amended Answer was filed after the case was already set for
hearing. It is clear from Sections 2 and 3 of Rule 10 that once a case has already been set for hearing,
regardless of whether a responsive pleading has been served, substantial amendments may only be made
upon leave of court.
Section 6 and 14 of Rule 6 provides that a counterclaim may be filed only against an opposing party. The
filing of a counterclaim against a third party is not allowded, but the court may order such party to be
brought in as defendant.
As per Section 12 of Rule 6, the bringing of a third-party defendant is proper if the latter would be liable to
the plaintiff or to the defendant or both for all or part of the plaintiffs claim against the original defendant,
although the third-party defendants liability arises out of another transaction.
Thus, the SC held that Maranan should have filed, with proper leave of court, a third-party complaint.

RULE 7

13. BORRA, ET AL. v. COURT OF APPEALS (2013)


Only the identities of parties and issues are required for the operation of the principle of conclusiveness of
judgment (res judicata). There can be no forum shopping if the grounds cited by in in motions to dismiss filed
in a case and in another case are different.
Borra and several others filed a complaint with the NLRC against Hawaiian Philippine Company for recognition as
regular employees and other labor-related demands. HPC tried to dismiss the case on the ground of res judicata,
citing an earlier case (Perez v. HPC) involving essentially the same parties, wherein it was held that Borra and the
others are not employees of HPC but rather of some other guy. The Labor Arbiter did not dismiss the case. The CA
reversed and ordered the dismissal of the labor case, prompting Borra and his friends to go to the SC, even accusing
HPC of forum-shopping.
First, the SC held that insofar as the question of employer and employee relations is concerned, the final
judgment in Perez has the effect and authority of res judicata by conclusiveness of judgment. Conclusiveness
of judgment finds application when a fact or question has been squarely put in issue, judicially passed upon,
and adjudged in a former suit by a court of competent jurisdiction; only the identities of parties and issues
are required for the operation of the principle of conclusiveness of judgment. Second, there can be no forum
shopping, because the grounds cited by HPC in its motions to dismiss filed in the previous and present cases
are different. Further, the relief prayed for in HPCs motion to dismiss subject of the instant case is founded
on totally different facts and issues.
20. LUCENA B. RALLOS V. CITY OF CEBU (2013)
Forum shopping is the act of the litigants who respectively avail themselves of multiple judicial remedies in
different fora or simultaneous or successively, all substantially founded on the same transactions and the
same essential facts and circumstances; and raising substantially similar issues either pending in or already
resolved adversely by some other court.
Certain parcels of land were expropriated by the City of Cebu to be used as a public road. The Heirs of the alleged
owner of the lots filed for Forfeiture or Payment of FMV against the City. Judgment was rendered in the Heirs
favor. A Petition for Certiorari and Mandamus was again filed by the Heirs against the RTCs Order which they
alleged to have effectively modified the aforementioned decision. Subsequently, petitioner filed for indirect

REM Digests ALS 2014B Justice Gesmundo


contempt against the respondents alleging that the latter were impeding the execution of the first decision. Petioner
is guilty of forum shopping. Forum shopping exists when the elements of litis pendentia are present or where
a final judgment in one case will amount to res judicata in another. Stripped of unnecessary details, the
reliefs sought in both cases are founded on the same set of facts. Moreover, the citation for indirect contempt
would amount to res judicata in the other considering the identities of the parties and the issues involved.
The penalty is summary dismissal of both cases.
36. Lim v. Co (2012) (Forum shopping)
There is NO forum shopping where a private complainant to pursue a civil complaint for specific
performance and damages while appealing the judgment on the civil aspect of a criminal case for estafa.
Because of the distinct and independent nature of the two kinds of civil liabilities, jurisprudence holds that
the offended party may pursue the two types of civil liabilities simultaneously and cumulatively, without
offending the rules on forum shopping, litis pendentia, or res judicata.
The essence of forum shopping is the filing of multiple suits involving the same cause of action, either
simultaneously or successively, to secure a favorable judgment. Although the cases filed by the offended
party arose from the same act or omission of the offender, they are, however, based on different causes of
action. (civil action ex delicto and a civil action arising from a contractual obligation and tortious conduct.)
75. VALLACAR TRANSIT INC., vs. JOCELYN CATUBIG , G.R. No. 175512, (MAY 30, 2011);
CERTIFICATE OF AGENCY, VERIFICATION, Section 3, Rule 7
Petitioner asserts that respondents complaint for damages should be dismissed for the latters failure to verify the
same. The certification against forum shopping attached to the complaint, signed by respondent, is not a valid
substitute for respondents verification that she has read the pleading and that the allegations therein are true and
correct of her personal knowledge or based on authentic records.
Respondent filed her complaint for damages against petitioner on July 19, 1995, when the 1964 Rules of Court was
still in effect. Rule 7, Section 6 of the 1964 Rules of Court provided:
SEC. 6. Verification.A pleading is verified only by an affidavit stating that the person
verifying has read the pleading and that the allegations thereof are true of his own knowledge.
Verifications based on "information and belief," or upon "knowledge, information and belief,"
shall be deemed insufficient.
On July 1, 1997, the new rules on civil procedure took effect. The foregoing provision was carried on, with a few
amendments, as Rule 7, Section 4 of the 1997 Rules of Court, viz:
SEC. 4. Verification. Except when otherwise specifically required by law or rule, pleadings
need not be under oath, verified or accompanied by affidavit.
A pleading is verified by an affidavit that the affiant has read the pleading and that the
allegations therein are true and correct of his knowledge and belief.
A pleading required to be verified which contains a verification based on information and
belief, or upon knowledge, information and belief, or lacks a proper verification, shall be
treated as an unsigned pleading.
The same provision was again amended by A.M. No. 00-2-10, which became effective on May 1, 2000. It now
reads:
SEC. 4. Verification. - Except when otherwise specifically required by law or rule, pleadings
need not be under oath, verified or accompanied by affidavit.
A pleading is verified by an affidavit that the affiant has read the pleading and that the
allegations therein are true and correct of his personal knowledge or based on authentic
records.

REM Digests ALS 2014B Justice Gesmundo


A pleading required to be verified which contains a verification based on information and
belief or upon knowledge, information and belief, or lacks a proper verification, shall be
treated as an unsigned pleading.
The 1997 Rules of Court, even prior to its amendment by A.M. No. 00-2-10, clearly provides that a pleading
lacking proper verification is to be treated as an unsigned pleading which produces no legal effect. However, it also
just as clearly states that [e]xcept when otherwise specifically required by law or rule, pleadings need not be under
oath, verified or accompanied by affidavit. No such law or rule specifically requires that respondents complaint
for damages should have been verified.
A partys failure to sign the certification against forum shopping is different from the partys failure to sign
personally the verification. The certificate of non-forum shopping must be signed by the party, and not by
counsel. The certification of counsel renders the petition defective.On the other hand, the requirement on
verification of a pleading is a formal and not a jurisdictional requisite. It is intended simply to secure an assurance
that what are alleged in the pleading are true and correct and not the product of the imagination or a matter of
speculation, and that the pleading is filed in good faith. The party need not sign the verification. A partys
representative, lawyer or any person who personally knows the truth of the facts alleged in the pleading may sign
the verification.
78. Myrna P. Anotne v. Leo R. Beronilla (2010)
Proper verification; Legal standing Improper verification is merely a formal, not a jurisdictional defect. The
Court may relax the rule in order that the ends of justice may be served. The OSG has the authority to file
petitions with the CA and SC and not the Prosecutor. This rule, however, may be relaxed as in the present
case.
Wife filed bigamy case against husband. Motion to Quash was granted, case was dismissed. A petition for
certiorari under Rule 65 was filed with the CA. The CA dismissed the petition because: 1. verification was
defective as it did not include the assurance that the allegations in the petition are based on authentic records; 2. It
was the Prosecutor, not the OSG, in behalf of the People of the Philippines who filed the petition.
The Rules of Court provides that a pleading required to be verified which lacks a proper verification shall be
treated as unsigned pleading. This, notwithstanding, we have, in a number of cases, opted to relax the rule in
order that the ends of justice may be served. The defect being merely formal and not jurisdictional, we ruled
that the court may nevertheless order the correction of the pleading, or even act on the pleading "if the
attending circumstances are such that xxx strict compliance with the rule may be dispensed with in order
that the ends of justice xxx may be served." At any rate, a pleading is required to be verified only to ensure
that it was prepared in good faith, and that the allegations were true and correct and not based on mere
speculations.
Prosecutor is limited to the proceedings in the trial court. OSG is the one with authority to represent the
Government before the CA or SC. General rule: case may be summarily dismissed if it is not the OSG who
initiates the petition with the CA or SC. Exception: for justice to prevail, the scales must balance, for justice
is not to be dispensed for the accused alone." OSG may just be required to file a comment on the petition.
80. and 81. Pentacapital Investment Corporation v. Makalito B. Mahinay (2010)
Res judicata; Forum-shopping It is well-settled that when material facts or questions in issue in a former
action were conclusively settled by a judgment rendered therein, such facts or questions constitute res
judicata and may not again be litigated in a subsequent action between the same parties or their privies
regardless of the form of the latter. What is important in determining whether forum-shopping exists is
the vexation caused the courts and parties-litigants by a party who asks different courts and/or
administrative agencies to rule on the same or related causes and/or grant the same or substantially the same
reliefs, in the process creating the possibility of conflicting decisions being rendered by the different fora
upon the same issues. There is no forum shopping where the first case originated from an interlocutory
order of the RTC, while the second case is an appeal from the decision of the court on the merits of the case.
Pentacapital filed a complaint for sum of money against Mahinay based on two separate loans but these loans were
denied by Mahinay and said that these loans were not perfected for lack of consideration. Prior to this action,
respondent instituted an action for Preliminary Mandatory Injunction against Pentacapital Realty, before the RTC
REM Digests ALS 2014B Justice Gesmundo
of Cebu City. On motion of Pentacapital Realty, the court dismissed the complaint on two grounds: 1) non-payment
of the correct filing fee considering that the complaint was actually a collection of sum of money although
denominated as Preliminary Mandatory Injunction; and 2) lack of cause of action. The court treated the complaint
as a collection suit because respondent was seeking the payment of his unpaid commission or share in the proceeds
of the sale of the Molino Properties. Additionally, the RTC found that respondent had no cause of action against
Pentacapital, there being no privity of contract between them. Lastly, the court held that it was CRDI which agreed
that 20% of the total consideration of the sale be paid and delivered to respondent. Instead of assailing the said
Order, respondent filed his supplemental compulsory counterclaim, demanding again the payment of his
commission, this time, against petitioner in the instant case. The Order, therefore, became final and executory. The
RTC ruled in favor of Mahinay finding instead, Pentacapital to be liable and found that the award of the
supplemental compulsory counterclaim proper. CA affirmed the RTC ruling. Pentacapital argued that there was res
judicata in the prior case dismissed by the other court.
It is well-settled that when material facts or questions in issue in a former action were conclusively settled by
a judgment rendered therein, such facts or questions constitute res judicata and may not again be litigated in
a subsequent action between the same parties or their privies regardless of the form of the latter. Absolute
identity of parties is not required, and where a shared identity of interest is shown by the identity of the relief
sought by one person in a prior case and the second person in a subsequent case, such was deemed sufficient.
What is important in determining whether forum-shopping exists is the vexation caused the courts and
parties-litigants by a party who asks different courts and/or administrative agencies to rule on the same or
related causes and/or grant the same or substantially the same reliefs, in the process creating the possibility
of conflicting decisions being rendered by the different fora upon the same issues.
These elements are not present in this case. In the first case, petitioner assails the propriety of the admission
of respondents supplemental compulsory counterclaim; while in the present case, petitioner assails the grant
of respondents supplemental compulsory counterclaim. In other words, the first case originated from an
interlocutory order of the RTC, while the second case is an appeal from the decision of the court on the
merits of the case. There is, therefore, no forum-shopping for the simple reason that the petition and the
appeal involve two different and distinct issues.
85. Mayor Quintin B. Saludaga v. COMELEC and Artemio Balag, G.R. Nos. 189341 & 191120 (2010) Rule
7; Rule 39
1. Under paragraph 2, Section 5, Rule 7 of the 1997 Rules of Civil Procedure, as amended, if the acts of the
party or his counsel clearly constitute willful and deliberate forum shopping, the same shall be ground
for summary dismissal with prejudice and shall constitute direct contempt, as well as a cause for
administrative sanctions.
2. A writ of execution pending resolution of the motion for reconsideration of a decision of the division is
not granted as a matter of right.
Petitioner Saludaga and Respondent Balag were candidates for Mayor of Lavezares, Northern Samar in the May 14,
2007 elections. The Municipal Board of Canvassers proclaimed Saludaga as winner. Balag filed an election protest
against Saludaga. Saludaga, too, filed a counter-protest. The RTC declared Balag as the winning mayoralty
candidate. Saludaga appealed to the COMELEC. Balag moved for execution pending appeal, but the motion was
denied in an Order. The trial court found no special reason to warrant execution pending appeal, opining that
Balags victory in the polls had not been clearly established. The COMELEC, Second Division, affirmed with
modification the Decision of the RTC. Balag again promptly moved for execution and filed a Motion for Execution
Pending Motion for Reconsideration. Saludaga, filed a Verified MR of the COMELEC Resolution. The Second
Division of COMELEC issued an order granting Balags Motion for Execution Pending Motion for
Reconsideration. Aggrieved, Saludaga filed an Extremely Urgent MR with the COMELEC en banc. Later,
Saludaga also filed a Petition for Certiorari with the SC, challenging the Order of the COMELEC, Second
Division. Thus, Balag filed a Manifestation and Motion to Dismiss with the COMELEC en banc contending that
Saludaga engaged in forum shopping. Balag in the meantime had taken his oath and assumed the post of Mayor of
Lavezares, Northern Samar.

REM Digests ALS 2014B Justice Gesmundo


Forum shopping is the institution of two (2) or more suits in different courts, either simultaneously or
successively, in order to ask the courts to rule on the same or related causes and/or to grant the same or
substantially the same reliefs. There is forum shopping when as a result of an adverse decision in one (1)
forum, or in anticipation thereof, a party seeks favorable opinion in another forum through means other
than appeal or certiorari.
Under paragraph 2, Section 5, Rule 7 of the 1997 Rules of Civil Procedure, as amended, if the acts of the
party or his counsel clearly constitute willful and deliberate forum shopping, the same shall be ground for
summary dismissal with prejudice and shall constitute direct contempt, as well as a cause for administrative
sanctions.
In determining whether a party violated the rule against forum shopping, the most important factor to ask is
whether the elements of litis pendencia are present, or whether a final judgment in one case will amount to
res judicata in another. For the principle of res judicata to apply, the following elements must be present: (1)
the judgment sought to bar the new action must be final; (2) the decision must have been rendered by a court
having jurisdiction over the subject matter and the parties; (3) the disposition of the case must be a
judgment on the merits; and (4) there must be as between the first and second actions, identity of parties,
subject matter, and cause of action. In the present case, the second element is wanting.
Furthermore, the discretion to allow execution pending reconsideration belongs to the division that rendered
the assailed decision, order or resolution, or the COMELEC en banc, as the case may benot to the
Presiding Commissioner. To be sure, a writ of execution pending resolution of the motion for reconsideration
of a decision of the division is not granted as a matter of right such that its issuance becomes a ministerial
duty that may be dispensed even just by the Presiding Commissioner.
88. Mediserv, Inc. v. CA, G.R. No. 161368 (2010) Rule 45/46; Rule 7
1. A pleading is verified by an affidavit that the affiant has read the pleading and that the allegations
therein are true and correct of his personal knowledge or based on authentic records. The party need not
sign the verification. A partys representative, lawyer or any person who personally knows the truth of the
facts alleged in the pleading may sign the verification.
2. The SC has allowed the belated filing of the certification against forum shopping, with more reason should
it allow the timely submission of such certification though proof of the signatorys authority was submitted
thereafter.
Petitioner Mediserv, Inc. executed a real estate mortgage in favor of China Banking Corporation as security for a
loan. Mediserv defaulted on its obligation with Chinabank and the real estate mortgage was foreclosed. At the
public auction sale, private respondent Landheights Development Corporation emerged as the highest bidder.
Landheights filed with the RTC of Manila an Application for Possession of Real Estate Property Purchased at an
Auction Sale under Act No. 3135. The title of the property was consolidated in favor of Landheights, and seeking
to recover possession of the subject property, it filed a verified complaint for ejectment against Mediserv before the
MeTC of Manila. Said court rendered a decision in favor of Landheights. Mediserv appealed to the RTC which
reversed the ruling. Landheights MR was denied. Accordingly, it filed a Petition for Review with the CA which
however dismissed the petition. Landheights seasonably filed an MR and subsequently submitted a Secretarys
Certificate stating that the Board of Directors affirms the authority of Mr. Dickson Tan to file the Petition for
Review.
The CA granted Landheights a new period of ten (10) days within which to correct and rectify the deficiencies in
the petition. Mediserv filed an MR and on the same date, Landheights filed its Manifestation of Compliance. The
CA reinstated the petition for review. Mediserv argues that from the beginning, the CA found the petition filed
before it to be defective for failure to comply with the rules. It points out that there is no showing that the
respondent corporation, through its board of directors, had authorized its Corporate Secretary to file the petition for
review in its behalf and to sign the verification and certification against forum-shopping. However, instead of
upholding the dismissal of the petition, the CA allowed private respondent to rectify its deficiency, which is
contrary to jurisprudence.
The SC is not persuaded. Under Rule 46, Section 3, paragraph 3 of the 1997 Rules of Civil Procedure, as amended,
petitions for certiorari must be verified and accompanied by a sworn certification of non-forum shopping. A
REM Digests ALS 2014B Justice Gesmundo
pleading is verified by an affidavit that the affiant has read the pleading and that the allegations therein are true and
correct of his personal knowledge or based on authentic records.The party need not sign the verification. A partys
representative, lawyer or any person who personally knows the truth of the facts alleged in the pleading may sign
the verification.
On the other hand, a certification of non-forum shopping is a certification under oath by the plaintiff or principal
party in the complaint or other initiatory pleading asserting a claim for relief or in a sworn certification annexed
thereto and simultaneously filed therewith, (a) that he has not theretofore commenced any action or filed any claim
involving the same issues in any court, tribunal or quasi-judicial agency and, to the best of his knowledge, no such
other action or claim is pending therein; (b) if there is such other pending action or claim, a complete statement of
the present status thereof; and (c) if he should thereafter learn that the same or similar action or claim has been filed
or is pending, he shall report that fact within five (5) days therefrom to the court wherein his aforesaid complaint or
initiatory pleading has been filed.
The requirement that a petitioner or principal party should sign the certificate of non-forum shopping
applies even to corporations, considering that the mandatory directives of the Rules of Court make no
distinction between natural and juridical persons. A corporation, however, exercises its powers through its
board of directors and/or its duly authorized officers and agents. Unquestionably, there is sufficient
jurisprudential basis to hold that Landheights has substantially complied with the verification and
certification requirements. The SC in several cases held that there is substantial compliance with the Rules
of Court when there is a belated submission or filing of the secretarys certificate through a motion for
reconsideration of the Court of Appeals decision dismissing the petition for certiorari.
105. COMMISSIONER ON APPOINTMENTS V. PALER ( MARCH 3, 2010) Rule 7
There was no need for the chairman of the commission himself to sign the verification; With regard to the
certification of non-forum shopping, the established rule is that it must be executed by the plaintiff or any of
the principal parties and not by counsel. Respondent Paler questions the authority of the Commission Secretary
to file the petition and sign the the verification and non-forum shopping in behalf of the Commission Chairman.
The petitioner in this case is the Commission on Appointments, a government entity created by the
Constitution, and headed by its Chairman. There was no need for the Chairman himself to sign the
verification. Its representative, lawyer or any person who personally knew the truth of the facts alleged in
the petition could sign the verification. With regard, however, to the certification of non-forum shopping, the
established rule is that it must be executed by the plaintiff or any of the principal parties and not by counsel.
In this case, Atty. Tiu failed to show that he was specifically authorized by the Chairman to sign the
certification of non-forum shopping, much less file the petition in his behalf. There is nothing on record to
prove such authority. Atty. Tiu did not even bother to controvert Palers allegation of his lack of authority.
This renders the petition dismissible.Furthermore, the petition is bereft of merit as it merely restates the
arguments presented before the CSC and CA. It does not advance any cogent reason that will convince this
Court to deviate from the rulings of both tribunals.
106. PARENTS TEACHER ASSOCIATION (PTA) OF ST. MATTHEW CHRISTIAN ACADEMY v.
METROPOLITAN BANK AND TRUST CO. (MARCH 2, 2010) Rule 7
A Non-forum shopping is required only in a complaint or a petition which is an initiatory pleading
Petitioners claim that the lack of authority to sign the certificate on non-forum shopping attached to the Petition for
the Issuance of the Writ of Possession rendered the same worthless and should be deemed as non-existent.
Respondent MBTC asserts otherwise, citing Spouses Arquiza v. Court of Appeals where we held that an application
for a writ of possession is a mere incident in the registration proceeding which is in substance merely a motion, and
therefore does not require such a certification.
Petioners contention lack basis.It bears stressing that a certification on non-forum shopping is required only
in a complaint or a petition which is an initiatory pleading. In this case, the subject petition for the issuance
of a writ of possession filed by private respondent is not an initiatory pleading. Although private respondent
denominated its pleading as a petition, it is more properly a motion. What distinguishes a motion from a
petition or other pleading is not its form or the title given by the party executing it, but its purpose. The

REM Digests ALS 2014B Justice Gesmundo


purpose of a motion is not to initiate litigation, but to bring up a matter arising in the progress of the case
where the motion is filed.
118. GENATO vs VIOLA
where there is a conflict between the title of the case and the allegations in the complaint, the latter will
prevail in determining the parties to the action
Put in issue is whether the HLURB acquired jurisdiction over Viola. Violas name did not appear in the title as a
party, but she was one of the persons who caused the preparation of the complaint and who verified the same. She
was set forth in the body of the complaint as a complainant. It is not the caption of the pleading but the
allegations therein that are controlling. Viola is a party to the case. Also, only her name was omitted in the title
but she was already a party to the case and was, more importantly, heard through counsel who she herself chose to
prepare the complaint and represent them in the case before the HLURB. There is no unfairness or surprise in
allowing the amendment to merely correct a technical error. The inclusion of the names of all parties in the title
of the complaint is a formal requirement under Sec. 3 Rule 7 of the Rules of Court. However, the non-
inclusion of one or some of the names of all complainants in the title of a complaint is not fatal to the case,
provided there is a statement in the body of the complaint indicating that such complainants was/were made
party to such action. This is specifically true before the HLURB where the proceedings are summary in nature
without regard to legal technicalities in courts of law and where the pertinent interest is to assist parties in obtaining
just, speedy and inexpensive determination of every action.
120. MID-PASIG LAND DEVELOPMENT CORPORATION vs TABLANTE
TOPIC: Rule 7
Mid-Pasig Development (MPD) owns land. MPD, through its President, Ronaldo Salonga, leased to ECRM
Enterprises, represented by Mario Tablante, and area of the land for 3 months to be used as the staging area for the
Home Garden Exhibition Fair. Upon expiration of the agreement, Tablante assigned his rights to Rockland
Construction. Rockland tried to compel MPD to execution another 3-year lease contract. The RTC noted that the
contract of lease was renewed before the expiration of the lease period, and appellant consented to the renewal and
assignment of the lease. A petition for certiorari was filed with the CA. The CA resolved to dismiss the petition on
the grounds that (a) the verification and certification against non-forum shopping was signed by Antonio Merelos
as General Manager of the petitioner-corporation without attaching a Corporate Secretarys certificate or board
resolution that he is authorized to sign for and on behalf of the petitioner; and (b) Lack of pertinent and necessary
documents which are material portions of the record as required by the Rules. The failure to attach the Secretarys
Certificate, attesting to General Manager Antonio Merelos authority to sign the Verification and Certification of
Non-Forum Shopping, should not be considered fatal to the filing of the petition. Nonetheless, the requisite board
resolution was subsequently submitted to the CA, together with the pertinent documents. Considering that
petitioner substantially complied with the rules, the dismissal of the petition was, therefore, unwarranted. Dismissal
of an appeal on a purely technical ground is frowned upon especially if it will result in unfairness. The rules
of procedure ought not to be applied in a very rigid, technical sense for they have been adopted to help
secure, not override, substantial justice. For this reason, courts must proceed with caution so as not to deprive a
party of statutory appeal; rather, they must ensure that all litigants are granted the amplest opportunity for the
proper and just ventilation of their causes, free from the constraint of technicalities.
132. Spouses Barias v. Heirs of Bartolome (2009) Rule 7
The test in determining the presence of forum shopping is whether in two or more cases pending, there is
identity of (1) parties, (2) rights or causes of action, and (3) reliefs sought.
Heirs of Boneo (the heirs) are registered owners of a parcel of land. The heirs allege that Sps. Barias have been
occupying a portion of the property on mere tolerance and that despite demands, Sps. Barias refused to vacate. The
heirs file a complaint for unlawful detainer. Sps. Barias allege that the heirs are guilty of forum shopping because
the portion of the property subject of the complaint was also the subject of a case between Sps. Barias and the heirs
predecessor-in-interest Silvestra Boneo, which is currently pending appeal before the CA. Sps. Barias also claims
that the mother of petitioner bought a portion of the property from Silvestra. MTCC dismissed on the ground of
forum shopping. In the RTC, the heirs denied that they are Silvestras successors-in-interest, she being the second
wife of their grandfather. RTC, although not finding the heirs guilty of forum shopping, nevertheless dismissed the
REM Digests ALS 2014B Justice Gesmundo
appeal, holding that the heirs have a superior right to possess the property. The CA reversed the RTC decision,
stating that the heirs right to the possession of the property should be upheld because their claim is predicated upon
a Free Patent, which has the force and effect of a Torrens Title and cannot be collaterally attacked as Sps. Barias are
trying to do.
The test in determining the presence of forum shopping is whether in two or more cases pending, there is
identity of (1) parties, (2) rights or causes of action, and (3) reliefs sought. The case filed by Silvestra was for
annulment of the deed of sale while that filed by the heirs was for unlawful detainer. The cause of action for
the first case is the alleged fraud in inducing Silvestra to execute the deed of sale, while the cause of action of
the second is the alleged unlawful possession of the petitioners allegedly sold by Silvestra. The reliefs sought
were also different. In unlawful detainer, the sole issue is the physical possession. The parties can raise the
issue of ownership but the court may only pass upon the same in order to determine who has the right to
possess the property. The adjudication of ownership is merely provisional. As both parties raise the issue of
ownership, its resolution boils down to which of their respective documentary evidence deserves more
weight. The heirs Torrens Title issued in 1991 gives them superior right to possess over Sps. Barias deed of
sale in 1994.
184. HEIRS OF ARCILLA V. TEODORO (2008)
Belated filing of a sworn certification of non-forum shopping is substantial compliance with rules.
Teodoro and the heirs of Arcilla (HA) are arguing over the ownership of a parcel of land. Each of the parties claim
to have inherited the land from their respective predecessors in interest. Teodoro filed an application for land
registration in 1995. Trial of the case ensued. Subsequently, in 1998. Teodoro filed a Motion for Admission
contending that through oversight and inadvertence she failed to include in her application, the verification and
certificate against forum shopping required by Supreme Court (SC) Revised Circular No. 28- 91 in relation to SC
Administrative Circular No. 04-94. HA filed a Motion to Dismiss Application saying that the certification of non-
forum shopping should have been filed simultaneously with the petition for land registration and such failure to
include the certification should be a cause for dismissal of the application for registration.
The Court ruled that the belated filing of the certification of non-forum shopping is substantial compliance
with the rules, even if there was a delay of more than two years. It must be kept in mind that while the
requirement of the certificate of non-forum shopping is mandatory, nonetheless the requirement must not be
interpreted too literally and thus defeat the objective of preventing the undesirable practice of forum shopping.
Teodoro had no intention to violate the Rules with impunity, as she was the one who invited the attention of the
court to the inadvertence committed by her counsel, should be deemed as special circumstances or compelling
reasons to decide the case on the merits.
185. Spouses Melo v. CA (November 16, 1999) (Rule 7, Section 5 Verification, Forum Shopping Certification)
Arsenia Coronel mortgaged to the Rural Bank of Mabalacat a parcel of land to secure a loan. The bank caused the
extra-judicial foreclosure of the mortgage. The Spouses Melo bought the land. Spouses Melo filed a petition for the
ex-parte issuance of a writ of possession with RTC Branch 60. Coronel filed a complaint for injunction with RTC
Branch 57. Petitioners moved to dismiss this action on the following grounds: (1) litis pendentia, (2) forum
shopping, and (3) failure of Coronel to attach a certification of non-forum shopping to her complaint. Coronel then
amended her complaint by including the certification of non-forum shopping. RTC Branch 37 did not dismiss the
case. While Coronel did not commit forum shopping since the two cases involved different causes of action
(petitioners sought possession of the property, whereas Coronel sought to enjoin them from consolidating title over
the same), Coronels case should be dismissed because she failed to comply with the requirements of
Administrative Circular No. 09-93 on non-forum shopping. The requirement to file a certificate of non-forum
shopping is mandatory. Failure to comply with this requirement cannot be excused by the fact that plaintiff
is not guilty of forum shopping. The Circular applies to any complaint, petition, application, or other
initiatory pleading, regardless of whether the party filing it has actually committed forum shopping.
Compliance with the certification against forum shopping is separate from and independent of the avoidance
of forum shopping itself. The former is merely a cause of the dismissal, without prejudice, of the complaint or
initiatory pleading, while the latter is a ground for summary dismissal and constitutes direct contempt. Nor can
subsequent compliance with the requirement excuse a partys failure to comply in the first instance. With
respect to the contents of the certification which the pleader may prepare, the rule of substantial compliance
REM Digests ALS 2014B Justice Gesmundo
may be availed of. While the Circular requires that it be strictly complied with, it merely underscores its
mandatory nature in that it cannot be altogether dispensed with or its requirements completely disregarded
but it does not thereby prevent substantial compliance on this aspect of its provisions under justifiable
circumstances. In the case at bar, Coronel failed to give any reason for her failure to submit the certificate in
question.

RULE 8

1. Michelle Lana Brown-Araneta, et. al v. Juan Ignacio Araneta (October 9, 2013) (Rule 8, Section 5 Forum-
shopping)
The SC held that Michelle had committed forum-shopping when she filed with the Muntinlupa RTC a
petition for protection order while the petition for custody with the Makati RTC was still pending, there
being identity of parties and interests.
Spouses Araneta separated and the kids remained in Michelles custody. Thereafter, Juan Ignacio filed a Petition
for the Custody of the kids with the Makati RTC. Eventually, Michelle, in her Answer, asked for a Temporary
Protection Order and the dismissal of Juan Ignacios petition for custody based on the fact that Juan Ignacio
allegedly was a drug user, sexual pervert, emotionally unstable, and temperamental. The Makati Court refused to
grant the TPO, claiming that it was not inclined to issue a TPO in favor of Michelle because she did not bother to
appear in Court. Michelle thereafter withdrew her motion for TPO. Then, she filed a separate case for a Petition for
Protection Order with the Muntinlupa RTC, again claiming that Juan Ignacio was a sexual pervert. In the
verification position of her petition for protection order, Michelle stated that there is a pending petition for the
custody of their kids. The Muntinlupa RTC granted Michelles prayer for a TPO. Juan Ignacio alleged that
Michelles act of filing with the Muntinlupa RTC constitutes forum shopping.
Michelle committed forum-shopping. The test for determining whether a litigant violated the rule against forum
shopping is where the elements of litis pendentia are present or where a final judgment in one case will amount to
res judicata in the other. Thus, it has been held that there is forum shopping (1) whenever as a result of an
adverse decision in one forum, a party seeks a favorable decision (other than by appeal or certiorari) in
another; or (2) if, after he has filed a petition before the SC, a party files another before the CA since in such
case said party deliberately splits appeals in the hope that even as one case in which a particular remedy is
sought is dismissed, another case (offering a similar remedy) would still be open; or (3) where a party
attempts to obtain a preliminary injunction in another court after failing to obtain it from the original court.
As a result or in anticipation of an adverse ruling of the Makati RTC, Michelle sought the favorable opinion of the
Muntinlupa RTC. Absolute identity of parties is not required, it being enough that there is substantial identity
of the parties or at least such parties represent the same interests in both actions. That a party is the petitioner
in one case and at the same time, the respondent in the other case does not, without more, remove the said cases
from the ambit of the rules on forum shopping. The rights asserted and reliefs prayed for are based on the same
facts, considering that both cases call for the application of the best interest of the child doctrine.
61. ASIAN CONSTRUCTION AND DEVELOPMENT CORPORATION v. LOURDES K. MENDOZA
(2012) [Rule 8, Section 7]
Charge invoices are not actionable documents. A document is actionable when an action or defense is
grounded upon such written instrument or document.
Mendoza, owner of Highett Steel Fabricators, filed a complaint for sum of money against ACDC alleging that the
latter purchased from Highett various fabricated steel materials and supplies amounting to P1,206,177 and failed to
pay. RTC ruled in favor of Mendoza. CA affirmed. In the SC, ACDC argues that a charge or sales invoice is not an
actionable document; thus, its failure to deny under oath its genuineness and due execution does not constitute an
admission thereof.
Held: The charge invoices are not actionable documents. Based on Sec. 7 of Rule 8 of the Rules of Court, a
document is actionable when an action or defense is grounded upon such written instrument or document. In
the instant case, the Charge Invoices are not actionable documents per se as these only provide details on
REM Digests ALS 2014B Justice Gesmundo
the alleged transactions. These documents need not be attached to or stated in the complaint as these are
evidentiary in nature. In fact, Mendozas cause of action is not based on these documents but on the contract of
sale between the parties.
77. Philippine Bank of Communications v. Spouses Go (2011)
Rule 8, Sec. 10 - Specific Denial; Summary Judgment - To specifically deny a material allegation, a defendant
must specify each material allegation of fact the truth of which he does not admit, and whenever practicable,
shall set forth the substance of the matters upon which he relies to support his denial. Under the Rules,
summary judgment is appropriate when there are no genuine issues of fact which call for the presentation of
evidence in a full-blown trial.
Spouses Go obtained loans from PBCom secured by shares of stocks. Two years later, the price of the stocks
plunged prompting PBCom to notify the Spouses Go of the renunciation of the security. PBCom then filed a
Complaint for sum of money. In their Answer, the Spouses denied the material allegations of the Complaint by
alleging that the loan was not yet due and demandable and no demand has been made by PBCom. PBCom,
however, moved for summary judgment alleging otherwise (that Answer admits the fact of default). RTC granted
PBComs motion for summary judgment and ordered the Spouses to pay the loan. The CA, finding that Spouses did
not admit the material allegations, reversed and set aside the summary judgment.
Under the Rules, summary judgment is appropriate when there are no genuine issues of fact which call for
the presentation of evidence in a full-blown trial. Even if on their face the pleadings appear to raise issues,
when the affidavits, depositions and admissions show that such issues are not genuine, then summary
judgment as prescribed by the Rules must ensue as a matter of law. The determinative factor, therefore, in a
motion for summary judgment, is the presence or absence of a genuine issue as to any material fact. A
"genuine issue" is an issue of fact which requires the presentation of evidence as distinguished from a sham,
fictitious, contrived or false claim.
To specifically deny a material allegation, a defendant must specify each material allegation of fact the truth
of which he does not admit, and whenever practicable, shall set forth the substance of the matters upon
which he relies to support his denial. Where a defendant desires to deny only a part of an averment, he shall
specify so much of it as is true and material and shall deny only the remainder. Where a defendant is without
knowledge or information sufficient to form a belief as to the truth of a material averment made in the
complaint, he shall so state, and this shall have the effect of a denial.
79. GSIS v. Dinnah Villaviza (2010)
Rule 8, Sec. 11 Though the Rules of Court apply in a suppletory character in administrative cases, the
Rules will only apply where there is an insufficiency in the applicable rule (specific rule/charter of the
agency). Failure to file an Answer in an administrative case with the GSIS does not amount to an
admission.
Several GSIS employees were charged for Grave Misconduct in the Office of the President of GSIS. They were
directed to submit their written answers under oath but failed to do so. Thus, they were found guilty of the charges
with penalty of suspension. On appeal with the CSC, the CSC found them to be guilty of a lesser offense, ruling
that respondents were not denied due process but there was no substantial evidence to hold them guilty of the
greater offense. GSIS argue that there being no answers, the allegations in the formal charges should have been
deemed admitted pursuant to Rule 8, Sec. 11 which provides that allegations not specifically denied deemed
admitted.
This rule is not applicable in the present case. Special rules are found under the GSIS law which requires
GSIS, in case of failure to file an answer, to render judgment as may be warranted by the facts and
evidence submitted by the prosecution.
144. ANGELINA SORIENTE v. THE ESTATE OF THE LATE ARSENIO E. CONCEPCION (2009) [Rule
5; Rule 8; Rule on Summary Procedure]
In unlawful detainer case, there must be alleged in the complaint material information such as the date of
demand because defendants possession becomes unlawful only one year after the date of demand.

REM Digests ALS 2014B Justice Gesmundo


To make out a case of unlawful detainer under Section 1, Rule 70 of the Rules of Court, the Complaint must
allege that the defendant is unlawfully withholding from the plaintiff the possession of certain real property
after the expiration or termination of the formers right to hold possession by virtue of a contract, express or
implied, and that the action is being brought within one year from the time the defendants possession
became unlawful. The one-year period within which a complaint for unlawful detainer can be filed should be
counted from the date of demand, because only upon the lapse of that period does the possession become
unlawful.
Courts in ejectment cases decide questions of ownership only it is necessary to decide the question of
possession. The reason for this rule is to prevent the defendant from trifling with the summary nature of an
ejectment suit by the simple expedient of asserting ownership over the disputed property. The courts
adjudication of ownership in an ejectment case is merely provisional, and affirmance of the trial courts
decision would not bar or prejudice an action between the same parties involving title to the property, if and
when such action is brought seasonably before the proper forum.
Lastly, this unlawful detainer case is filed under the Rules on Summary Procedure. A provision therein says that
if the defendant fails to appear in the preliminary conference, then the plaintiff will be entitled to judgment.
This provision will not apply when there there are multiple defendants (sued under a common cause of
action and pleaded a common defense) and only one (not all) of the defendants fail to appear in the
preliminary conference.

RULE 9

33. Magdiwang Realty v. Manila Banking Corporation (2012) (Rule 9, Default)


The petitioners default by their failure to answer [filed a Motion for Leave to Admit Motion to Dismiss and a
Motion to Dismiss beyond the 15-day period] led to certain consequences. Where defendants before a trial
court are declared in default, they thereby lose their right to object to the reception of the plaintiffs evidence
establishing his cause of action. This is akin to a failure to, despite due notice, attend in court hearings for
the presentation of the complainants evidence, which absence would amount to the waiver of such
defendants right to object to the evidence presented during such hearing, and to cross-examine the witnesses
presented therein.
45. ROBERTO OTERO v. ROGER TAN (2012)
There are 4 remedies for a party who has been declared in default.
Otero is the debtor. Tan is the creditor. This is an action for collection of a sum of money with damages filed by
Tan against Otero. Summons was served to Otero but he failed to promptly file an answer. The trial court declared
Otero in default. It ruled in favor of Tan, the creditor. Otero appeals. The appellate courts rebuff him because they
say that when a defendant is declared in default, he waives any defense that could be raised. Otero disagrees with
this.
Held: Otero could appeal. Indeed, a defendant who fails to file an answer loses his standing in court. Loss of
standing in court, the forfeiture of ones right as a party litigant, contestant or legal adversary, is the consequence of
an order of default. A party in default loses his right to present his defense, control the proceedings, and examine or
cross-examine witnesses. He has no right to expect that his pleadings would be acted upon. However, he has four
remedies.

Name of Remedy When Filed Ground or Pertinent


Provision

1. Motion to set aside the At any time after discovery of declaration of default FAME (Sec 3 Rule 18)
order of default and before judgment

REM Digests ALS 2014B Justice Gesmundo


2. Motion for New Trial If judgment has already been rendered when the Section 1a Rule 37
defendant discovered the default, but before the
declaration of default has become final and executory

3. Petition for Relief If the defendant discovered the default after the Section 2 Rule 38
judgment has become final and executory

4. IMPORTANT! APPEAL IMPORTANT! Only LIMITED GROUNDS FOR Section 2 Rule 41


from judgment rendered APPEAL!
against him, as contrary to
1. failure of the plaintiff to prove the MATERIAL
the evidence or to the law
ALLEGATIONS of the complaint
2. DECISION is contrary to law
3. the AMOUNT of judgment is excessive or
different in kind from that prayed for

RULE 10

143. AGRIFINA PANGANIBAN v. SPOUSES ROLDAN (2009) [Rule 10 Amended and Supplemental
Pleadings]
Where there is a variance in defendants pleadings and the evidence adduced at the trial, the court may
treat the pleading as if it had been amended to conform to the evidence.
This case is one of recovery of possession and damages. During trial, presented as evidence were a TCT and a
Kasunduan. These were never raised in the pleadings. The court resolved the case based on these two documents.
It is settled that issues NOT raised by the pleadings may be tried by express or implied consent of the parties,
as if they had been raised in the pleadings and the court can validly resolve them.
The rule on amendment of pleadings to conform to evidence presented during trial need not be applied
rigidly, particularly where no surprise or prejudice is caused the objecting party. Where there is a variance in
defendants pleadings and the evidence adduced at the trial, the court may treat the pleading as if it had been
amended to conform to the evidence.

RULE 11

38) and 39) DEL MONTE v. DOW CHEMICAL CO. (2012) and DOW CHEMICAL v. GRAGEDA (2012)
(consolidated) - omitted counterclaim or cross claim, two requisites allow
Farm workers sue several companies for an alleged exposure to the chemical DBCP which had negative impacts to
their health. There are four sets of defendants; Chiquita, DOW, Del Monte and DOLE. Chiquita and Del Monte
filed a Motion to Admit Amended Answer with Counter-Claim and Cross-Claim, respectively. Dole defendants
filed a Motion to Admit Amended Answer with the Amended Answer with Cross-Claim Ad Cautelam. They
alleged that since they were in imminent danger of being the only defendants left, they were constrained to file a
cross-claim against their co-defendants in order to secure their right to contribution and reimbursement.
Based on Rule 11, Sec. 10: there are two requisites for a court to allow an omitted counterclaim or cross-
claim by amendment: (1) there was oversight, inadvertence, or excusable neglect, or when justice requires;
and (2) the amendment is made before judgment.

REM Digests ALS 2014B Justice Gesmundo


The CA correctly held that there is basis for allowing the cross-claims of the Dole, Del Monte and Chiquita
defendants against the Dow/Occidental defendants as they complied with the rules. It is undisputed that the
Dole, Del Monte and Chiquita defendants sought to amend their answers to include their cross-claims before
judgment. More importantly, justice requires that they be allowed to do so in consonance with the policy
against multiplicity of suits.

RULE 12

RULE 13

49. BAUTISTA V. CRUZ, SHERIFF OF RTC BRANCH 53, ROSALES PANGASINAN, A.M.NO. P-12-
3062 (2012)
For Rules 13 and 39 Notice to the client and not to the counsel of record is not notice within the meaning of
the law.
Bautista, Posadas and Ramos filed a Complaint for Ejectment with Prayer for Writ of Demolition and Damages
against Vallejos and Basconsillo concerning a parcel of land situated in Pangasinan. The MTC ruled in favor of
plaintiffs, while the RTC, CA and SC later affirmed, making the decision final and executory. The corresponding
writ of execution was issued by the MTC ordering sheriff Cruz to implement the final decision. On the day of the
implementation however, despite Bautistas insistence to use all reasonable force to implement the writ of
execution, the sheriff refused to do so because of the absence of a special order of demolition, as there were
improvements made on the subject lot. Bautista later found out that the Notice to Vacate was served only on the
opposing parties and not their counsel, preventing the sheriff from executing the writ. Alleging gross ignorance of
the law, inefficiency, misfeasance of duty, bias and partiality, the complainant filed this administrative case.
While the sheriff was correct in initially refusing to implement the writ of execution absent a special order of
demolition, the sheriff indeed committed an error when he served the Notice to Vacate on the defendants,
and not their counsel. Section 10c of Rule 39 requiring service of Notice to Vacate upon a judgment obligor
must be read in conjunction with Section 2, Rule 13 of the Rules of Court which requires that service of
pleadings or papers must be made on the counsel if a party is already represented by one. It is a settled rule
that notice to the client will only be binding and effective if specifically ordered by the Court. Sheriff Cruz is
guilty of inefficiency and incompetence and is hereby reprimanded.
60. Mojar, et al. v. Agro Commercial Security Service Agency, Inc. et al. (2012)
Rule 13, Service If a party to a case has appeared by counsel, service of pleadings and judgments shall be
made upon his counsel or one of them, unless service upon the party is specifically ordered by the court. It is
not the duty of the courts to inquire, during the progress of a case, whether the law firm or partnership
representing one of the litigants continues to exist lawfully, whether the partners are still alive, or whether its
associates are still connected with the firm.
Rule 46 in petition for certiorari; actual address of respondent and petitioners mandatory, exceptions
where the petitioner clearly mentioned that the parties may be served with the courts notices or processes
through their respective counsels, whose addresses have been clearly specified as in this case, this act would
constitute substantial compliance with the requirements of Section 3, Rule 46
Rule 138 valid substitution of counsel Under Section 26, Rule 138 of the Rules of Court and established
jurisprudence, a valid substitution of counsel has the following requirements: (1) the filing of a written
application for substitution; (2) the client's written consent; (3) the consent of the substituted lawyer if such
consent can be obtained; and, in case such written consent cannot be procured, (4) a proof of service of
notice of such motion on the attorney to be substituted in the manner required by the Rules. Where death of
the previous attorney is the cause of substitution of the counsel, a verified proof of the death of such attorney
(usually a death certificate) must accompany the notice of appearance of the new counsel.

REM Digests ALS 2014B Justice Gesmundo


Petitioners, security guards, filed a complaint for illegal dismissal against Agro Security Agency before the NLRC.
The Labor Arbiter held that the petitioners were illegally dismissed. This decision was affirmed by the NLRC on
appeal. Thus, Agro filed a petition for certiorari before the CA. The CA rendered that there was no illegal
dismissal. Petitioners thereafter filed a Manifestation, stating that they had not been served a copy of the CA
Petition and they did not know whether their counsel before the NLRC, Atty. Espinas, was served a copy since he
was already bedridden until his demise. They also alleged that their new counsel, Atty, Aglipay, could not enter his
appearance before the CA since they failed to get the folder from the office of their former counsel. The petitioners
moved to annul the proceedings since Agro purposely intended to exclude petitioners from the proceedings before
the CA by omitting their actual addresses in the CA Petition, a mandatory requirement under Section 3, Rule 46; in
relation to Section 1, Rule 65 of the Rules of Court. Further, the petitioners allege that Agro failed to prove the
valid service of its CA Petition upon their former counsel of record.
Actual Addresses of Parties: In the 2008 case of Cendaa v. Avila, the Court ruled that the requirement that a
petition for certiorari must contain the actual addresses of all the petitioners and the respondents is
mandatory. The failure to comply with that requirement is a sufficient ground for the dismissal of a petition.
This rule, however, is not absolute. In the 2011 case Santos v. Litton Mills Incorporated, the Court ruled that
where the petitioner clearly mentioned that the parties may be served with the courts notices or processes
through their respective counsels, whose addresses have been clearly specified as in this case, this act would
constitute substantial compliance with the requirements of Section 3, Rule 46. The notice required by law is
notice to counsel if the party has already appeared by counsel, pursuant to Section 2, Rule 13 of the Rules of Court.
In the present case, it was clearly indicated in the Petition before the CA that the petitioners are represented by their
counsel, Atty. Espinas, and that they may be served with process through this counsel at his stated address. Thus, it
is considered as substantial compliance with Section 3, Rule 46.
Affidavit of Service: In Ferrer v. Villanueva, the Court held that failure to append the proof of service to the
petition for certiorari was a fatal defect. In the present case, the CA records revealed that Atty. Espinas, the
counsel of the petitioners, was duly served a copy of the petition for certiorari filed by Agro. Such service to
Atty. Espinas, as petitioners counsel of record, was valid despite the fact he was already deceased at the
time.
Substitution of Counsel: The Court held that having known that Atty. Espinas was already bedridden, petitioners
should have already obtained new counsel who could adequately represent their interests. The fact that petitioners
were unable to obtain their folder from Atty. Espinas is immaterial. Proof of service upon the lawyer to be
substituted will suffice where the lawyers consent cannot be obtained. With respect to the records of the case, these
may easily be reconstituted by obtaining copies thereof from the various courts involved.
123.) MARCELINO DOMINGO VS. COURT OF APPEALS, G.R. NO. 169122 (2010)
Rule 13, Sec.11: Priorities in modes of service and filing-Whenever practicable, the service and filing of
pleadings and other papers shall be done personally.
Julio executed a Deed of Absolute Sale over a parcel of land in favor of Marcelinos wife, Carmelita. One branch of
the Domingos filed a case of annulment of the deed of absolute sale alleging that Julios signature was forged. RTC
ruled that Julios signature was forged and ordered Marcelino and his wife to deliver possession of the property to
the Domingos. Appeals and MRs were filed in several venues. Last of which was the petition for certiorari under
Rule 65 filed in the CA by Marcelino. CA dismissed outright the petition. One reason for the dismissal was because
there was no written explanation to justify service by mail in lieu of the preferred mode of personal service in
violation of the ROC which makes the petition deemed not filed. Marcelino claims that the failure to incorporate a
written explanation to justify service by mail in lieu of preferred mode of personal service was not vital because
Sec. 11, Rule 13 is merely directory and it is incumbent upon the court to use discretion in determining whether
substantial justice will be served (or rights unjustifiably prejudiced). Court dismissed Marcelinos claim.
Personal Service and filing are preferred for obvious reasons. Plainly such should expedite action or
resolution on a pleading, motion or other paper; and conversely, minimize, if not eliminate, delays likely to
be incurred if service or filing is done by mail, considering the inefficiency of postal service. Sec. 11 gives the
court the discretion to consider a pleading or paper as not filed if the other modes were resorted to and no
written explanation was made as to why personal service was not done in the first place. Personal Service
and Filing is the general rule (or mandatory when practicable) and resort to other modes the exception.

REM Digests ALS 2014B Justice Gesmundo


151. HEIRS OF JOSE SY BANG, ET. AL., v. ROLANDO SY, ET. AL. (2009) AND 152. ILUMINADA TAN,
ET. AL., v. BARTOLOME SY, ET. AL. (2009)
NOTE: consolidated case yung 151 and 152 tapos nakalagay siya sa topic na Rule 14, lis pendens pero
walang anything on summons. Im assuming Rule 13, Sec 14 yung ibig sabihin ni sir (LIS PENDENS)-
"While the trial court has an inherent power to cancel a notice of lis pendens, such power is to be exercised
within the express confines of the law.
Rolando Sy filed a Complaint for Partition against his siblings from both 1st and 2nd marriages of Sy Bang before
CFI of Quezon. Sy Bang died intestate. On March 30, 1981, the Judge rendered a First Partial Decision based on
the Compromise Agreement dated November 10, 1980, on April 2, 1981, the Judge rendered a Second Partial
Decision based on the pretrial order of the court, dated March 25, 1981, and both partial decisions had long become
final, without an appeal having been interposed by any of the parties. On June 8, 1982, the Judge rendered a Third
Partial Decision. On August 17, 1982, the Judge issued two Orders: (1) in the first Order, Mrs. Lucita L. Sarmiento
was appointed as Receiver, and petitioners Motion for New Trial and/or Reconsideration, dated July 9, 1982 and
their Supplemental Motion, dated July 12, 1982, were denied for lack of merit; and (2) in the second Order, the
Judge ordered the immediate cancellation of the lis pendens annotated at the back of the certificates of title in the
names of Bartolome Sy, Rosalino Sy and Rolando Sy.
The Court held that YES, the order to cancel the notice of lis pendens is proper. The filing of a notice of lis
pendens has a two-fold effect: (1) to keep the subject matter of the litigation within the power of the court
until the entry of the final judgment in order to prevent the final judgment from being defeated by successive
alienations; and (2) to bind a purchaser, bona fide or not, of the land subject of the litigation to the judgment
or decree that the court will promulgate subsequently. While the trial court has an inherent power to cancel
a notice of lis pendens, such power is to be exercised within the express confines of the law. As provided in
Section 14, Rule 13 of the 1997 Rules of Civil Procedure, a notice of lis pendens may be cancelled on two
grounds: (1) when the annotation was for the purpose of molesting the title of the adverse party, or (2) when
the annotation is not necessary to protect the title of the party who caused it to be recorded. The notice is but
an incident in an action, an extrajudicial one, to be sure. It does not affect the merits thereof. It is intended
merely to constructively advise, or warn, all people who deal with the property that they so deal with it at
their own risk, and whatever rights they may acquire in the property in any voluntary transaction are
subject to the results of the action, and may well be inferior and subordinate to those which may be finally
determined and laid down therein. The cancellation of such a precautionary notice is therefore also a mere
incident in the action, and may be ordered by the Court having jurisdiction of it at any given time. The
ruling of the RTC and CA were upheld since Rosalino, Bartolome and Rolando were able to prove that the
notice was intended merely to molest and harass the owners of the property, some of whom were not parties
to the case and it was also proven that the interest of Oscar Sy, who caused the notice to be annotated, was
only 1/14 of the assessed value of the property. Moreover, Rosalino, Bartolome and Rolando were ordered to
post a P50,000.00 bond to protect whatever rights or interest Oscar Sy may have in the properties under litis
pendentia.

RULE 14

32. Planters Development Bank v. Chandumal


Court detailed the requisites for a valid substituted service of summons, summed up as follows:
(1) impossibility of prompt personal service the party relying on substituted service or the sheriff must show that
the defendant cannot be served promptly or there is impossibility of prompt service;
(2) specific details in the return the sheriff must describe in the Return of Summons the facts and circumstances
surrounding the attempted personal service;
(3) a person of suitable age and discretion the sheriff must determine if the person found in the alleged dwelling
or residence of defendant is of legal age, what the recipients relationship with the defendant is, and whether said
person comprehends the significance of the receipt of the summons and his duty to immediately deliver it to the
REM Digests ALS 2014B Justice Gesmundo
defendant or at least notify the defendant of said receipt of summons, which matters must be clearly and
specifically described in the Return of Summons; and
(4) a competent person in charge, who must have sufficient knowledge to understand the obligation of the
defendant in the summons, its importance, and the prejudicial effects arising from inaction on the summons.
In this case, there was no valid substituted service of summons. the sheriffs return failed to justify a resort to
substituted service of summons. It does not specifically show or indicate in detail the actual exertion of efforts or
any positive step taken by the officer or process server in attempting to serve the summons personally to the
defendant. The return merely states the alleged whereabouts of the defendant without indicating that such
information was verified from a person who had knowledge thereof.
Indeed, the sheriffs return shows a mere perfunctory attempt to cause personal service of the summons on
Chandumal. There was no indication if he even asked Chandumals mother (the one whom substituted service of
summons was made to) as to her specific whereabouts except that she was "out of the house", where she can be
reached or whether he even tried to await her return. The "efforts" exerted by the sheriff clearly do not suffice to
justify substituted service and his failure to comply with the requisites renders such service ineffective
98. EDNA LHUILLIER VS. BRITISH AIRWAYS (2010)
The special appearance of the counsel in filing the Motion to Dismiss and other pleadings before the trial
court cannot be deemed to be voluntary submission to the jurisdiction of the said trial court.
Edna Lhuillier filed a Complaint for damages against British Airways based on 3 instances during her flight. First,
was with a flight attendant who refused to help her and sarcastically said "If I were to help all 300 passengers in this
flight, I would have a broken back!". Second, another flight attendant made her appear to the other passengers to be
ignorant, uneducated, stupid, and in need of lecturing on the safety rules and regulations of the plane. Third, was
when she demanded an apology from the ground manager, which only said the flight stewards were only doing
their job.
British Airways, by way of special appearance through counsel, filed a Motion to Dismiss on grounds of lack of
jurisdiction over the case and over the person of the respondent. Respondent alleged that only the courts of London,
United Kingdom or Rome, Italy, have jurisdiction over the complaint for damages pursuant to the Warsaw
Convention. RTC granted the MTD. This is now a Petition for review on Certiorari.
Main Issue: Whether the Warsaw Convention applies YES, complaint dismissed for lack of jurisdiction.
(jurisdiction is with court of domicile or place of business of carrier).
REM Issue: Whether British Airways voluntarily submitted itself to the jurisdiction of the court NO. Sec. 20,
Rule 14 of the Revised Rules of Civil Procedure clearly provides: Sec. 20. Voluntary appearance. The
defendants voluntary appearance in the action shall be equivalent to service of summons. The inclusion in a
motion to dismiss of other grounds aside from lack of jurisdiction over the person of the defendant shall not be
deemed a voluntary appearance.
Thus, a defendant who files a motion to dismiss, assailing the jurisdiction of the court over his person,
together with other grounds raised therein, is not deemed to have appeared voluntarily before the court.
What the rule on voluntary appearance the first sentence of the above-quoted rule means is that the
voluntary appearance of the defendant in court is without qualification, in which case he is deemed to have
waived his defense of lack of jurisdiction over his person due to improper service of summons.
101. LEAH PALMA, PETITIONER, VS. HON. DANILO P. GALVEZ, IN HIS CAPACITY AS
PRESIDING JUDGE OF THE REGIONAL TRIAL COURT OF ILOILO CITY, BRANCH 24; AND
PSYCHE ELENA AGUDO, RESPONDENTS (2010)
if a resident defendant is temporarily out of the country, any of the following modes of service may be
resorted to: (1) substituted service set forth in section 7, Rule 14; (2) personal service outside the country,
with leave of court; (3) service by publication, also with leave of court; or (4) in any other manner the court
may deem sufficient
Petitioner Leah Palma filed with the RTC a case. The RTC's process server submitted his return of summons stating
that the alias summons, together with a copy of the amended complaint and its annexes, were served upon private

REM Digests ALS 2014B Justice Gesmundo


respondent thru her husband Alfredo Agudo, as private respondent was out of the country. The counsel of private
respondent filed a Notice of Appearance and a Motion for Extension of Time to File Answer stating that he was just
engaged by private respondent's husband as she was out of the country and the Answer was already due. Again,
private respondent's counsel filed a Motion for Another Extension of Time to File Answer, and stating that while
the draft answer was already finished, the same would be sent to private respondent for her clarification/verification
before the Philippine Consulate in Ireland. Subsequently, respondent filed a Motion to Dismiss on the ground that
the RTC had not acquired jurisdiction over her as she was not properly served with summons, since she was
temporarily out of the country.
Section 16 of Rule 14 uses the words "may" and "also," it is not mandatory. Other methods of service of
summons allowed. In this case the Sheriff's Return stated that private respondent was out of the country;
thus, the service of summons was made at her residence with her husband, Alfredo, acknowledging receipt
thereof. Alfredo was presumably of suitable age and discretion, who was residing in that place and,
therefore, was competent to receive the summons on private respondent's behalf. In addition, the RTC had
indeed acquired jurisdiction over the person of private respondent when the latter's counsel entered his
appearance on private respondent's behalf, without qualification and without questioning the propriety of
the service of summons, and even filed two Motions for Extension of Time to File Answer.
137. Constantino Pascual, et. al. v. Lourdes S. Pascual (Rule 14)
For a substituted service of summons to be valid, the details of the attendant circumstances or of the efforts
exerted to serve the summons, must be indicated in the return. A general statement that such efforts were
made will not suffice for purposes of complying with the rules of substituted service of summons.
Constantino Pascual, et. al. (Constantino, et. al.) filed a complaint for specific performance with a prayer for
issuance of preliminary mandatory injunction against Lourdes Pascual. The process server attempted to serve
summons four times, but failed. In his last report, he stated that a substituted service was effected. Lourdes Pascual
was declared in default. The RTC ruled in favor of Constantino, et. al. Lourdes Pascual filed a motion to set aside
order of default because the substituted service was improper and invalid, hence, court did not acquire jurisdiction
over her person. W/N the substituted service of summons was valid? NO. Nowhere in the returns of summons was
it showed that effort was actually exerted and positive steps were taken by either the process server or petitioners to
locate and serve the summons personally on respondents. At best, the return merely states the alleged whereabouts
of respondents without indicating that such information was verified from a person who had knowledge
thereof. Certainly, without specifying the details of the attendant circumstances or of the efforts exerted to
serve the summons, a general statement that such efforts were made will not suffice for purposes of
complying with the rules of substituted service of summons. Jurisdiction over the person of a defendant
cannot be acquired.
174. Ma. Imelda Manotoc v. Hon, Court of Appeals et al. (2006) Jurisdiction- rule 14 valid substituted
service of summons and Rule 16 Sheriffrs return
Without a valid service, the court cannot acquire jurisdiction over the defendant, unless the defendant
voluntarily submits to it. The defendant must be properly apprised of a pending action against him and
assured of the opportunity to present his defenses to the suit. Proper service of summons is used to protect
ones right to due process. The Sheriffs Return must specify details as to the efforts and attempts made to
justify substituted service of summons.
This case involves the enforcement of a foreign judgment by the US District Court of Hawaii for wrongful death of
a certain Trajano by Philippine military intelligence officials allegedly under the influence of the petitioner. The
trial court issued a Summons addressed to petitioner at Alexandra Condominium Corporation or Alexandra Homes,
E2 Room 104, at No. 29 Meralco Avenue, Pasig City. The Summons and a copy of the Complaint were allegedly
served upon (Mr.) Macky de la Cruz, an alleged caretaker of petitioner at the condominium unit mentioned earlier.
When petitioner failed to file her Answer, the trial court declared her in default. Petitioner, by special appearance
of counsel, filed a MTD on the ground of lack of jurisdiction of the trial court over her person due to an invalid
substituted service of summons. The grounds to support the motion were: (1) the address of defendant indicated in
the Complaint (Alexandra Homes) was not her dwelling, residence, or regular place of business as provided in
Section 8, Rule 14 of the Rules of Court and (2) the party (de la Cruz), who was found in the unit, was neither a
representative, employee, nor a resident of the place. RTC denied the MTD and relied on the presumption that the
REM Digests ALS 2014B Justice Gesmundo
sheriffs substituted service was made in the regular performance of official duty, and such presumption stood in
the absence of proof to the contrary. Upon appeal, the CA likewise held that there was valid substitute service of
summons. The SC reversed.
There was invalid substituted service of summons. The Sheriffs Return lacked material data on the serious
efforts to serve summons on the petitioner. Aside from generally couched phrases of on many occasions several
attempts were made to serve the summons x x x personally, at reasonable hours during the day, and to no avail
for the reason that the said defendant is usually out of her place and/or residence or premises,it cannot be
determined how many times, on what specific dates, and at what hours of the day the attempts were made. Given
the fact that the substituted service of summons may be assailed, as in the present case, by a Motion to
Dismiss, it is imperative that the pertinent facts and circumstances surrounding the service of summons be
described with more particularity in the Return or Certificate of Service. Before resorting to substituted
service, a plaintiff must demonstrate an effort in good faith to locate the defendant through more direct
means. The complaint only states that respondents were informed, and so [they] allege about the address and
whereabouts of petitioner. For substituted service of summons to be available, there must be several attempts
by the sheriff to personally serve the summons within a reasonable period [of one month] which eventually
resulted in failure to prove impossibility of prompt service. Several attempts means at least three (3) tries,
preferably on at least two different dates. In addition, the sheriff must cite why such efforts were unsuccessful.
The facts and circumstances should be stated with more particularity and detail: the number of attempts made at
personal service, dates and times of the attempts, inquiries to locate defendant, names of occupants of the alleged
residence, and the reasons for failure should be included in the Return to satisfactorily show the efforts undertaken.
Person of suitable age and discretion residing in defendants house or residence. There are 2 reqts under the
Rules: (1) recipient must be a person of suitable age and discretion; and (2) recipient must reside in the house
or residence of defendant. Both requirements were not met. In this case, the Sheriffs Return lacks information
as to residence, age, and discretion of Mr. Macky de la Cruz, aside from the sheriffs general assertion that de la
Cruz is the resident caretaker of petitioner as pointed out by a certain Ms. Lyn Jacinto, alleged receptionist and
telephone operator of Alexandra Homes. It is doubtful if Mr. de la Cruz is residing with petitioner Manotoc in the
condominium unit considering that a married woman of her stature in society would unlikely hire a male caretaker
to reside in her dwelling. With the petitioners allegation that Macky de la Cruz is not her employee, servant, or
representative, it is necessary to have additional information in the Return of Summons. Besides, Mr. Macky de la
Cruzs refusal to sign the Receipt for the summons is a strong indication that he did not have the necessary relation
of confidence with petitioner. To protect petitioners right to due process by being accorded proper notice of a
case against her, the substituted service of summons must be shown to clearly comply with the rules.
Likewise, the presumption of regularity in the performance of official functions by the sheriff is not
applicable in this case where it is patent that the sheriffs return is defective for lacking details of the efforts
and attempts to effect personal service.

RULE 15

131. Metropolitan Bank & Trust Company v. Hon. Salvador Abad Santos and Manfred Jacob De Koning
(2009) Rule 7; Rule 15
A certificate of non-forum shopping is required to be attached only to complaints and other initiatory
pleadings. A writ of execution is not a petition but a motion. The purpose of a motion is not to initiate new
litigation, but to bring a material but incidental matter arising in the progress of the case in which the
motion is filed. A motion is not an independent right or remedy, but is confined to incidental matters in the
progress of a cause. It relates to some question that is collateral to the main object of the action and is
connected with and dependent upon the principal remedy. An application for a writ of possession is a mere
incident in the registration proceeding.; A judicial proceeding, order, injunction, etc., is ex parte when it is
taken or granted at the instance and for the benefit of one party only, and without notice to, or contestation
by, any person adversely interested.

REM Digests ALS 2014B Justice Gesmundo


De Koning obtained a loan from Metrobank secured by a real estate mortgage over a condominium unit. When De
Koning failed to pay his loan despite demand, Metrobank instituted extrajudicial foreclosure proceedings where
Metrobank was the highest bidder. The redemtion period lapsed without De Koning redeeming th property.
Metrobank demanded that De Koning turn over the possession. When De Koning refused, Metrobank filed an ex
parte petition for a writ of possession. The lower court issued an order setting the ex parte hearing and directing that
a copy of such order be given to De Koning. During the ex parte hearing, De Konings counsel appeared and
manifested that he filed a motion to dismiss on the ground that Metrobanks ex parte petition violated Sec. 5, Rule 7
which required the attachment of a certification against forum shopping to a complaint or other initiatory pleading.
RTC agreed with De Koning and dismissed the petition for having a false certification of non-forum shopping (De
Koning said that there was another case pending involving the same parties, issues and subject matter, and a
pending petition for certiorari, both of which Metrobank did not disclose). CA affirmed the dismissal and denied an
MR.
A writ of possession is defined as a writ of execution employed to enforce a judgment to recover the
possession of land. It commands the sheriff to enter the land and give its possession to the person entitled
under the judgment. In order to obtain a writ of possession, the purchaser in a foreclosure sale must file a
petition, in the form of an ex parte motion, in the registration or cadastral proceedings of the registered
property. The reason why this pleading, although denominated as a petition, is actually considered a motion
is best explained in Sps. Arquiza v. CA, where the Court said: The certification against forum shopping is
required only in a complaint or other initiatory pleading. The ex parte petition for the issuance of a writ of
possession filed by the respondent is not an initiatory pleading. Although denominated as a petition, it is,
nonetheless, a motion. What distinguishes a motion from a petition or other pleading is not its form or the
title given by the party executing it, but rather its purpose. The office of a motion is not to initiate new
litigation, but to bring a material but incidental matter arising in the progress of the case in which the
motion is filed. A motion is not an independent right or remedy, but is confined to incidental matters in the
progress of a cause. It relates to some question that is collateral to the main object of the action and is
connected with and dependent upon the principal remedy. An application for a writ of possession is a mere
incident in the registration proceeding. Hence, although it was denominated as a petition, it was in
substance merely a motion.
Also, the lower court should not have allowed De Koning to intervene in the proceedings. A judicial
proceeding, order, injunction, etc., is ex parte when it is taken or granted at the instance and for the benefit
of one party only, and without notice to, or contestation by, any person adversely interested. The lower court
erred in notifying De Koning of Metrobanks ex parte petition. when it allowed De Koning to participate in
the proceedings and when it took cognizance and upheld the motion to dismiss.
147. FAJARDO ET. AL., v. COMANDANTE (2009) (RULE 15)
A final and executory order cannot be collaterally attacked through a motion, the sole object of which, is in
truth, the nullification of said order.
Fajardo et. Al, filed a case for annulment of sale against Comandante. Comandante filed a motion to dismiss the
case because the Fajardos failed to prosecute the case for an unreasonable length of time. The RTC ruled in his
favor and dismissed the case without prejudice. On April 24, 2006, Comandante moved that the case be dismissed
with prejudice. The RTC granted because it seemed that neither the Fajardos nor their counsel were interested in
pursuing the case. The case was dismissed with prejudice in an order dated May 11, 2006. On Sept. 11, 2006, the
Fajardos filed a motion for the RTC to treat the April 24 motion to dismiss as a mere scrap of paper because it did
not conform with the three-day rule, alleging that they only received it a day after the hearing of said motion. The
RTC denied the motion of the Fajardos finding that the latter already failed to file an appeal of the order which has
already become final and executory. The Fajardos filed a petition for certiorari alleging that the RTC gravely
abused its discretion when it did not treat the April 24 petition as a mere scrap of paper.
The motion of the Fajardos was in the nature of a motion for reconsideration of the May 11, 2006 Order,
crudely masked to hide the fact that they filed it out of time. Since the Fajardos did not appeal from the May
11, 2006 order of the RTC, the same became final and executory as a matter of course. It can no longer be
modified in any respect, even if the modification is meant to correct erroneous conclusions of fact and law,
and whether it is made by the court that rendered it or by an appellate court. As a final and valid order, it

REM Digests ALS 2014B Justice Gesmundo


could not be collaterally attacked through the Fajardos artful motion to treat Albertos April 24, 2006
motion as a scrap of paper, where the sole object, in truth, is the nullification of the May 11, 2006 Order.

RULE 16

4. Bagayas v. Bagayas (September 18, 2013) (Res judicata (Rule 16); partition (Rule 69))
Petitioner filed a complaint for annulment of sale and partition, claiming that the respondents intended to exclude
her from inheriting from the estate of her legally adoptive parents. The RTC dismissed the case. Thereafter, she
filed another case for the amendment of the TCTs to include her name and those of her heirs, under PD 1529. The
issue to be resolved in this case is W/N the dismissal of the earlier complaint on the ground that it is in the nature of
a collateral attack on the certificates of title constitutes a bar to a subsequent petition under PD 1529. In a
complaint for partition, the plaintiff seeks, first, a declaration that he is a co-owner of the subject properties;
and second, the conveyance of his lawful shares. An action for partition is at once an action for declaration of
co-ownership and for segregation and conveyance of a determinate portion of the properties involved. The
determination, therefore, as to the existence of co-ownership is necessary in the resolution of an action for
partition.
A resolution on the issue of ownership does not subject the Torrens title issued over the disputed realites to a
collateral attack. It must be borne in mind that what cannot be collaterally attacked is the certificate of title
and not the title itself.
Thus, the RTC erroneously dismissed petitioners petition for annulment of sale on the ground that it constituted a
collateral attack since she was actually assailing the respondents title to the subject lands and not any Torrens
certificate of title over the same. YET, the appeal was filed out of time!
Consequently, there is no res judicata in this case, because they involve different causes of action. As to the first
case, no partition was decreed as the action was dismissed. Second, petitioner cannot avail of the summary
proceedings under Section 108 of PD 1529 because the present controversy does not involve the amendment of the
certificates of title issued in favor of the respondents but the partition of the estate of the deceased.
22. PILAR DEVELOPMENT v. CA (2013)
"In order that there may be res judicata, it is requisite (a) that the former judgment is final; (b) that it has
been rendered by a court of competent jurisdiction; (c) that it is a judgment on the merits; and (d) that,
between the first and the second actions, there is identity of parties, subject-matter, and cause of action."
Spouses Lopez and Ng acquired some properties in Las Pinas from Dumbrique. Fusilero filed and adverse
claim but the court eventually held that the land belongs to the spouses. The Factors, who allegedly acquired
some property part of the subject lands in the previous case, sold them to herein petitioners. Initially the CFI
ruled in favor of the factors, however upon reopening of the case, the court ruled in favor of herein private
respondents. Neither of the parties appealed. Instead, they filed for annulment of the titles of the properties
which eventually was also dismissed by the SC. They filed for quieting of titles as a third case, which was also
eventually dismissed by the lower courts. They now come with a petition for review
The facts of this case clearly show that petitioners cause of action is already barred by the prior judgments
of the RTC in its Decision dated 8 December 1994 in Case 1 and of this Court in Case 2. The fundamental
principle behind the doctrine of res judicata is that parties ought not to be permitted to litigate the same
issue more than once. That is, when a right or a fact has been judicially tried and determined by a court of
competent jurisdiction, or an opportunity for such trial has been given, the judgment of the courtso long
as it remains unreversed should be conclusive upon the parties and those in privity with them in law or
estate. The Decisions of the RTC in Case 1 and of this Court in Case 2 both of which ruled that
respondents are the rightful owners of the property in questionhave all become final and unappealable. In
Case 2, this Court had jurisdiction over the subject matter and over the parties; the judgments were issued
on the merits; and there was a similarity of parties, subject matter, and cause of action. We rule that there is

REM Digests ALS 2014B Justice Gesmundo


identity of causes of action, the test for which is to look into the facts or evidence necessary to maintain the
two actions, to wit:
Hornbook is the rule that identity of causes of action does not mean absolute identity. Otherwise, a party
could easily escape the operation of res judicata by changing the form of the action or the relief sought. The
test to determine whether the causes of action are identical is to ascertain whether the same evidence will
sustain both actions, or whether there is an identity in the facts essential to the maintenance of the two
actions. If the same facts or evidence would sustain both, the two actions are considered the same, and a
judgment in the first case is a bar to the subsequent action.
24. DEUSTCHE BANK AG v. CIR (2013)
"A minute resolution, with respect to the same subject matter and the same issues concerning the same
parties, it constitutes res judicata."
The petitioner withheld and remitted to respondent on 21 October 2003 the amount of PHP 67,688,553.51,
which represented the fifteen percent (15%) branch profit remittance tax (BPRT) on its regular banking
unit (RBU) net income remitted to Deutsche Bank Germany (DB Germany) for 2002 and prior taxable years.
Believing that it made an overpayment of the BPRT, petitioner filed with the BIR Large Taxpayers
Assessment and Investigation Division on 4 October 2005 an administrative claim for refund or issuance of
its tax credit certificate in the total amount of PHP 22,562,851.17. On the same date, petitioner requested
from the International Tax Affairs Division (ITAD) a confirmation of its entitlement to the preferential tax
rate of 10% under the RP-Germany Tax Treaty. The CTA denied the petition on the ground that the
application for a tax treaty relief was not filed with ITAD prior to the payment by the former of its BPRT
and actual remittance of its branch profits to DB Germany, or prior to its availment of the preferential rate
of ten percent (10%) under the RP-Germany Tax Treaty provision. The CTA relied on the minute resolution
of Mirant v. CIR where the SC held that before the benefits of the tax treaty may be extended to a foreign
corporation wishing to avail itself thereof, the latter should first invoke the provisions of the tax treaty and
prove that they indeed apply to the corporation.
A minute resolution is not a binding precedent. With respect to the same subject matter and the same issues
concerning the same parties, it constitutes res judicata. However, if other parties or another subject matter
(even with the same parties and issues) is involved, the minute resolution is not binding precedent.
30. NHA vs Baello
The doctrine of res judicata has been explained as follows:
The rule is that when material facts or questions, which were in issue in a former action and were admitted or
judicially determined are conclusively settled by a judgment rendered therein, such facts or questions become res
judicata and may not again be litigated in a subsequent action between the same parties or their privies regardless of
the form of the latter.
Jurisprudence expounds that the concept of res judicata embraces two aspects. The first, known as "bar by prior
judgment," or "estoppel by verdict," is the effect of a judgment as a bar to the prosecution of a second action upon
the same claim, demand or cause of action. The second, known as "conclusiveness of judgment," otherwise known
as the rule of auter action pendent, ordains that issues actually and directly resolved in a former suit cannot again be
raised in any future case between the same parties involving a different cause of action.
In this case, the issue raise by NHA regarding the validity of the title of the respondents was already settled in an
action to annul the titled which had already become final. Also, the Republic did not oppose the application for
registration of the lot in question.
94. DONA ROSANA REALTY AND DEVELOPMENT CORPORATION AND SY KA KIEND v.
MOLAVE DEVELOPMENT (2010)
RULE 16: Section 1, Rule 16 of the Rules of Civil Procedure provides that the trial court may dismiss a
complaint on the ground that the claim or demand set forth in the plaintiffs complaint has been paid,
waived, abandoned, or otherwise extinguished.

REM Digests ALS 2014B Justice Gesmundo


Medina owned land which sold to Molave Development, represented by its president Tinitgan for
P14million. Molave Development paid P1 million to Medina upon the signing of the contract and P1.3 million
more as first installment. But it refused to pay the rest of the installments on being informed by the Department of
Agrarian Reform (DAR) of the existence of alleged tenants on the land. Two years later, Medina wrote respondent
Molave Development a letter, rescinding the contract to sell between them. Molave Development later learned that
a month earlier, Medina sold the land to petitioner Doa Rosana Realty and Development Corporation.
The issue presented in this case is whether or not the CA erred in holding that no ground existed for dismissing
respondent Molave Developments complaint against petitioner Doa Rosana Realty given that such complaint
stated a cause of action.
It will be recalled that petitioner Doa Rosana Realty filed a motion with the RTC to hear and resolve its
affirmative defenses. The RTC did so and resolved to deny the motion. On a petition filed with the CA, however,
the latter court directed the RTC to hear and resolve Doa Rosana Realtys affirmative defense of good faith in
buying Medinas property. The RTC complied and, after hearing the evidence of the parties, dismissed the case,
holding that Doa Rosana Realty and its president were buyers of the property in good faith and Molave
Development did not have a cause of action against them. Clearly, The RTC did not dismiss the case on the ground
that the complaint did not state a cause of action, which is an entirely different matter.
Section 1, Rule 16 of the Rules of Civil Procedure provides that the trial court may dismiss a complaint on the
ground that the claim or demand set forth in the plaintiffs complaint has been paid, waived, abandoned, or
otherwise extinguished. This ground essentially admits the obligation set out in the complaint but points out that
such obligation has been extinguished, in this case apparently by abandonment after respondent Molave
Development received partial reimbursement from Medina as a consequence of the cancellation of contract to sell
between them.
96. THE MUNICIPALITY OF HAGONOY BULACAN v. HON. SIMEION DUMDUM (2010)
RULE 16: No other principle in remedial law is more settled than that when a motion to dismiss is filed, the
material allegations of the complaint are deemed to be hypothetically admitted This hypothetical admission
extends not only to the relevant and material facts well pleaded in the complaint, but also to inferences that
may be fairly deduced from them.
A complaint was filed by Lim Chao against the Municipality of Hagonoy,Bulacan for collection of sum of money
and damages. The complaint alleged that a contract was entered into by Lim Chao and the Municipality for the
delivery of motor vehicles, which supposedly were needed to carry out certain developmental undertakings in the
municipality. Lim Chao then delivered to the Municipality of Hagonoy 21 motor vehicles amounting to
P5,820,000.00. However, despite having made several deliveries, the Municipality allegedly did not heed Lim
Chaos claim for payment. Thus, she filed a complaint for full payment of the said amount, with interestand
damages and prayed for the issuance of a writ of preliminary attachment againstthe Municipality. The trial court
issued the Writ of Preliminary Attachment directing the sheriff "to attach the estate, real and personal properties" of
the Municipality.The Municipality filed a Motion to Dismiss on the ground that the claim on which the action had
been brought was unenforceable under the statute of frauds, pointing out that there was no written contract or
document that would evince the supposed agreement they entered into with respondent.
To begin with, the Statute of Frauds found in paragraph (2), Article 1403 of the Civil Code requires for
enforceability certain contracts enumerated therein to be evidenced by some note or memorandum. The term
Statute of Frauds is descriptive of statutes that require certain classes of contracts to be in writing; and that do not
deprive the parties of the right to contract with respect to the matters therein involved, but merely regulate the
formalities of the contract necessary to render it enforceable.
In other words, the Statute of Frauds only lays down the method by which the enumerated contracts may be
proved. But it does not declare them invalid because they are not reduced to writing inasmuch as, by law, contracts
are obligatory in whatever form they may have been entered into, provided all the essential requisites for their
validity are present. The object is to prevent fraud and perjury in the enforcement of obligations depending, for
evidence thereof, on the unassisted memory of witnesses by requiring certain enumerated contracts and transactions
to be evidenced by a writing signed by the party to be charged. The effect of noncompliance with this requirement
is simply that no action can be enforced under the given contracts.If an action is nevertheless filed in court, it shall

REM Digests ALS 2014B Justice Gesmundo


warrant a dismissal under Section 1(i), Rule 16 of the Rules of Court, unless there has been, among others, total or
partial performance of the obligation on the part of either party.
It has been private respondents consistent stand, since the inception of the instant case that she has entered into a
contract with petitioners. As far as she is concerned, she has already performed her part of the obligation under the
agreement by undertaking the delivery of the 21 motor vehicles contracted for by Ople in the name of petitioner
municipality. This claim is well substantiated at least for the initial purpose of setting out a valid cause of action
against petitioners by copies of the bills of lading attached to the complaint, naming petitioner municipality as
consignee of the shipment. Petitioners have not at any time expressly denied this allegation and, hence, the same is
binding on the trial court for the purpose of ruling on the motion to dismiss. In other words, since there exists an
indication by way of allegation that there has been performance of the obligation on the part of respondent, the case
is excluded from the coverage of the rule on dismissals based on unenforceability under the statute of frauds, and
either party may then enforce its claims against the other.
No other principle in remedial law is more settled than that when a motion to dismiss is filed, the material
allegations of the complaint are deemed to be hypothetically admitted This hypothetical admission extends not
only to the relevant and material facts well pleaded in the complaint, but also to inferences that may be fairly
deduced from them. Thus, where it appears that the allegations in the complaint furnish sufficient basis on which
the complaint can be maintained, the same should not be dismissed regardless of the defenses that may be raised by
the defendants. Stated differently, where the motion to dismiss is predicated on grounds that are not indubitable,
the better policy is to deny the motion without prejudice to taking such measures as may be proper to assure that the
ends of justice may be served.
It is interesting to note at this point that in their bid to have the case dismissed, petitioners theorize that there could
not have been a contract by which the municipality agreed to be bound, because it was not shown that there had
been compliance with the required bidding or that the municipal council had approved the contract. The argument
is flawed. By invoking unenforceability under the Statute of Frauds, petitioners are in effect acknowledging the
existence of a contract between them and private respondent only, the said contract cannot be enforced by action
for being non-compliant with the legal requisite that it be reduced into writing. Suffice it to say that while this
assertion might be a viable defense against respondents claim, it is principally a matter of evidence that may be
properly ventilated at the trial of the case on the merits.
99.) HONORIO BERNARDO VS. HEIRS OF EUSEBIO VILLEGAS (2010)
Estoppel sets in when a party participates in all stages of a case before challenging the jurisdiction of the
lower court.
This controversy stemmed from a Complaint for accion publiciana filed by Heirs of Eusebio Villegas against
Honorio Bernardo. The heirs had earlier filed an ejectment case MTC, which case was dismissed on the ground of
lack of jurisdiction for having been filed beyond the one-year prescriptive period for filing a forcible entry case.
The trial court held that the suit, being an accion publiciana, falls within its jurisdiction and held in favour of the
Heirs. CA affirmed. On appeal, Honorio questioned the jurisdiction of the trial court over the subject matter and
argued that in their complaint, the Heirs failed to state the assessed value of the property in dispute. The appellate
court ruled that Honorio is estopped from raising the issue of jurisdiction because he failed to file a motion to
dismiss on such ground and, instead, actively participated in the proceedings before the trial court. (To sum it up,
case goes on for 10 years ruled in favour of respondents. Petitioner now asserts that court of origin (RTC) has no
jurisdiction because assessed value was not given in the complaint. Estoppel na)
The general rule is that the jurisdiction of a court may be questioned at any stage of the proceedings. Lack of
jurisdiction is one of those excepted grounds where the court may dismiss a claim or a case at any time when
it appears from the pleadings or the evidence on record that any of those grounds exists, even if they were
not raised in the answer or in a motion to dismiss. The reason is that jurisdiction is conferred by law, and
lack of it affects the very authority of the court to take cognizance of and to render judgment on the action.
However, estoppel sets in when a party participates in all stages of a case before challenging the jurisdiction
of the lower court. One cannot belatedly reject or repudiate its decision after voluntarily submitting to its
jurisdiction, just to secure affirmative relief against one's opponent or after failing to obtain such relief. The

REM Digests ALS 2014B Justice Gesmundo


Court has, time and again, frowned upon the undesirable practice of a party submitting a case for decision
and then accepting the judgment, only if favorable, and attacking it for lack of jurisdiction when adverse.
102. SPOUSES FERNANDO TORRES AND IRMA TORRES, PETITIONERS, VS. AMPARO MEDINA
AND THE EX-OFFICIO SHERIFF OF THE RTC OF QUEZON CITY (2010)
Respondent Medina wrote a letter to the Office of the Sheriff, RTC, applying for the extrajudicial foreclosure of
mortgage of the property of petitioner Spouses Torres. The property was sold to Medina being the highest bidder. A
Certificate of Sale was issued to Medina. The Spouses Torres filed a Complaint before the RTC for the declaration
of nullity of the extrajudicial foreclosure of mortgage conducted by the Sheriff. The same was docketed as Civil
Case No. Q-99-38781. Medina filed a Motion to Dismiss raising the grounds of res judicata and forum
shopping. Medina argued that the Spouses Torres had filed an earlier Complaint praying for the annulment of the
real estate mortgage involving the same property and which was docketed as Civil Case No. Q-94-18962 before
the RTC of Quezon City. Medina contended that said complaint was already dismissed.
The elements of res judicata are: (1) the judgment sought to bar the new action must be final; (2) the decision must
have been rendered by a court having jurisdiction over the subject matter and the parties; (3) the disposition of the
case must be a judgment on the merits; and (4) there must be as between the first and second action identity of
parties, subject matter, and causes of action.
Spouses Torres admits to the presence to the first three elements but not the 4th. One test of proving identity of
cause of action is the "absence of inconsistency test" where it is determined whether the judgment sought will be
inconsistent with the prior judgment. If inconsistency is shown, the prior judgment shall not constitute a bar to
subsequent actions. This Court finds that the first three causes of action inevitably deal with the validity of the
real estate mortgage. The conclusion is certain in that any affirmative relief that this Court may grant on
said causes of action would affect the validity of the real estate mortgage; an issue which could no longer be
revived, as the same has been settled.
Anent the fourth cause of action in Civil Case No. Q-99-38781, this Court finds that the Spouses Torres had
already raised, in Civil Case No. 94-18962, the fact that 11 counts of B.P. Blg. 22 violations are pending with
MeTC. Thus, the RTC is correct in its observation that res judicata lies, as the RCBC checks referred to in
the complaint in Civil Case No. Q-99-38781 are the very same documents subject of Civil Case No. Q-94-
18962.
The foregoing findings notwithstanding, the Spouses Torres contend that the election of Medina to sue them
for violation of B.P Blg. 22 bars Medina from the remedy of foreclosure of mortgage. The argument of the
Spouses Torres is misplaced. The intention of the framers of B.P. Blg. 22 is to make the mere act of issuing a
worthless check malum prohibitum. That B.P. Blg .22 is not the collection suit contemplated by law can be
seen by the fact that the law seeks to punish the mere issuance of a bum check notwithstanding the
presence of damage or prejudice to the offended party. There can be no double compensation as the
indemnity award is distinct from the underlying obligation of the check. Thus, a person guilty of violating
B.P Blg. 22 may be subject to imprisonment or a fine at the discretion of the court and the fact that the
underlying obligation has been paid is of no moment.
129. Rizalina Positos v. Jacob Chua (2009) Rule 2; Rule 16
A dismissal without prejudice does not operate as a judgment on the merits. The proper remedy is not
appeal but to file a separate action. This does not violate the rule against the splitting of a single cause of
action.
Positos, member of Sto. Tomas de Villanueva Settlers Association (the Association), occupies land owned by
Ansuicio, Inc. together with other members of the Association. Ansuicio, Inc. transferred its rights and interests to
respondent Chua. The Association filed a complaint against Chua for prohibitory injuction. A compromise
agreement was reached and approved wherein the Association agreed to vacate the premises provided Chua extends
financial assistance to its members. The Association refused to abide by the compromise agreement. The conflict
was referred to conciliation before the Lupon. No settlement was reached so Chua filed a complaint for Unlawful
Detainer. In her answer, Positos alleges as one of her defenses that Chuas failure to appear personally during the
conciliation proceedings render the complaint dismissible. During the preliminary conference, the parties stipulated
Chuas failure to personally appear during conciliation. MTCC ruled in favor of Chua. As Positos failed to file a
REM Digests ALS 2014B Justice Gesmundo
supersedeas bond the stay the execution, the MTCC issued a Writ of Execution. Positos filed and was granted a
TRO to stay the execution. The RTC affirmed the MTCC Decision, holding that since Chua was duly represented
in the conciliation proceedings by an attorney-in-fact, the LGC was substantially complied with. Positos appealed
to the CA, which dismissed without prejudice the complaint of Chua on the ground of lack of cause of action,
having failed to comply with the barangay conciliation procedure. Positos filed an MR but the CA said that since
the complaint was dismissed without prejudice, Positos cuase of action should be ventilated in a separate action.
CA denied the MR. Positos filed a petition for review on certiorari under Rule 45. Positos argues that to compel her
to file a separate action for restoration to the premises runs counter to Sec. 3, Rule 2 which states the a party may
not institute more than one suit for a single cause of action.
The Court held that under Sec. 1, Rule 41, the remedy of appeal is not available from an order dismissing an
action without prejudice. Since Positos petition prays for a modification of the CAs decision, the SC cannot
treat it as one for certiorari, the allegations not being constitutive of grave abuse of discretion. Chuas
complaint was dismissed for failure to comply with the conciliation process, which affected the sufficiency of
his cause of action on the ground of prematurity. A dismissal without prejudice does not operate as a
judgment on the merits.
150. ASSOCIATED BANK v. SPS. JUSTINIANO S. MONTANO, SR. ET. AL. (2009) (RULE 16)
"The option of whether to set the case for preliminary hearing after the filing of an answer which raises
affirmative defenses, or to file a motion to dismiss raising any of the grounds set forth in Section 1, Rule 16 of
the Rules are procedural options which are not mutually exclusive of each other.
The Montanos own 3 parcels of land in Tanza, Cavite. When the President Marcos declared martial law, Justiniano
Montano a congressman of Cavite went on self-exile to the US. While on exile, the 3 parcels of land were
transferred by the Montanos to Tres Cruces Agro-Industrial Corp (TCAIC) in exchange for shares of stock.
Subsequently, TCAIC sold the properties to International Country Club Inc. (ICCI) and later on, it was mortgaged
to Associated Bank. When the loan matured and remained unpaid, it was foreclosed and Associated Bank as the
highest bidder became the owner and new TCTs were issued in its name. Upon the return of the Montanos they
took physical possession of the land, cultivated it and filed an action for reconveyance praying that the transfer of
properties from TCAIC to Associated Bank be declared null and void since they only transferred the land to TCAIC
to avoid the confiscatory acts of Marcos and the sale to ICCI was due to the fact that ICCI threatened the relatives
of Montano who were left in the country. 8years after Associated Bank filed its answer, the bank filed a Motion for
Preliminary Hearing on the Affirmative Defenses and/or Motion to Dismiss, for which the trial court issued an
Order dismissing the complaint for lack of cause of action and that it is barred by statute of limitations. Upon
appeal CA reversed the RTCs ruling.
The Court held that YES, the a motion to dismiss can be filed after filing of an answer. It is inconsequential
that petitioner had already filed an answer to the complaint prior to its filing of a motion to dismiss. The
option of whether to set the case for preliminary hearing after the filing of an answer which raises
affirmative defenses, or to file a motion to dismiss raising any of the grounds set forth in Section 1, Rule 16 of
the Rules are procedural options which are not mutually exclusive of each other. Moreover, as petitioner
correctly pointed out, respondents failed to oppose the motion to dismiss despite having been given the
opportunity to do so by the RTC. Therefore, any right to contest the same was already waived by them.
The Court also ruled that NO, the case cannot be dismissed for failure of the complaint to state a cause of
action nor can it be dismissed due to prescription. When the ground for dismissal is that the complaint states
no cause of action, such fact can be determined through the complaint the test being whether, assuming the
allegations of fact in the complaint to be true, a valid judgment could be rendered in accordance with the
prayer stated therein. Where the allegations are sufficient but the veracity of the facts is assailed, the motion
to dismiss should be denied. As in this case, the complaint on its face clearly states a cause of action and
raises issues of fact that can be properly settled only after a full-blown trial. For the issue on prescription the
court said that although it is true that an action for reconveyance of real property resulting from fraud may
be barred by the statute of limitations, which requires that the action shall be filed within 4 years from the
discovery of the fraud, the RTC have overlooked the fact that the basis of respondents complaint for
reconveyance is not fraud but threat, duress and intimidation, allegedly employed by Marcos cronies upon
the relatives of the Montanos while the latter were on self-exile. In fact, fraud was neither specifically alleged

REM Digests ALS 2014B Justice Gesmundo


nor remotely implied in the complaint. According to Art 1391 of the Civil Code, in cases of intimidation,
violence or undue influence the action for annulment must be brought within 4 yrs from the time the defect
of consent ceases. Since it must be reckoned from the date of the ouster of Marcos, the action for
reconveyance is filed on time (Feb 21, 1986: ouster; Sept 21, 1989: action).
177. J. C. LOPEZ & ASSOCIATES, INC vs. COMMISSION ON AUDIT and NATIONAL POWER
CORPORATION (2001)
Material facts or questions which were in issue in a former action and were there admitted or judicially
determined are conclusively settled by a judgment rendered therein and that such facts or questions become
res judicata and may not again be litigated in a subsequent action between the same parties or their privies,
regardless of the form the issue may take in the subsequent action
Petitioner entered into a contract with the National Power Corporation (NAPOCOR) for the dredging of the vicinity
of the Intake Tower at the Ambuklao Hydroelectric Plant in Bokod, Benguet. Pursuant to the contract, NAPOCOR
paid 15% of the contract price and subsequently paid the balance of the mobilization cost.(This mobilization cost
however was later disallowed by COA.) However, due to the delays incurred by petitioner, the contract was
terminated by NAPOCOR. This lead NAPOCOR to enter into a new contract with another servicer, MIESCOR.
Petitioner filed a complaint for injunction in the Regional Trial Court assailing NAPOCORs termination of its
contract and enjoining NAPOCOR to proceed with the new contract with MIESCOR. TC granted the complaint and
ruled that dredging operations were not covered under the term infrastructure project of PD 1818 and thus may be
enjoined. MIESCOR filed then a petition for certiorari with the CA prohibiting the TC from enforcing the writ of
injunction. CA then ruled that the TC committed GADLEJ and dredging operations are indeed covered by the term
infrastructure project under PD 1818. This CA decision BECAME FINAL AND EXECUTORY.
Meanwhile, the petitioner is questioning the suspension and eventual disallowance of payment by the COA
concerning the mobilization cost. COA disallowed the payment of the mobilization cost because it is illegal
considering that it is beyond the limit allowed by law. (Essentially, petitioners argument is that dahil hindi nga siya
infrastructure project, hindi siya covered sa 15% limit cap imposed by law) It sought a reconsideration of the COA
decision but was denied and thus it filed this instant case for certiorari.
Material facts or questions which were in issue in a former action and were there admitted or judicially
determined are conclusively settled by a judgment rendered therein and that such facts or questions become
res judicata and may not again be litigated in a subsequent action between the same parties or their privies,
regardless of the form the issue may take in the subsequent action, whether the subsequent action involves
the same or a different form of proceeding, or whether the second action is upon the same or a different
cause of action, subject matter, claim or demand, as the earlier action. In such cases, it is also immaterial
that the two actions based on different grounds, or tried on different theories, or instituted for different
purposes, and seek different reliefs. By the same token, whatever is once irrevocably established as the
controlling legal principle or decision continues to be the law of the case between the same parties in the
same case, whether correct on general principles or not, so long as the facts on which such decision was
predicated continue to be the facts of the case before the court. In the instant case, the issue of whether or
not the dredging contract between the petitioner and NAPOCOR involves an infrastructure project as
defined in Executive Order No. 380, was already passed upon and resolved by the Court of Appeals in
Meralco Industrial Engineering Services Corporation vs. Hon. Romeo F. Zamora and J.C. Lopez, Inc.,
docketed as CA-G.R. SP No. 30141. Consequently, upon attaining finality, the said decision became the law
of the case and constituted a bar to any re-litigation of the same issue in any other proceeding under the
principle of res judicata.
199. RIAMBUNAN v. ORIENTAL WOOD (2005)
"an order denying a motion to dismiss is an interlocutor order which cannot be questioned in a special civil
action for certiorari (MOTION TO DISMISS)
A complaint for sum of money was filed by Rimbunan Hijau Group of Companies and Niugini Lumber Merchants
Pty., Ltd. against Oriental Wood Processing Corporation. Rimbunan and Niugini are foreign corporations duly
organized and existing under the laws of Papua New Guinea. A Motion to Dismiss was filed by Oriental Wood on

REM Digests ALS 2014B Justice Gesmundo


the ground that Rimbunan and Niugini has no legal capacity to sue in this jurisdiction. This motion was denied
because the petitioners were not doing business in the Philippines but were merely suing on an isolated transaction.
An order denying a motion to dismiss is an interlocutory order which neither terminates nor finally disposes
of a case as it leaves something to be done by the court before the case is finally decided on the merits. As
such, the general rule is that the denial of a motion to dismiss cannot be questioned in a special civil action
for certiorari which is a remedy designed to correct errors of jurisdiction and not errors of judgment. To
justify the grant of the extraordinary remedy of certiorari, therefore, the denial of the motion to dismiss
must have been tainted with grave abuse of discretion. The abuse of discretion must be grave as where the
power is exercised in an arbitrary or despotic manner by reason of passion or personal hostility and must be
so patent and gross as to amount to an evasion of positive duty or to a virtual refusal to perform the duty
enjoined by or to act all in contemplation of law.
200. PANGANIBAN v. PILIPINAS SHELL PETROLEUM CORPORATION (2003)
"the mere fact that the action for declaratory relief was filed earlier than the case for unlawful detainer does
not necessarily mean that the first case will be given preference (LITIS PENDENTIA)"
Panganiban entered into a Sublease and Dealer Agreement for a gasoline station with Pilipinas Shell Petroleum
Corporation (PSPC). A letter was sent by PSPC to Panganiban informing her of the expiration of the agreement on
1995. However, she continued paying rentals until 2002 because she believed that the agreement was still
effective. PSPC refused to accept the payments, so Panganiban filed a petition for declaratory relief with the
RTC. An Unlawful Detainer case was filed and decided against her. Panganiban appealed the case and is now
pending with the RTC. Then, the RTC ordered the dismissal of the petition for declaratory relief on the ground of
litis pendentia.
Litis pendentia as a ground for the dismissal of a civil action refers to that situation wherein another action is
pending between the same parties for the same cause of actions and that the second action becomes
unnecessary and vexatious. The following factors must be considered to determine which case should be
dismissed given the pendency of two actions:
1. the date of filing, with preference generally given to the first action filed to be retained;
2. whether the action sought to be dismissed was filed merely to preempt the latter action or to anticipate
its filing and lay the basis for its dismissal; and
3. whether the action is the appropriate vehicle for litigating the issues between the parties
The mere fact that the action for declaratory relief was filed earlier than the case for unlawful detainer does
not necessarily mean that the first case will be given preference.

RULE 17

64. SHIMIZU PHILIPPINES CONTRACTORS, INC. vs. MRS. LETICIA B. MAGSALIN, doing business
under the trade name KARENS TRADING, ET AL. (2012) [Sec. 3, Rule 17 - Dismissal; Sec. 1, Rule 36 -
Judgment or final order; Rule 41 - Appeal]
(a) When a complaint is dismissed for failure to prosecute and the dismissal is unqualified, the dismissal has
the effect of an adjudication on the merits. Thus, the order must conform with Sec. 1, Rule 36. (b) A
determination of the existence of a factual basis for the dismissal of a case involves a question of fact
properly cognizable by the CA on petition for review under Rule 41. (c) The fundamental test for non
prosequitur is whether, under the circumstances, the plaintiff is chargeable with want of due diligence in
failing to proceed with reasonable promptitude. There must be unwillingness on the part of the plaintiff to
prosecute.
Shimizu filed a complaint for damages arising from breach of contract against Magsalin and FGU Insurance. The
RTC dismissed the case, merely stating in the order: For failure of [petitioner] to prosecute, the case is hereby
DISMISSED. SO ORDERED. Shimizu appealed the case to the CA via petition for review under Rule 41. The CA
REM Digests ALS 2014B Justice Gesmundo
denied the appeal on the ground of lack of jurisdiction ruling that since the facts presented in the appeal were not
disputed, the appeal raises a pure question of law; hence, the proper remedy is an appeal filed directly with the SC
under Rule 45.
Held:
1. The Dismissal Order is Void. When a complaint is dismissed for failure to prosecute and the dismissal is
unqualified, the dismissal has the effect of an adjudication on the merits. As an adjudication on the merits,
the dismissal order must conform with Sec. 1, Rule 36 of the Rules of Court on the writing of valid
judgments and final orders. The dismissal order clearly violates this rule for its failure to disclose how and why
the petitioner failed to prosecute its complaint. It simply states its conclusion that the case should be dismissed for
non prosequitur, a legal conclusion, but does not state the facts on which this conclusion is based.
2. The appeal was properly filed under Rule 41 of the Rules of Court. Where a case is submitted upon an agreement
of facts, or where all the facts are stated in the judgment and the issue is the correctness of the conclusions drawn
therefrom, the question is one of law which is properly subject to the review of the SC. In this case, however, this
rule does not apply since the facts supposedly supporting the trial courts conclusion of non prosequitur were not
stated in the judgment. At any rate, the appeal in this case necessarily involved questions of fact. A critical factual
question that must be answered by the CA is whether, based on the records, there had been factual basis for the
dismissal of the complaint. This question is particularly significant given that the dismissal order appealed from
does not even indicate the factual basis for the dismissal of the case. There is thus a need to delve into the records to
check whether facts to justify the prejudicial dismissal even exist.
3. The dismissal by the RTC is not supported by the facts of the case. None of the events that occurred prior to the
issuance of the dismissal order square with the grounds specified by Sec. 3, Rule 17 for the motu proprio dismissal
of a case for failure to prosecute. The fundamental test for non prosequitur is whether, under the
circumstances, the plaintiff is chargeable with want of due diligence in failing to proceed with reasonable
promptitude. There must be unwillingness on the part of the plaintiff to prosecute. In this case, the parties
own narrations of facts demonstrate the petitioners willingness to prosecute its complaint. Indeed, the respondents
were not able to point to any specific act committed by the petitioner to justify the dismissal of their case.
87. PCI Leasing and Finance, Inc. v. Antonio Milan under A Milan Trading, et. al., G.R. No. 151215
(2010) Rule 17
Instances when a complaint may be dismissed due to the plaintiffs fault: (1) if he fails to appear on the date
for the presentation of his evidence in chief on the complaint; (2) if he fails to prosecute his action for an
unreasonable length of time; or (3) if he fails to comply with the Rules or any order of the court.
Petitioner PCI Leasing and Finance, Inc. (PCI Leasing) filed of a Complaint for Sum of Money against herein
respondents Antonio C. Milan (Antonio) and Laura M. Milan. PCI Leasing alleged that it extended loans to
respondents for which Deeds of Assignment were duly executed by respondents. Eventually, despite repeated
demands, respondents failed to settle their obligation. PCI Leasing was then compelled to litigate to enforce
payment of the total loan obligation, plus interests, penalties, attorneys fees, expenses of litigation and costs of
suit. The RTC issued summons to respondents, however the summons and the copy of the complaint were, returned
unserved.
In view of the above situation, PCI Leasing filed a Motion to Archive the civil case subject to its reinstatement after
the whereabouts of the respondents was determined. The RTC denied. PCI Leasing filed a Motion for Issuance of
Alias Summons but said motion was denied. Petitioner filed another Motion for Issuance of Alias Summons, which
the RTC scheduled for hearing. During the hearing of the motion on said date, there was no appearance from both
counsels of PCI Leasing and respondents; the RTC dismissed the case for failure to prosecute. PCI Leasing filed an
MR but was denied. PCI Leasing filed an Ex Parte MR but was again denied.
The Court held that RTC grievously erred in dismissing the case. The dismissal of a case for failure to
prosecute has the effect of adjudication on the merits, and is necessarily understood to be with prejudice to
the filing of another action, unless otherwise provided in the order of dismissal. Stated differently, the
general rule is that dismissal of a case for failure to prosecute is to be regarded as an adjudication on the
merits and with prejudice to the filing of another action, and the only exception is when the order of
dismissal expressly contains a qualification that the dismissal is without prejudice.
REM Digests ALS 2014B Justice Gesmundo
Furthermore, in Marahay v. Melicor, the Court held that [w]hile a court can dismiss a case on the ground of
non prosequitur, the real test for the exercise of such power is whether, under the circumstances, plaintiff is
chargeable with want of due diligence in failing to proceed with reasonable promptitude. In the absence of a
pattern or scheme to delay the disposition of the case or a wanton failure to observe the mandatory
requirement of the rules on the part of the plaintiff, as in the case at bar, courts should decide to dispense
with rather than wield their authority to dismiss.
108. CHAN-TAN v. TAN ( FEBRUARY 25, 2010) Rule 17
Nothing is more settled in law than that when a judgment becomes final and executory, it becomes
immutable and unalterable. In its March 30, 2004 Decision, the trial court declared the marriage between
petitioner Susie Chan-Tan and respondent Jesse Tan void under Article 36 of the Family Code. In its May 17, 2004
Resolution, the trial court granted to respondent custody of the children, ordered petitioner to turn over to
respondent documents and titles in the latters name, and allowed respondent to stay in the family dwelling. In
its February 15, 2005 Resolution, the trial court denied petitioners motion for reconsideration of the December 28,
2004 Resolution denying petitioners motion to dismiss and motion for reconsideration of the October 12 2004
Resolution, which in turn denied for late filing petitioner's motion for reconsideration of the May 17, 2004
resolution. Respondent Tan maintains that the March 30, 2004 decision and the May 17 , 2004 resolution of the
trial court are now final and executory and could no longer be reviewed, modified, or vacated.
Section 1 of the Rule states that the Rules of Court applies suppletorily to a petition for the declaration of
absolute nullity of void marriage or the annulment of voidable marriage. In this connection, Rule 17 of the
Rules of Court allows dismissal of the action upon notice or upon motion of the plaintiff, to wit:
Section 1. Dismissal upon notice by plaintiff. A complaint may be dismissed by the plaintiff by filing a notice
of dismissal at any time before service of the answer or of a motion for summary judgment. Upon such notice
being filed, the court shall issue an order confirming the dismissal. x x x
Section 2. Dismissal upon motion of plaintiff. Except as provided in the preceding section, a
complaint shall not be dismissed at the plaintiffs instance save upon approval of the court and upon
such terms and conditions as the court deems proper. x x x (Emphasis supplied)
However, when petitioner filed the motion to dismiss on 4 November 2004, the 30 March 2004 decision and
the 17 May 2004 resolution of the trial court had long become final and executory upon the lapse of the 15-
day reglementary period without any timely appeal having been filed by either party. The 30 March 2004
decision and the 17 May 2004 resolution may no longer be disturbed on account of the belated motion to
dismiss filed by petitioner. The trial court was correct in denying petitioners motion to dismiss. Nothing is
more settled in law than that when a judgment becomes final and executory, it becomes immutable and
unalterable. The same may no longer be modified in any respect, even if the modification is meant to correct
what is perceived to be an erroneous conclusion of fact or law. The reason is grounded on the fundamental
considerations of public policy and sound practice that, at the risk of occasional error, the judgments or
orders of courts must be final at some definite date fixed by law. Once a judgment has become final and
executory, the issues there should be laid to rest.
124.) BENEDICTA M. SAMSON VS. HON. JUDGE GERALDINE C. FIEL-MACARAIG, G.R. NO. 166356
(2010)
Rule 17, Sec. 3: Dismissal due to fault of plaintiff.
Petitioners obtained a loan from FEBTC. The loan was secured by a real estate mortgage over four parcels of land
located in Marikina City. When petitioners failed to comply with the terms of the loan agreement, FEBTC filed an
application for extra-judicial foreclosure. Application was given due course and a public auction sale was
conducted with prior publication. Mortgaged properties were sold to FEBTC as the highest bidder. After two years,
Petitioners filed a case for Annulment of Extra-Judicial Foreclosure and/or Nullification of Sale and the Certificates
of Title. They questioned the auction sale for alleged lack of posting and publication requirements. RTC denied and
issued an order dismissing the complaint for failure to prosecute for an unreasonable length of time. CA affirmed.
Petitioners claimed that RTC judge gravely abused her discretion when she dismissed the case for failure to
prosecute despite that one of the defendants failure to file a responsive pleading, hence issues have not yet been
joined and move for pre-trial wouldve been premature. Court ruled against petitioners. Lower court did not act
REM Digests ALS 2014B Justice Gesmundo
in excess of jurisdiction or with GADLEJ. Petitioners failed to move for pre-trial in accordance with Rule 18
Sec1 and case was dismissed pursuant to Rule 17 Sec3 which provides If, for no justifiable cause, the
plaintiff fails to appear on the date of the presentation of his evidence in chief on the complaint, or to
prosecute his action for an unreasonable length of time, or to comply with these rules or any order of the
court, the complaint may be dismissed upon motion of the defendant or upon the courts own motion,
without prejudice to the right of the defendant to prosecute his counterclaim in the same or in a separate
action. This dismissal shall have the effect of an adjudication upon the merits, unless otherwise declared by
the court.

RULE 18

89. CECILIA GADRINAB SENARLO v. JUDGE MAXIMO PADERANGA (2010)


Mediation is part of Pre-trial. Failure to appear in mediation shall have the effect of failure to appear in Pre-
trial. Failure of Plaintiff to appear in mediation shall be cause for dismissal of action.
A civil case is on its pre-trial stage. Judge Paderanga referred the case for mediation and directed the parties to
appear on Nov. 4, 2005. Subsequently, Pres. Arroyo declared Nov. 4 as a legal holiday in celebration of the Feast of
Ramadan. Both parties did not appear in the Mediation. Judge Paderanga ordered the dismissal of the action.
Plaintiff in the civil case filed an administrative complaint against Judge Paderanga for gross ignorance of the law
and grave abuse of authority. SC held that the judge is only guilty of simple misconduct.
A.M. No. 01-10-5-SC-PHILJA dated 16 October 2001, otherwise known as the Second Revised Guidelines for the
Implementation of Mediation Proceedings and Section 5, Rule 18 of the Rules of Court grant judges the
discretion to dismiss an action for failure of the plaintiff to appear at mediation proceedings.
A.M. No. 01-10-5-SC-PHILJA considers mediation a part of pre-trial and states that since mediation is part of Pre-
Trial, the trial court shall impose the appropriate sanction including but not limited to censure, reprimand, contempt
and such sanctions as are provided under the Rules of Court for failure to appear for pre-trial, in case any or both of
the parties absent himself/themselves, or for abusive conduct during mediation proceedings.
Under Rule 18, Section 5 of the Rules of Court, failure of the plaintiff to appear at pre-trial shall be cause for
dismissal of the action:
SEC. 5. Effect of failure to appear. - The failure of the plaintiff to appear when so required pursuant to the next
preceding section shall be cause for dismissal of the action. The dismissal shall be with prejudice, unless otherwise
ordered by the court. A similar failure on the part of the defendant shall be cause to allow the plaintiff to present his
evidence ex parte and the court to render judgment on the basis thereof.
The judge had legal basis for ordering the dismissal. However, he failed to consider that the date set for
Mediation was declared a legal holiday. The declaration of November 4, 2005 as a holiday was a development
totally outside the Plaintiffs control for which she should not be sanctioned with the dismissal. The judge
failed to exercise the necessary diligence, which makes him liable for simple negligence.
141. JAZMIN L. ESPIRITU, ET. AL., v. VLADIMIR G. LAZARO, ET. AL. (2009) [Rule 18 Pre Trial]
After last pleading is served AND filed, plaintiff must set case for PRE-TRIAL; otherwise, case could be
dismissed; A Cautionary Answer is considered a last pleading.
This is a complaint for recovery of personal property with damages and preliminary attachment. The defendant
served and filed a Cautionary Answer with Manifestation and Motion to File a Supplemental/Amended Answer.
The plaintiff never set the case for pre-trial. He contends that he is waiting for the court to resolve the motion to file
supplemental / amended answer. The court dismissed the case after one year because a pre-trial was never set.
Held: Section 1 of Rule 18 of the Rules of Court imposes upon the plaintiff the duty to set the case for pre-
trial after the last pleading is served and filed. Under Section 3 of Rule 17, failure to comply with the said duty

REM Digests ALS 2014B Justice Gesmundo


makes the case susceptible to dismissal for failure to prosecute for an unreasonable length of time or failure
to comply with the rules.
Since respondents in the case already filed a cautionary answer and petitioners did not file any reply to it, the case
was already ripe for pre-trial. There is no need to wait for court to resolve the motion included therein.
Dismissal is allowed even if it is not raised that the plaintiff lacks interest to prosecute the action, or that there is
prejudice to the defendant resulting from the failure of the plaintiff to comply with the rules.
197. PEOPLE v. BOCBOSILA (2007)
"the matters agreed upon in the pre-trial conference and as stated in the pre-trial order shall bind the
parties (PRE-TRIAL)"
This is a murder case against Bocbosila. In the Pre-Trial Order, the defense named only four witnesses. In the same
order, the RTC informed all parties that witnesses and documents which were not mentioned in the Order shall not
be entertained during the trial on the merits. During the trial, only two were able to testify. The other two witnesses
failed to appear and testify in court several times. So the defense counsel moved to substitute them explaining that
they were hesitant to testify, and that one of them went home to his province. Bocbosila avers that his
constitutional right to produce evidence on his behalf and to due process were violated when the trial court denied
the motion of his counsel to present substitute witness.
Section 4, Rule 118 of the Revised Rules on Criminal Procedure mandates that the matters agreed upon in
the pre-trial conference and as stated in the pre-trial order shall bind the parties. The pre-trial order of the
RTC dated 29 February 2000 clearly shows that the defense named only four witnesses. The parties were
also informed therein that witnesses who were not mentioned in the pre-trial order will not be entertained
during the trial on the merits. Thus, pursuant to Section 4, Rule 118 and its purpose of preventing undue
delay in the disposition of criminal cases and ensuring fair trial, the denial of the defense counsels motion
for substitution of witnesses is justified.
198. BAYAS v. SANDIGANBAYAN (2002)
"stipulations freely and voluntarily made are valid and binding and will not be set aside unless for good
cause (PRETRIAL)"
This is a trial for malversation through falsification. Petitioners Matuday and Bayas pled not guilty during their
arraignment. A Joint Stipulation of Facts and Documents was submitted by the parties, it was duly signed by the
two accused, their lawyer, and the prosecutor. Thereafter, the accused, represented by a new lawyer, moved to
withdraw the Joint Stipulation of Facts and Documents.
According to Section 2, Rule 118, for a pretrial agreement to be binding on the accused, it must satisfy the
following conditions: (1) the agreement or admission must be in writing, and (2) it must be signed by both
the accused and their counsel. The courts approval is not needed to make the stipulations binding on the
parties. Such approval is necessary merely to emphasize the supervision by the court over the case and to
enable it to control the flow of the proceedings. Once the stipulations are reduced into writing and signed by
the parties and their counsels, they become binding on the parties who made them. They become judicial
admissions of the fact or facts stipulated. Even if placed at a disadvantageous position, a party may not be
allowed to rescind them unilaterally; it must assume the consequences of the disadvantage. If the accused
are allowed to plead guilty under appropriate circumstances, by parity of reasoning, they should likewise be
allowed to enter into a fair and true pre-trial agreement under appropriate circumstances.

RULE 19

55. ONGCO v. DALISAY (2012)


intervention is not a matter of right, but is left to the discretion of the court; intervention can only be filed at any
time before rendition of judgment by the trial court

REM Digests ALS 2014B Justice Gesmundo


Rule 19
Dalisay applied for registration of a parcel of land. Other than the Republic, there was no oppositor nor written
oppositions at the hearing, so a general order of default was issued. The trial court rendered judgment in favor of
Dalisay, and the Republic appealed. While the case was pending appeal, Ongco filed a motion for leave to intervene
with the CA, claiming that he had a pending free patent claim with the DENR over the same parcel of land. The CA
denied the motion.
The Court ruled that intervention was improper. In order for intervention to be proper, the intervenor must show
that he/she has legal interest in the matter in litigation, or in the success of either party, or an interest against both,
or when he/she will be adversely affected by the distribution or disposition of property held by the court. The court,
moreover, must likewise consider whether the intervention will unduly delay or prejudice the adjudication of the
rights of the parties, and whether the intervenors rights can be fully protected in a separate proceeding. Finally,
intervention is allowed only before the rendition of judgment by the trial court.
Intervention is not a matter of right but is left to the trial courts discretion. The trial court must not only determine
if the requisite legal interest is present, but must also consider the delay and prejudice to the original parties the
intervention might cause. Both requirements must concur. The reason for allowing intervention only before
rendition of judgment by the trial court is to prevent delay.
In this case, Ongco has not shown any legal interest, as the free patent claim is merely pending, and thus indirect,
contingent, and inchoate. The motion to intervene was likewise filed out of time, as the trial court had already
rendered judgment. Even if not filed out of time, the proper remedy would have been a motion to lift order of
default, not a motion to intervene, as land registration proceedings are in rem and thus binding on the whole world.
Finally, Ongco still has a remedy, as he can file a petition for reopening and reviewing the decree of registration
within 1 year from entry if actual fraud is claimed.

RULE 20

34. GSIS v. Executive Judge Cancino-Erum et al (2012) (Rule 20, Assignment of cases)
[based on a practice pre-dating the 2007 ROC which requires the assignment of cases to be done exclusively by
raffle, the Executive Judge would assign TRO/Injunction cases on a round system. If one of the 4 courts had
a pending TRO case, it would not be included in the raffle for the next TRO case. Here, 3 of the 4 courts already
had pending TRO cases so the present TRO case was assigned to the only court left insomuch as it was the only
court without a TRO case for that round.]
The avowed purpose of instituting the raffle as the EXCLUSIVE METHOD of assigning cases among
several branches of a court is two-fold:
(1) to equalize the distribution of cases among the several branches, (speedy disposition);
(2) to ensure the impartial adjudication of cases (obviate suspicion of predetermined judges)
Court declared that it cannot sanction any practice that does not conform to the raffle as the exclusive
method of assigning cases among several branches within a judicial region. It reiterated that the raffle
should always be the rule rather than the exception.
[exceptions are enumerated in Circular No. 7 Sept. 23, 1974. Example: special raffle at the instance of an
interested party where the matter is incidental or interlocutory and of urgent nature that it cannot wait for a
regular raffle]
RULE 21

RULE 22

REM Digests ALS 2014B Justice Gesmundo


RULE 23

RULE 24

RULE 25

RULE 26

52. SPOUSES VILLUGA V. KELLY HARDWARE AND CONSTRUCTION SUPPLY INC., G.R. NO.
176570 (2012)
For Rules 10, 26, and 35. The Second Amended Complaint supersedes only its Amended Complaint and
nothing more; A request for admission that merely reiterates the allegations in an earlier pleading is
inappropriate under Rule 26 of the Rules of Court; For summary judgment to be proper (1) there must be
no genuine issue as to any material fact, except for the amount of damages and (2) the party presenting the
motion for summary judgment must be entitled to a judgment as a matter of law.
Kelly Hardware filed a complaint for sum of money and damages against the petitioner spouses before the RTC of
Bacoor amounting to Php 259,809.50, which corresponds to various construction materials that the spouses
allegedly failed to pay for. The spouses claimed to have made partial payments in their Answer. The respondent
then filed an Amended Complaint, claiming that the aggregate value of the unpaid materials amounted to Php
279,809.50, that only Php 20,000 of such amount had been paid, leaving a balance of Php 259,809.50. The
respondent thereafter filed a Request for Admission, asking petitioners to admit the genuineness and truth of
various documents which show that petitioners liability indeed amounted to Php 259,809.50. Respondent filed
their Comment to the Request for Admission out of time. Subsequently, however, a SECOND Amended Complaint
was filed by respondent, amending the period covered by complaint, and alleging that although partial payment was
made, such payment covered prior obligations of petitioners not included contemplated in the present complaint.
After petitioners filed their Answer, respondent filed a Motion, praying that the Comment of the petitioners on their
Request for Admission be expunged for being filed out of time, that the petitioners be considered to have admitted
the matters covered by the Request, and to be granted summary judgment in that regard. The Motion was granted.
Petitioners appeal and subsequent Motion for Reconsideration grounded upon the abandonment of the earlier
Request for Admission by virtue of the filing of the Second Amended Complaint were unavailing, hence this
petition for review on certiorari.
The Second Amended Complaint supersedes only its Amended Complaint and nothing more. The
respondents Request for Admission is not deemed abandoned or withdrawn by the filing of the Second
Amended Complaint. Despite this, the petitioners cannot be said to have impliedly admitted their liability in
the amount of Php 259,809.50. This is because the Request for Admission merely reiterated respondents
main allegation in its Amended Complaint, against which the petitioners had already set up the affirmative
defense of partial payment. If the factual allegations in the complaint are the very same allegations set forth
in the request for admission and have already been specifically denied, the required party cannot be
compelled to deny them anew. A request for admission that merely reiterates the allegations in an earlier
pleading is inappropriate under Rule 26 of the Rules of Court.
However, notwithstanding the defense of partial payment, summary judgment was still proper. When the
pleadings on file show that there are no genuine issues of fact to be tried, the Rules of Court allow a party to
obtain immediate relief by way of summary judgment. Conversely, where the pleadings tender a genuine
issue, summary judgment is not proper. A genuine issue" is such issue of fact which requires the
presentation of evidence as distinguished from a sham, fictitious, contrived or false claim. Since the
petitioners failed to specifically deny the respondents allegation in its Second Amended Complaint to the
effect that the partial payments claimed to have been made were applied to prior obligations not covered by

REM Digests ALS 2014B Justice Gesmundo


the present complaint, petitioners the Answer to Second Amended Complaint no longer raised genuine
issues of fact.

RULE 27

RULE 28

RULE 29

RULE 30

RULE 31

RULE 32

RULE 33

RULE 34

59. First Leverage and Services Group, Inc. v. Solid Builders, Inc. (2012)
Rule 34, Sec. 1 summary judgment; Rule 35 In a proper case for judgment on the pleadings, there is no
ostensible issue at all because of the failure of the defending partys answer to raise an issue. The answer
would fail to tender an issue, of course, if it does not deny the material allegations in the complaint or admits
said material allegations of the adverse partys pleadings by confessing the truthfulness thereof and/or
omitting to deal with them at all. In the case of a summary judgment, issues apparently exist i.e., facts are
asserted in the complaint regarding which there is as yet no admission, disavowal or qualification; or specific
denials or affirmative defenses are in truth set out in the answer but the issues thus arising from the
pleadings are sham, fictitious or not genuine, as shown by affidavits, depositions, or admissions.
First Leverage filed a complaint for annulment of promise to sell, mandamus and prohibitory injunction against
PNB and Solid Builders. PNB and Solid Builders had an agreement that the parcels of land owned by PNB will be
sold to Solid Builders. First Leverage alleged that there was no valid perfected contract to sell since it was not
approved by PNBs board of directors and hence, the act was ultra vires. Both PNB and Solid Builders filed their
respective answers, stating that the complaint states no cause of action because the PNB board validly approved the
contract. First Leverage moved for a judgment on the pleadings. The RTC thus rendered a judgment based on the
pleadings and ruled in favor of First Leverage. The Supreme Court held that what was rendered by the RTC is not a
judgment on the pleadings but a summary judgment. In the present case, Solid Builders' pleadings tendered factual
issues. Hence, the RTC rendered a summary judgment and not a judgment on the pleadings. However, even a
summary judgment is not proper in the instant case. There can be no summary judgment where questions of
fact are in issue or where material allegations of the pleadings are in dispute. A party who moves for
summary judgment has the burden of demonstrating clearly the absence of any genuine issue of fact, or that
the issue posed in the complaint is so patently unsubstantial as not to constitute a genuine issue for trial, and
any doubt as to the existence of such an issue is resolved against the movant. In the present case, the Court
held that genuine issues exist which call for a full blown trial. First Leverage asserted in its amended complaint

REM Digests ALS 2014B Justice Gesmundo


that there was no such valid perfected contract to sell. PNB insisted in its answer that the board of directors validly
approved the award of the properties to Solid Builders. Its co-defendant, Solid Builders, maintained also in its
answer that the perfection, approval and execution of the deed of promise to sell in its favor were legal and not ultra
vires. Thus, PNB Republic's and Solid Builders' respective answers to the complaint tendered an issue.
Genuine issues of fact were properly raised before the RTC, particularly with regard to the validity and existence of
a perfected contract to sell, and that these issues could only be resolved through a full-blown hearing. Therefore, a
summary judgment is not proper in the instant case.
125. DORIS SUNBANUN VS. AURORA GO (2010) [RULE 34]
The trial court has the discretion to grant a motion on the pleadings filed by a party if there is no
controverted matter in the case after the answer is filed
Petitioner owns a residential lot which the ground floor she leases to respondent. Their lease contract expires on
April 1996. Respondent subleases the premises to her relatives. Petitioner drove away the sublessees and told them
that they can only stay up to April. They left and petitioner padlocked the premises. Respondent sued for damages
due to the premature termination of the lease contract and for lost income. During trial, both parties assented to a
judgment on the pleadings since the only contention is the correct interpretation of the lease contract. Petitioner, in
moving for a judgment on the pleadings without offering proof as to the truth of her own allegations and
without giving respondent the opportunity to introduce evidence, is deemed to have admitted the material
and relevant averments of the complaint, and to rest her motion for judgment based on the pleadings of the
parties. In this case, it is undisputed that petitioner ejected respondents lodgers three months before the
expiration of the lease contract on 7 July 1996. Petitioner maintains that she had the right to terminate the
contract prior to its expiration because respondent allegedly violated the terms of the lease contract by
subleasing the rented premises. Petitioners assertion is belied by the provision in the lease contract which
states that the lessee can use the premises as a dwelling or as lodging house. Furthermore the lease
contract clearly provides that petitioner leased to respondent the ground floor of her residential house for a
term of one year commencing from 7 July 1995. Thus, the lease contract would expire only on 7 July 1996.
However, petitioner started ejecting respondents lodgers in March 1996 by informing them that the lease
contract was only until 15 April 1996. Clearly, petitioners act of ejecting respondents lodgers resulted in
respondent losing income from her lodgers. Hence, it was proper for the trial court and the appellate court
to order petitioner to pay respondent actual damages in the amount of P45,000.

RULE 35

54. SOLLER v. HEIRS OF ULAYAO (2012)


defense of acquisitive prescription is question of fact
Rule 35
Spouses Soller allegedly own a parcel of land in Mindoro, covered by a TCT, which land was held by them and
their predecessors openly, peacefully, and continuously since time immemorial. In February of 1996, they claim
that Ulayao entered their property using force, intimidation, stealth, and violence and erected a house thereon
despite repeated demands to vacate. Spouses then filed a complaint for recovery of possession.
Ulayao contended in his answer that he had been in long, continuous, and adverse possession of the property for
more than 30 years, and had thus acquired the property by prescription, and that the registration of the property was
a sham and was acquired fraudulently. Spouses Soller moved for summary judgment, which the trial court granted.
The trial court based its judgment on the TCT, which is indefeasible and cannot be attacked collaterally, and
therefore, there was no genuine issue of fact.
The Court, however, said the trial court was wrong, and summary judgment was not proper. Summary judgments
are proper when the court finds that the answer filed by the defendant does not tender a genuine issue as to any
material fact and that one party is entitled to a judgment as a matter of law. In this case, Ulayao raised the defense
of acquisitive prescription. This defense inevitably involves the issue of actual, physical, and material possession,

REM Digests ALS 2014B Justice Gesmundo


which is always a question of fact. The existence of this issue necessitates the presentation of evidence done in a
full-blown trial.
74. ANICETO CALUBAQUIB VS. REPUBLIC OF THE PHILIPPINES, G.R. No. 170658, (June 22,
2011); Summary of Judgment; Rule 35
Due process rights are violated by a motu proprio rendition of a summary judgment.
President Quezon issued proclamation no.80 which declared a parcel of land as a military reservation
site. Respondent filed a case against petitioners alleging that the petitioners unlawfully entered the military
reservation site through strategy and stealth. During the pre-trial conference the basic facts of the case were
undisputed, thus the trial advised the parties to file a motion for summary judgment. Neither party filed the
motion. In fact, respondent expressed on two occasions its objection to a summary judgment. It explained that
summary judgment is improper given the existence of a genuine and vital factual issue, which is the petitioners
claim of ownership over the subject property. It argued that the said issue can only be resolved by trying the case
on the merits. The crux of the case is the propriety of rendering a summary judgment. Summary judgments are
proper when, upon motion of the plaintiff or the defendant, the court finds that the answer filed by the
defendant does not tender a genuine issue as to any material fact and that one party is entitled to a judgment
as a matter of law. Relief by summary judgment is intended to expedite or promptly dispose of cases where the
facts appear undisputed and certain from the pleadings, depositions, admissions and affidavits. Where the facts
pleaded by the parties are disputed or contested, proceedings for a summary judgment cannot take the place
of a trial. In the case at bar, the trial court proceeded to render summary judgment with neither of the parties filing
a motion therefor. In fact, the respondent itself filed an opposition when the trial court directed it to file the motion
for summary judgment. Respondent insisted that the case involved a genuine issue of fact. Under these
circumstances, it was improper for the trial court to have persisted in rendering summary judgment. Considering
that the remedy of summary judgment is in derogation of a party's right to a plenary trial of his case, the trial court
cannot railroad the parties rights over their objections.
107. GUBAT V. NATIONAL POWER CORPORATION (FEBRUARY 26, 2010) Rule 35
For a full blown trial to be dispensed with, the party who moves for summary judgment has the burden of
demonstrating clearly the absence of genuine issues of fact, or that the issue posed is patently insubstantial as
to constitute a genuine issue.
Petitioner filed a Motion for Partial Summary Judgment on his attorneys fees. He claimed that his clients and
the respondent NPC deliberately did not inform him about the execution of the compromise agreement, and that
said parties connived with each other in entering into the compromise agreement in order to unjustly deprive him of
his attorneys fees. Respondent NPC opposed the motion for partial summary of judgment. It alleged that a client
may compromise a suit without the intervention of the lawyer and that petitioners claim for attorneys fees should
be made against the plaintiffs. NPC likewise claimed that it settled the case in good faith and that plaintiffs were
paid in full satisfaction of their claims which included attorneys fees.
A summary judgment is allowed only if, after hearing, the court finds that except as to the amount of
damages, the pleadings, affidavits, depositions and admissions show no genuine issue as to any material fact
and that the movant is entitled to a judgment as a matter of law. The purpose of a summary judgment is to
avoid drawn out litigations and useless delays because the facts appear undisputed to the mind of the
court. Such judgment is generally based on the facts proven summarily by affidavits, depositions, pleadings,
or admissions of the parties. For a full-blown trial to be dispensed with, the party who moves for summary
judgment has the burden of demonstrating clearly the absence of genuine issues of fact, or that the issue
posed is patently insubstantial as to constitute a genuine issue. Genuine issue means an issue of fact which
calls for the presentation of evidence as distinguished from an issue which is fictitious or
contrived. Petitioners resort to summary judgment is not proper. He is not entitiled to an immediate relief
as a matter of law for the existence of bad faith is a genuine issue of fact to be tried.
112. ELAND PHILIPPINES, INC. vs. GARCIA, FEBRUARY 17, 2010, RULE 35
Due process, a constitutional precept, does not always and in all situations a trial-type proceedingits essence is
found in the reasonable opportunity to be heard and submit ones evidence in support of his defense.Petitioner
contended that the ten-day notice rule in Section 3 Rule 35 was violated, because the copy of the motion for
REM Digests ALS 2014B Justice Gesmundo
summary judgment was served only on August 20, 1999 or on the same day it was set for hearing. It also added that
even if the petitioner received a copy of the motion only on August 20, 1999, there was no hearing conducted on
that date because the trial court issued an order giving petitioner 10 days within which to file its comment or
opposition. The above specific contention, however, is misguided. The CA was correct in its observation that there
was substantial compliance with due process. The CA ruled, as the records show, that the ten-day notice rule was
substantially complied with because when the respondents filed the motion for summary judgment on August 9,
1999, they furnished petitioner with a copy thereof on the same day as shown in the registry receipt and that the
motion was set for hearing on August 20, 1999, or 10 days from the date of the filing thereof. Due process, a
constitutional precept, does not, therefore, always and in all situations a trial-type proceeding. The essence of due
process is found in the reasonable opportunity to be heard and submit ones evidence in support of his defense.
What the law prohibits is not merely the absence of previous notice, but the absence thereof and the lack of
opportunity to be heard.
Summary Judgments; Any action can be the subject of a summary judgment with the sole exception of
actions for annulment of marriage or declaration of its nullity or for legal separation.Petitioner further
argues that summary judgment is not proper in an action for quieting of title. This particular argument, however, is
misplaced. This Court has already ruled that any action can be the subject of a summary judgment with the sole
exception of actions for annulment of marriage or declaration of its nullity or for legal separation.
The non-existence of a genuine issue is the determining factor in granting a motion for summary judgment,
and the movant has the burden of proving such nonexistence. The SC held that the grant of summary
judgment was not proper. A summary judgment is permitted only if there is no genuine issue as to any material fact
and a moving party is entitled to a judgment as a matter of law. A summary judgment is proper if, while the
pleadings on their face appear to raise issues, the affidavits, depositions, and admissions presented by the moving
party show that such issues are not genuine. It must be remembered that the non-existence of a genuine issue is the
determining factor in granting a motion for summary judgment, and the movant has the burden of proving such
nonexistence. The trial court found no genuine issue as to any material fact that would necessitate conducting a full-
blown trial. However, a careful study of the case shows otherwise.
When the facts as pleaded by the parties are disputed or contested, proceedings for summary judgment
cannot take the place of trial; A genuine issue is an issue of fact that requires the presentation of evidence
as distinguished from a sham, fictitious, contrived or false claim.The facts pleaded by the respondents in their
motion for summary judgment have been duly disputed and contested by petitioner, raising genuine issues that must
be resolved only after a full-blown trial. When the facts as pleaded by the parties are disputed or contested,
proceedings for summary judgment cannot take the place of trial. In the present case, the petitioner was able to
point out the genuine issues. A genuine issue is an issue of fact that requires the presentation of evidence as
distinguished from a sham, fictitious, contrived or false claim.
The fact that the one party seeks to nullify the original certificate of title issued to the other party on the
claim that the former was in possession of the same land for a number of years, is a clear indicium that a
genuine issue of a material fact exists.By granting the summary judgment, the trial court has in effect annulled
its former ruling based on a claim of possession and ownership of the same land for more than thirty years without
the benefit of a full-blown trial. The fact that the respondents seek to nullify the original certificate of title issued to
the petitioner on the claim that the former were in possession of the same land for a number of years, is already a
clear indicium that a genuine issue of a material fact exists. This, together with the failure of the respondents to
show that there were no genuine issues involved, should have been enough for the trial court to give the motion for
summary judgment, filed by respondents, scant consideration. Trial courts have limited authority to render
summary judgments and may do so only when there is clearly no genuine issue as to any material fact.

REM Digests ALS 2014B Justice Gesmundo


RULE 36

11 and 12. COMMISSIONER OF INTERNAL REVENUE v. FORTUNE TOBACCO CORPORATION


(2013); FORTUNE TOBACCO CORPORATION v. COMMISSIONER OF INTERNAL REVENUE (2013)
"the object of a judgment nunc pro tunc is not the rendering of a new judgment and the ascertainment and
determination of new rights, but is one placing in proper form on the record, that has been previously
rendered, to make it speak the truth (JUDGMENT)"
Fortune Tobacco Corporation (FTC) claimed tax refund against CIR for the overpaid excise taxes. Since the clam
was not acted upon, FTC filed a Petition for Review before the CTA, the CTA in Division ordered CIR to refund
FTC. A Motion for Reconsideration was filed by CIR. FTC filed another Petition for Review, questioning the
validity of R.R. No. 17-99 and praying for the refund. CTA in Division declared R.R. No. 17-99 invalid and
ordered CIR to refund FTC. CIR filed a Petition for Review with CA questioning CTAs decision. CA issued a
Decision denying CIRs Petition and affirming FTCs refund claims. CIR then filed a Petition for Review on
Certiorari with the SC. SC affirmed CTA and CAs decision. FTC filed a motion for execution praying for the
issuance of writ of execution of the Decision. It was granted. A motion for the issuance of an additional writ of
execution was filed by FTC, but this time it was denied.
After a scrutiny of the body of the July 21, 2008 Decision, the Court finds it necessary to render a judgment
nunc pro tunc and address an error in the fallo of said decision. The office of a judgment nunc pro tunc is to
record some act of the court done at a former time which was not then carried into the record, and the
power of a court to make such entries is restricted to placing upon the record evidence of judicial action
which has actually been taken. The object of a judgment nunc pro tunc is not the rendering of a new
judgment and the ascertainment and determination of new rights, but is one placing in proper form on the
record, that has been previously rendered, to make it speak the truth, so as to make it show what the judicial
action really was, not to correct judicial errors, such as to render a judgment which the court ought to have
rendered, in place of the one it did erroneously render, not to supply non-action by the court, however
erroneous the judgment may have been. The Court would thus have the record reflect the deliberations and
discussions had on the issue. In this particular case it is a correction of a clerical, not a judicial error. The
body of the decision in question is clear proof that the fallo must be corrected, to properly convey the ruling
of this Court.

RULE 37

9. REPUBLIC OF THE PHILIPPINES, REPRESENTED BY THE DPWH, v. BANK OF THE


PHILIPPINE ISLANDS (2013)
"the 15-day period to file a motion for new trial will start to run from receipt of judgment or final order
(MOTION FOR NEW TRIAL)"
DPWH filed a case for expropriation against portions of properties of BPI and Villanueva. No objections were
made so a Board of Commissioners was instituted to determine the just compensation. The Decision set the fair
market value at P40,000.00 per square meter. BPI filed a Motion for Partial New Trial on 16 December to
determine the just compensation. BPI claimed that its motion was timely filed since it received a copy of the
Decision on 01 December.
Pursuant to Section 1, Rule 37 of the Rules of Civil Procedure, the period within which an aggrieved party
may move the trial court to set aside the judgment or final order and file a motion for new trial is within the
period to file an appeal, which is fifteen (15) days from receipt of the judgment or final order. A judgment,
final order or resolution shall be served upon a party either personally or through registered mail.
Moreover, Section 9 of Rule 13 of the Rules of Court states that judgments, final orders or resolutions shall
be served either personally or by registered mail. Section 13 of the same Rule provides that proof of personal

REM Digests ALS 2014B Justice Gesmundo


service shall consist of a written admission of the party served, or the official return of the server, or the
affidavit of the party serving, containing a full statement of the date, place and manner of service.
A careful review of the record shows the absence of any proof that the Decision of 25 November was served
upon BPI. Hence, the Court of Appeals correctly held that absent any proof of service to BPI of the Decision,
the period of 15 days within which to file its motion for partial new trial did not begin to run against BPI.
However, BPIs admission that it received a copy of the Decision on 01 December is binding on it, and was
correctly considered by the Court of Appeals as the reckoning date to count the 15-day period.

RULE 38

65. RP v. SUNVAR (2012)


[Sorry mahaba yung gist of doctrine. Important talaga yung mga nilagay ko.]
Rule 38 and 65
Rule 38 - Petition for Relief of Judgment [obiter]; Rules on Summary Procedure, prohibits relief under summary
procedure; exceptions
Rule 65 - SCA, Unlawful Detainer
(1) MTC to RTC: When Section 19 of the Revised Rule on Summary Procedure bars a petition for relief
from judgment, or a petition for certiorari, mandamus, or prohibition against any interlocutory order issued
by the court, it has in mind no other than Section 1, Rule 38 regarding petitions for relief from judgment,
and Rule 65 regarding petitions for certiorari, mandamus, or prohibition, of the Rules of Court, respectively.
(2a) RTC to CA: No appeal may be taken from an order of the RTC dismissing an action without prejudice,
but the aggrieved party may file a certiorari petition under Rule 65. Nevertheless, the Rules do not prohibit
any of the parties from filing a Rule 45 Petition with this Court, in case only questions of law are raised or
involved."
(2b) three modes of appeal from decisions of the RTC, to wit: (1) by ordinary appeal or appeal by writ of
error under Rule 41, whereby judgment was rendered in a civil or criminal action by the RTC in the exercise
of its original jurisdiction; (2) by a petition for review under Rule 42, whereby judgment was rendered by
the RTC in the exercise of its appellate jurisdiction; and (3) by a petition for review on certiorari before the
Supreme Court under Rule 45. The first mode of appeal is taken to the [Court of Appeals] on questions of
fact or mixed questions of fact and law. The second mode of appeal is brought to the CA on questions of fact,
of law, or mixed questions of fact and law. The third mode of appeal is elevated to the Supreme Court only
on questions of law. There is a question of law when the issue does not call for an examination of the
probative value of the evidence presented or of the truth or falsehood of the facts being admitted, and the
doubt concerns the correct application of law and jurisprudence on the matter.[50] The resolution of the
issue must rest solely on what the law provides on the given set of circumstances.
(3) General Rule: Under the Rules on Summary Procedure, a certiorari petition under Rule 65 against an
interlocutory order issued by the court in a summary proceeding is a prohibited pleading.
Two Exceptional Cases: Bayog v. Natino and Go v. CA.
In Bayog, an illiterate farmer, received the Summons from the MCTC to file his answer within 10 days, he
was stricken with pulmonary tuberculosis and was able to consult a lawyer in San Jose, Antique only after
the reglementary period.
In Go v. CA, the preliminary conference in the subject ejectment suit was held in abeyance [a patently
erroneous order] by the Municipal Trial Court in Cities (MTCC) of Iloilo City until after the case for
specific performance involving the same parties shall have been finally decided by the RTC; in situations
wherein a summary proceeding is suspended indefinitely, a petition for certiorari alleging grave abuse of
discretion may be allowed.

REM Digests ALS 2014B Justice Gesmundo


(4) The one-year period for filing a complaint for unlawful detainer is reckoned from the date of the last
demand, the reason being that the lessor has the right to waive his right of action based on previous demands
and let the lessee remain meanwhile in the premises.
The Republic of the Philippines (RP) and National Power Corporation (NPC) leased land to Philippine
Development Alternatives Foundation (PDAF). PDAF subleased to Sunvar. The lease agreement to PDAF expired.
RP and NPC was not willing to renew the lease, and send a demand to vacate to Sunvar on July 3, 1995. Sunvar did
not vacate. RP and NPC sent another demand on October 24, 1996. Sunvar did not vacate. RP and NPC filed a case
for unlawful detainer before the MeTC on Feb. 25, 1997. Sunvar argued in a motion to dismiss that the case filed
should not be unlawful detainer, but be one for accion publiciana which fell under the jurisdiction of the RTC. The
motion to dismiss and subsequent motion for reconsideration was denied by the MeTC. Sunvar filed its answer in
the MeTC. Despite the answer, Sunvar filed a Rule 65 Petition for Certiorari with the RTC of Makati City to assail
the denial by the MeTC of respondents Motion to Dismiss. In answer to the Rule 65 Petition of Sunvar, RP and
NPC placed in issue the jurisdiction of the RTC and reasoned that the Rules on Summary Procedure expressly
prohibited the filing of a petition for certiorari against the interlocutory orders of the MeTC. Hence, they prayed for
the outright dismissal of the certiorari Petition of respondent Sunvar. The RTC denied RP and NPCs motion to
dismiss. The RTC granted the Rule 65 Petition and directed the MeTC to dismiss the Complaint for unlawful
detainer for lack of jurisdiction. Sunvar filed a petition for review on certiorari to the Supreme Court under Rule
45.
MTC to RTC: Rule 65 petition on the MTD is a prohibited pleading in a case which applies summary
procedure.
RTC to SC: Rule 45 petition for review on certiorari before the Supreme Court is proper. The question of
whether or not the Rule 65 petition before the RTC is proper is a question of law.
Unlawful Detainer v. accion publiciana: The MTC has jurisdiction. The one-year period for filing a
complaint for unlawful detainer is reckoned from the date of the last demand, in this case October 24, 1996,
the reason being that the lessor has the right to waive his right of action based on previous demands and let
the lessee remain meanwhile in the premises.
190. Sps. Mesina v. Meer (July 2, 2002) (petition for relief)
Meer owns land in Manila. He discovered that his TCT was cancelled and one was issued in the name of the
Spouses Bunquin. Apparently, the Bunquins acquired the property by virtue of a deed of sale purportedly executed
by Meer in their favor. Meer then sought the cancellation of the Bunquins TCT. While such case was pending, a
new TCT was issued in favor of the spouses Mesina. So Meer impleaded the Mesinas as additional party
defendants. The MeTC ruled that while the alleged sale between Meer and Bunquin was fraudulent, the Mesinas
were buyers in good faith. RTC reversed. CA upheld. After the reglementary period for appeal lapsed, petitioners
filed a Petition for Relief from Judgment with the CA. The CA denied, on the ground that a petition for relief may
not be availed of from a judgment of the CA in the exercise of its appellate jurisdiction.
Relief from judgment is an equitable remedy and is allowed only under exceptional circumstances and only
if fraud, accident, mistake or excusable negligence is present. Where the defendant has other available or
adequate remedy such as a motion for new trial or appeal from the adverse decision, he cannot avail himself
of this remedy. The Rules provide that a petition for relief should be filed with the same court which
rendered the decision. As it stands, neither the Rules of Court nor the Revised Internal Rules of the Court of
Appeals allow the remedy of petition for relief in the CA. While the Court may, in the interest of justice, construe
the Rules with liberality, nothing in this case convinces the SC to do so. They should have raised the issue of fraud
at the first instance. It is a settled rule that relief will not be granted to a party who seeks to be relieved from
the effects of the judgment when the loss of the remedy at law was due to his own negligence, or a mistaken
mode of procedure; otherwise the petition for relief will be tantamount to reviving the right of appeal which
has already been lost either because of inexcusable negligence or due to mistaken mode of procedure by
counsel.

REM Digests ALS 2014B Justice Gesmundo


RULE 39

6. OFFICE OF COURT ADMINISTRATOR V. MACUSI (2013)


The raison d etre behind the requirement of periodic reports under Rule 39, Section 14 of the Rules of Court
is to update the court on the status of the execution and to take necessary steps to ensure the speedy
execution of decisions. Neither the Rules nor jurisprudence recognizes any exception from the periodic filing
of reports by sheriffs.
Facts: Macusi is the sheriff of MTCC Tabuk, Kalinga. An administrative case was filed against him after a plaintiff
in a civil case filed before said court wrote a letter to judge Dalanao (judge of said court) inquiring about the status
of the writ plaintiff applied for. Judge Dalanao thereafter indorsed a letter to the Office of the Court Administrator
(OCA) for appropriate action. The OCA thereafter assigned Atty. Andomang to comment and make a report. It was
thereafter found by Atty, Andomang that indeed in several cases Macusi never filed a report regarding the status of
the writs given to him for execution. Some cases he did. Also Judge Dalanao stated that Macusi never also filed any
report as regards such writs whether served or unserved as well as his estimate of expenses for the judges approval.
Macusi refuted such accusations saying that he was only sheriff from 2005 while some of the writs in question were
from as early as 1997. According to him the rule states that the Sheriff must act with celerity and promptness when
they are handed the Writs of Execution; yet, the rule also states that when party litigants, in whose favor the Writs,
have been issued, frustrate the efforts of the Sheriffs to implement those Writs, the latter are relieved from such
duty and incur no administrative liability therefor. Macusi additionally wrote that he did not report regularly
despite the presence of the rules since he relied on the dictates of practicality so as not to waste supplies. Rules,
accordingly are there to guide but they are not absolute, what matters is what one accomplishes.
Held: Macusi exercised excessive discretion in the execution of the writs and in the filing of reports thereon.
He seemed to have entirely overlooked that the nature of a sheriffs duty in the execution of a writ issued by
a court is purely ministerial. As such, a sheriff has the duty to perform faithfully and accurately what is
incumbent upon him. Conversely, he exercises no discretion as to the manner of executing a final judgment.
Any method of execution falling short of the requirement of the law deserves reproach and should not be
countenanced.
18. LIGAYA ESGUERRA V. HOLCIM PHILIPPINES, INC. (2013)
Sec 36 and 37 of Rule 39 of the ROC are resorted to only when the judgment remains unsatisfied and there is
a need for judgment obligor to appear and be examined concerning the property and income for their
application to the unsatisfied amount in the judgment.
This case is an offshoot of a prior action to annul the Free Patent which was under the name of de Guzman, filed by
Esquerra. The land in question was at that time, being offered for sale to HOLCIM. The RTC dismissed the
complaint, but the CA reversed this decision on appeal. The SC affirmed in toto the decision of the CA and after the
case attained finality, it was remanded to the RTC for execution. Thereafter the heirs of Esguerra filed an Omnibus
Motion with RTC, manifesting that the latter decision has yet to be executed and thus prayed for its execution.
Without the participation of HOLCIM, the RTC approved the claim of the petitioners as to the proper amount of
royalties due to them from HOLCIM and also issued for the issuance of an Alias Writ of Execution. After the
issuance of the writ, HOLCIM filed an MR alleging that it did not owe any amount of royalty to the petitioners for
the extracted limestone from the subject land. This MR was denied by the RTC, stating that to reopen the case
despite the fact that the trial court has no more power to do so since the execution of the Courts decision in the first
case is now a matter of right on the petitioners part. HOLCIM filed a petition for certiorari before the CA, which
found the case meritous. Based form the records of the case, the petitioners asked for the Examination of de
Guzman ad HOLCIM under Section 36 and 37 of Rule 39 of the ROC. Section 36 (Examination of judgment
obligor when judgment unsatisfied) and 37 (Examination of obligor of judgment obligor) of Rule 39 of the
ROC is not the proper remedy of the Heirs of Esguerra. The decision sought to be implemented in this case
simply calls on HOLCIM to make an accounting of the royalty paid to de Guzman.

REM Digests ALS 2014B Justice Gesmundo


47. BANK OF THE PHILIPPINE ISLANDS v. CARLITO LEE (2012)
For Rules 3, 19, and 39. A party who is served with garnishment notice and who acknowledged possessing
the garnished property becomes a VIRTUAL PARTY or FORCED INTERVENOR to the case even if he
is not impleaded; the surviving corporation in a merger is also a VIRTUAL party; No need for institution
of separate action under Rule 39 Section 43 if the garnishee (bank) does not deny the existence of the fund or
does not claim interest adverse to the debtor (defendant).
Carlito Lee filed an action for collection of sum of money against a swindler couple. The swindler couple owned a
bank account with CITYTRUST BANK. Carlito moved for the garnishment of the couples bank account with
CITYTRUST. The court eventually decided in favor of Carlito, but by this time, Citytrust had a MERGER with
BPI. Now, Carlito wanted to execute on the garnished funds. He moved for execution against BPI (not anymore
Citytrust). BPI says that it is a stranger to the case.
Held: It is a settled rule that upon service of the writ of garnishment, the garnishee becomes a virtual party
or forced intervenor to the case and the trial court thereby acquires jurisdiction to bind the garnishee to
comply with its orders and processes.
Citytrust, therefore, upon service of the notice of garnishment and its acknowledgment that it possessed the
bank accounts, became a virtual party to or a forced intervenor in the civil case against the swindler
couple. As such, Citytrust became bound by the orders and processes issued by the trial court despite not
having been properly impleaded therein. Then, by virtue of Citytrusts merger with BPI, BPI, as the
surviving corporation, effectively also became the garnishee, who is again, the virtual party to the civil
case.
Also, BPI could not ask for the institution of a separate action against a garnishee under Rule 39 Section 43
because this provision contemplates a situation where the garnishee (third person) claims an interest in the
property adverse to him (judgment debtor) or denies the debt. Neither of these situations exists in this case.
The garnishee Citytrust / BPI does not claim any interest in the deposit accounts of the defendants, nor does it deny
the existence of the deposit accounts.
BPIs liability for the garnished deposits of the swindler couple has been clearly established.
50. MADRIAGA V. CHINA BANKING CORPORATION, G.R. NO. 192377 (2012)
A third party is one who holds the property by title or right adverse to that of the mortgagor, such as that
of a co-owner, tenant or usufructuary, and not one who is a mere transferee or successor of such
mortgagor.
Spouses Trajano owned 2 residential properties located in Bulacan. They sold these properties to Cesar Madriaga
Sr. in favor of whom they executed a deed of absolute sale. By virtue of said deed, Madriaga Sr. took possession of
the properties. However, the spouses failed to deliver the titles and were sued by Madriaga, Sr. for specific
performance. The parties executed a compromise agreement which the spouses failed to comply with. As a result,
Madriaga, Sr. moved for the issuance of a writ of execution against the spouses. The 2 properties were made
subject of an auction where Madriaga, Sr. was declared winning bidder. After the lapse of the 1-year redemption
period, he was issued a final deed of sale and the TCTs of the 2 properties were cancelled and replaced by new ones
in his name. It appears, however, that the 2 properties were subsequently mortgaged in favor of China Bank using
the already cancelled TCTs. This mortgage was also foreclosed and the 2 lots were again auctioned, and China
Bank was declared highest bidder. A new set of TCTs were issued in China Banks name, to replace, for the second
time, the cancelled TCTs. China Bank filed an ex parte petition for writ of possession, impleading the spouses and
all persons claiming rights under their name. The writ was granted, a copy of which was served upon Madriaga, Sr.
who opposed. Madriaga Sr. insists that he was deprived of due process since he was not served notice of China
Banks petition for writ of possession, and was only notified of the issuance of China Banks titles when the writ of
possession itself was already served upon him. Madriaga Jr., successor, filed a petition for certiorari before the CA
alleging grave abuse of discretion. CA denied. Hence, this petition before the SC.
Madriaga, Sr. was not deprived of due process. After consolidation of ownership and issuance of a new TCT
in an extrajudicial foreclosure, the general rule is that a purchaser becomes entitled to possession of the
property as a matter of right. Thus, the issuance of a writ of possession by the RTC becomes a ministerial
function. Notice need not be served upon persons interested in the property. The ex parte nature of the
REM Digests ALS 2014B Justice Gesmundo
proceeding does not deny due process to the petitioners because the issuance of the writ not bar a separate
case for annulment of mortgage and foreclosure sale.
Section 33, Rule 39 of the Rules of Court however, presents an exception. The possession of the
extrajudicially foreclosed property shall be withheld from the purchaser if a third-party is actually holding
the same adversely to the mortgagor/debtor. In such cases, the issuance of the writ of possession ceases to be
ministerial, and cannot be done ex parte. A third party is one who holds the property by adverse title or
right, such as that of a co-owner, tenant or usufructuary. Such persons possess the property in their own
right, and they are not mere successors or transferees of the right of possession of another. Madriaga Sr.
gained possession of the properties by virtue of the sale between him and the spouses Trajano, the
mortgagors of China Bank. Hence, Madriaga Sr. is not a third-party whose possession of the disputed
properties is adverse to that of the spouses. The exception does not apply. Petition denied.
71. Dela Merced vs GSIS (Nov. 23, 2011. Lis Pendens)
Once a notice of lis pendens has been duly registered, any cancellation or issuance of the title of the land
involved as well as any subsequent transaction affecting the same, would have to be subject to the outcome of
the litigation A transferee pendente lite of registered land, whose title bears a notice of a pending litigation
involving his transferors title to the said land, is bound by the outcome of the litigation, whether it be for or
against his transferor. Given this principle, the modification of the final decision against the transferor in
order to include the transferee pendente lite does not violate the doctrine of immutability of final judgments.
His inclusion does not add to or change the judgment; it is only a legal consequence of the established
doctrine that a final judgment binds the privy of a litigating party.
The case involves five registered parcels of land located within Pasig City originally owned by the Zulueta
Spouses. These lands were mortgaged and eventually foreclosed. Upon learning of the foreclosure, the Zulueta
spouses predecessor Dela Merced filed a complaint for nullity of the GSIS foreclosure. Dela Merced also caused
the annotation of lis pendens on the GSISs TCT. On July 29, 1985 and August 24, 1998, GSISs TCT was
cancelled with respect to Lots 7 and 8 of Block 2 and new individual titles were issued to Victorino and Dimaguila.
Both titles had the notice of lis pendens which was carried over from GSISs TCT.
Ineluctably, both Victorino and Dimaguila had notice of the litigation involving GSISs ownership over the
subject properties, and were bound by the outcome of the litigation. When a transferee pendente lite takes
property with notice of lis pendens, such transferee undertakes to respect the outcome of the litigation. An
order to cancel the transferors title may be enforced against his transferee, whose title is expressly subject to
the outcome of the litigation by the fact of the annotation of lis pendens. The existence of these entries on
Dimaguilas and Victorinos titles bars any defense of good faith against petitioners and effectively makes
Dimaguila and Victorino mere privies of GSIS and subject to whatever rights GSIS might have in the
subject properties, which as it turns out is none at all. As explained by this Court in another case, the title
obtained by the transferee pendente lite affords him no special protection; he cannot invoke the rights of a
purchaser in good faith and cannot acquire better rights than those of his predecessor-in-interest. The Court
explained that an action is binding on the privies of the litigants even if such privies are not literally parties
to the action. Their inclusion in the writ of execution does not vary or exceed the terms of the judgment. In
the same way, the inclusion of the derivative titles in the writ of execution will not alter the Decision
ordering the cancellation of GSISs title.
73. Golden Sun Finance Corp v. Albano (July 27, 2011. Rule 39, Execution of Judgments)
In determining properties to be levied upon, the Rules require the sheriff to levy only on those properties of
the judgment debtor which are not otherwise exempt from execution. For purposes of the levy, a
property is deemed to belong to the judgment debtor if he holds a beneficial interest in such property that he
can sell or otherwise dispose of for value; A mortgaged property may still be levied upon by the sheriff to
satisfy the judgment debtors obligations. The encumbrance, until foreclosed, will not in any way affect the
judgment debtors rights over the property or exempt the property from the levy. A sheriffs duty to execute
a writ is simply ministerial, and he is bound to perform only those tasks stated under the Rules of Court and
no more. Any interest a third party may have on the property levied upon by the sheriff to enforce a
judgment is the third partys responsibility to protect through the remedies provided under Rule 39 of the
Rules of Court.
REM Digests ALS 2014B Justice Gesmundo
Golden Sun Finance filed a complaint for the recovery of a Honda Civic Sedan with the RTC against Reyes. The
subject motor vehicle, registered in the name of Reyes, was encumbered in its favor, as shown in the Certificate of
Registration issued by the Land Transportation Office.The RTC decided in favor of Golden Sun and issued a writ
of replevin. However, the complainant found out that the motor vehicle had already been levied upon by the Sheriff
Albano by virtue of a writ of execution issued on March 27, 2009 by the MeTC for violation of Batas Pambansa
Bilang 22 against Reyes. It was sold at a public auction to the highest bidder. Golden Sun averred that the levy and
sale of the motor vehicle by the Albano was illegal for he levied upon the motor vehicle and proceeded with the
auction sale without looking into the cars Certificate of Registration to determine whether it was encumbered or
not.
According to Sec. 9, Rule 39, (b)Satisfaction by levy.If the judgment obligor cannot pay all or part of the
obligation in cash, certified bank check or other mode of payment acceptable to the judgment obligee, the officer
shall levy upon the properties of the judgment obligor of every kind and nature whatsoever which may be
disposed of for value and not otherwise exempt from execution giving the latter the option to immediately
choose which property or part thereof may be levied upon, sufficient to satisfy the judgment. If the judgment
obligor does not exercise the option, the officer shall first levy on the personal properties, if any, and then on
the real properties if the personal properties are insufficient to answer for the judgment.
It was thus irrelevant for the complainant to argue that had the respondent checked the cars certificate of
registration, the respondent would have been aware of the encumbrance. The encumbrance, until foreclosed,
will not in any way affect the judgment debtors rights over the property or exempt the property from the
levy. Even the pendency of the proceeding for replevin that the complainant instituted would not serve to
prevent the sheriff from levying on the car, since Reyes default and the Golden Suns right to foreclose still
had to be settled in the proceeding.
82.) Narciso Tumibay v Soro; G.R. 152016; (2010) (Rule 39 Sec. 10/47, writ of execution)
Though the decision made no mention of the demolition of the improvements in this case, the writ of execution may
include this in the case of recovery of ownership because Rule 39 Section 10 provides that surrender of possession,
ousting of defeated party are necessary in the execution of the judgement for the delivery or restitution of real
property.
A case for annulment and recovery of ownership with damages was filed by respondent Soro concerning a certain
Cabanatuan land. The RTC ruled in favour of Soro and in its dispositive portion ordered for annulment of TCT
issued to petitioner Narciso; declared that Soro are the sole heirs of the deceased and owner of the land thus
ordering for the reconveyance of the land. After finality, RTC issued a writ of execution and in pursuant to this
respondents Soro also filed a motion to be restored to the possession of subject property and demolish the
improvements thereon. The petitioners argued that this motion should be denied because the decision did not
expressly ordain the demolition of existing improvements and that writ of execution cannot go beyond what is
written in the dispositive portion. The Supreme Court held that even though the writ of execution must conform
strictly to what is stated in the decision, the judgment should not be confined to what appears on the face of the
decision but extends to those necessarily included therein. Declaration of ownership include a declaration of right
to possess if the defeated party shows no right to possess thus anything essential or necessary to assert the right of
the owner must be considered. Based in Rule 39, Sec 10 the removal of the improvements is deemed read into the
decision thus no express order need be stated.
91. CATALINA BALAIS-MABANAG v. REGISTER OF DEEDS OF QUEZON CITY, ET. AL. (2010)
The issue of citizenship of the registered owner of land cannot anymore be raised to forestall the execution of
a final and executory judgment where the objecting party had the opportunity to raise the issue prior to the
finality of the judgment. The time for assailing the capacity of the winning party to acquire the land was
during the trial, not during the execution of a final decision.
The Coronels sold a parcel of land to Alcaraz for 1.2M. Alcaraz gave a downpayment of 50k. It was further agreed
that the Coronels with transfer the title of the land in the name of Alcaraz upon receipt of downpayment. The
Coronels rescinded the sale of the land to Alcaraz and deposited the downpayment, 50K, in the bank in trust for
Alcaraz. They sold the land at higher price of 1.5M to Catalina. Catalina made a downpayment of 300K. Catalina
intervened in a civil case involving the subject parcel of land between Alcaraz and the Coronels. RTC ruled in favor

REM Digests ALS 2014B Justice Gesmundo


of Alcaraz and ordered specific performance of the sale of the land and that the Coronels and the Catalina vacate
the property and deliver possession of the land to Alcaraz. CA affirmed and its decision became final and
executory. RTC issued a writ of execution. Coronels failed to issue a deed of absolute sale in favor of Alcaraz.
Branch Clerk of Court issued it instead.
Hence, this appeal, in which the Catalina submits that the CA erred in sustaining the registration by the Registrar of
Deeds of the deed of absolute sale despite the lack of indication of the citizenship of the buyer of the subject
property; and in sustaining the order of the RTC directing the Branch Clerk of Court to execute the deed of absolute
sale without first requiring the Coronels to execute the deed of absolute sale as required by the decision.
Catalina did not raise any issue against Ramonas qualifications to own land in the Philippines during the trial or, at
the latest, before the finality of the RTC judgment. She was thereby deemed to have waived the objection, pursuant
to Section 1, Rule 9 of the Rules of Court:
Section 1. Defenses and objections not pleaded. Defenses and objections not pleaded either in a motion to
dismiss or in the answer are deemed waived. However, when it appears from the pleadings or the evidence on
record that the court has no jurisdiction over the subject matter, that there is another action pending between the
same parties for the same cause, or that the action is barred by a prior judgment or by statute of limitations, the
court shall dismiss the claim.
In every action, indeed, the parties and their counsel are enjoined to present all available defenses and objections in
order that the matter in issue can finally be laid to rest in an appropriate contest before the court. The rule is a wise
and tested one, borne by necessity. Without the rule, there will be no end to a litigation, because the dissatisfied
litigant may simply raise "new" or additional issues in order to prevent, defeat, or delay the implementation of an
already final and executory judgment. The endlessness of litigation can give rise to added costs for the parties, and
can surely contribute to the unwarranted clogging of court dockets. The prospect of a protracted litigation between
the parties annuls the very rationale of every litigation to attain justice. Verily, there must be an end to litigation.
Catalina cannot now insist that the RTC did not settle the question of the respondents qualifications to own land
due to non-citizenship. It is fundamental that the judgment or final order is, with respect to the matter directly
adjudged or as to any other matter that could have been raised in relation thereto, conclusive between the parties
and their successors in interest by title subsequent to the commencement of the action or special proceeding,
litigating for the same thing and under the same title and in the same capacity. Res judicata.
109. Reforzado vs. Lopez, (February 24, 2010) (Rule 39).
An important requisite for the principle of res judicata is that the judgment must be a final and executory
decision on the merits. A probate court issued a writ of possession asking N to turn over a parcel of land in his
possession, he questioned this via certiorari this is case CA-G.R. No. 33118. He then filed a case to annul the title
of persons who took over possession of his property, this case Civil Case No. 67043. Now there is no res judicata
because the appellate courts ruling in CA-G.R. SP No. 33118 was not a final and executory decision on the
merits to put the present case within the ambit of res judicata. the ruling in CA-G.R. No. 33118, relied upon by the
appellate court in holding that res judicata bars petitioners present complaint for annulment of title and
reconveyance, is not a decision on the merits on the ownership of the property, the appellate court in said case
having merely resolved the propriety of the probate courts issuance of a writ of possession in favor of herein
petitioner. The appellate court in fact declared in CA-G.R. SP No. 33118 that N had the remedy of filing a separate
action for recovery of the property.
Related Doctrines:
The doctrine of res judicata lays down two main rules which may be stated as follows: (1) The judgment or decree
of a court of competent jurisdiction on the merits concludes the litigation between the parties and their privies and
constitutes a bar to a new action or suit involving the same cause of action either before the same or any other
tribunal; and (2) any right, fact, or matter in issue directly adjudicated or necessarily involved in the determination
of an action before a competent court in which a judgment or decree is rendered on the merits is conclusively
settled by the judgment therein and cannot again be litigated between the parties and their privies whether or not the
claims or demands, purposes, or subject matters of the two suits are the same. These two main rules mark the
distinction between the principles governing the two typical cases in which a judgment may operate as evidence. In
speaking of these cases, the first general rule above stated, and which corresponds to the afore-quoted paragraph (b)
REM Digests ALS 2014B Justice Gesmundo
of Section 47, Rule 39 of the Rules of Court, is referred to as bar by former judgment; while the second general
rule, which is embodied in paragraph (c) of the same section and rule, is known as conclusiveness of judgment.
Identity of causes of action does not mean absolute identity, otherwise, a party could easily escape the operation of
res judicata by changing the form of the action or the relief sought.From the earlier-stated allegations gathered
from petitioners complaint subject of the present petition, she is suing respondents for the annulment of the title to
the property issued to them and for the reconveyance of the property to Fr. Balbinos estate. There is thus identity
of parties and subject matter in the two cases. As to identity of causes of action, it is hornbook rule that identity of
causes of action does not mean absolute identity, otherwise, a party could easily escape the operation of res judicata
by changing the form of the action or the relief sought. CA-G.R. SP No. 33118 which emanated from SP. Proc. No.
B-894 involved estate proceedings, while Civil Case No. 67043 subject of the present petition is for Annulment of
Title, Reconveyance, Recovery of Possession and Ownership and Damages. These two cases differ in the form of
action, but they raise the same issueownership and possession of the same property, and they invoke the same
relieffor Fr. Balbinos estate to be declared the owner of the property and for it reconveyed to his estate, and for
the TCT in the name of herein respondents to be annulled. And the evidence required to substantiate the respective
claims of the parties is substantially the same.
117. CUA LAI CHU vs LAQUI
The right to possession of a purchaser at an extrajudicial foreclosure sale is not affected by a pending case
questioning the validity of the foreclosure proceeding
TOPIC: Rule 39, Writ of Possession
Cua Lai Chu obtained a loan from Philippine Bank of Communication (PBC) secured by a Real Estate Mortgage.
Upon default, there was an extrajudicial foreclosure sale where a certificate of sale was issued in favor of PBC.
After the lapse of the one-year redemption period, PBC filed an affidavit of consolidation to consolidate its
ownership and title to the foreclosed property. A TCT was issued in the name of PBC. PBC filed a writ of
possession of the foreclosed property, which was issued. The RTC declared Chua Lai Chu in default and allowed
PBC to present evidence ex parte. Chua Lai Chu claims he was deprived of due process bec he did not have a
chance to file his opposition, and that there was a violation of the rule on forum shopping since, at the time the
application for the issuance of a writ of possession was filed, there was a pending case questioning the validity of
the extrajudicial foreclosure sale. The CA held that a proceeding for the issuance of a writ of possession is ex parte
in nature; thus, there is no violation of due process. The purchaser at an extrajudicial foreclosure sale has a
right to the possession of the property even during the one-year redemption period provided the purchaser
files an indemnity bond. After the lapse of the said period with no redemption having been made, that right
becomes absolute and may be demanded by the purchaser even without posting of a bond. PBC applied for
the issuance of a writ of possession, presenting a new TCT issued in its name. Thus, PBCs right to possession was
founded on ownership. As purchaser of the property at the foreclosure sale, in whose name title over the
property was already issued, the right of PBC over the property had become absolute, vesting in it the
corollary right of possession. The right to possession of a purchaser at an extrajudicial foreclosure sale is not
affected by a pending case questioning the validity of the foreclosure proceeding. Even pending such latter
proceeding, the purchaser (PBC) at a foreclosure sale is entitled to possession of the foreclosed property.
121.) FLOR MARTINEZ VS. ERNESTO GARCIA, G.R. NO. 166536 (2010)
Rule 39, Sec. 12: Effect of levy on execution as to third persons-The levy on execution shall create a lien in
favor of the judgment obligee over the right, title and interest of the judgment obligor in such property at the
time of the levy, subject to liens and encumbrances then existing.
Brua was the owner of a land in Mandaluyong. Property mortaged to the GSISinscribed in TCT on Feb. 1980.
Brua obtained a loan from Garcia and to secure payment of loan, the former mortgaged his property again to
Garcia. Since the property was under the possession of GSIS, Garcia could not register the Deed of Real Estate
Mortgage so he executed an Affidavit of Adverse Claiminscribed in June 1980. Garcia paid to GSIS Bruas loan
in 1991, thus property was released to him. However, it seemed that the previous title had some annotations at the
back. One of which was a Notice of Levy on Execution in favor of Flor Martinez inscribed on 1988. It appeared
that annotations were due to the case for collection of sum of money filed by Martinez. Court previously ruled in
favor of Martinez, Public Action conducted where property was awarded to Martinez as sole bidder. Martinez now

REM Digests ALS 2014B Justice Gesmundo


claiming she has rights to the property. Notice of levy could not prevail over the existing adverse claim of
respondent Garcia inscribed on the title. Martinezs claim over the subject property must yield to the earlier
encumbrance registered by Garcia. Martinez was not a buyer in Good faith at the time of public auction.
126. ELIGIO P. MALLARI VS. GSIS ET. AL. (2010) [RULE 39]
The mortgagor or his successor-in-interest must redeem the foreclosed property within one year from the
registration of the sale with the Register of Deeds in order to avoid the title from consolidating in the
purchaser.
Petitioner obtained two loans from the GSIS secured by two real estate mortgages. A series of dilatory tactics
ensues which ended when GSIS finally foreclosed the premises. Petitioner failed to redeem the property within 1
year from the date of registration of the sale hence the ownership was divested from him. He avers that the
reckoning date for foreclosure is from the date of the sale. Section 28 of Rule 39 states that The judgment
obligor, or redemptioner, may redeem the property from the purchaser, at any time within one (1) year from the
date of the registration of the certificate of saleBy failing to redeem thuswise, the mortgagor loses all
interest over the foreclosed property.The purchaser, who has a right to possession that extends beyond the
expiration of the redemption period, becomes the absolute owner of the property when no redemption is
made, that it is no longer necessary for the purchaser to file the bond required under Section 7 of Act No.
3135, as amended, considering that the possession of the land becomes his absolute right as the lands
confirmed owner. The consolidation of ownership in the purchasers name and the issuance to him of a new
TCT then entitles him to demand possession of the property at any time, and the issuance of a writ of
possession to him becomes a matter of right upon the consolidation of title in his name.
127. ROSARIO T. FLORENDO ET. AL. VS. PARAMOUNT INSURANCE CORP (2010) [RULE 39]
Execution pending appeal is the exception to the general rule must be strictly construed and firmly
grounded on the existence of good reasons. "Good reasons," it has been held, consist of compelling
circumstances that justify immediate execution lest the judgment becomes illusory.
Petitioners bought five parcels of land in Cavite. While they paid their real estate taxes religiously, they failed to
cause the transfer of titles to the lands in their name. 18 years later, the municipality refused to accept tax payments
due to the attachment of the lots in favour of respondent and judgement had been rendered in the latters favor in an
earlier civil case. Petitioners filed a case and won in the RTC; while the respondents moved for appeal, petitioners
filed a motion for execution pending appeal citing good reasons such as Rosario Florendos old age. RTC
approved execution pending appeal. The Florendos point out that Rosario is already in her old age and suffers
from life threatening ailments. But the trial court has allowed execution pending appeal for all of the
Florendos, not just for Rosario whose share in the subject lands had not been established. No claim is made
that the rest of the Florendos are old and ailing. Consequently, the execution pending appeal was indiscreet
and too sweeping. The RTC also justified the execution pending appeal on respondent Paramounts delaying
tactics and the possibility that it could become insolvent during the appeal. But these justifications are
purely speculative. Execution pending appeal rejected.
142. METROPOLITAN BANK & TRUST CO. v. LAMB CONSTRUCTION CONSORTIUM
CORPORATION (2009) [Rule 39 - Execution]
1. Issuance of writ of possession after foreclosure sale is generally ministerial and in all cases a) when the
period to redeem has expired or b) when ownership has been consolidated in favor of mortgagee. Exception
is only because of equity. 2. When the mortgagee-purchaser in a foreclosure sale fails to return the surplus,
the remedy of a mortgagor lies in a separate civil action for collection of a sum of money.
This case is a petition for the issuance of a writ of possession. As a general rule, the issuance of a writ of
possession is ministerial. The only exception is when equity calls for the withholding of the writ, as when the
mortgagee failed to deliver the surplus of the proceeds of the foreclosure sale equivalent to 40% of the debt.
Regardless of equity considerations however, the issuance of the writ of possession is always ministerial in any
of two cases:
1. when the period to redeem has already expired or

REM Digests ALS 2014B Justice Gesmundo


2. when ownership over the property has already been consolidated in favor of the mortgagee-
purchaser.
In other words, even if the mortgagee-purchaser fails to return the surplus, a writ of possession must still be issued,
when any of these two cases are present.
The failure of the mortgagee to deliver the surplus proceeds does not affect the validity of the foreclosure
sale. It only gives rise to a cause of action for the mortgagee to file an action to collect the surplus proceeds.
If the mortgagee is retaining more of the proceeds of the sale than he is entitled to, this fact alone will not
affect the validity of the sale but simply gives the mortgagor a cause of action to recover such surplus.
Lastly, an action to collect the surplus proceeds is improper where there is a pending action for the
nullification of the fore closure proceedings. It is only after the dismissal of complaint for annulment or when the
foreclosure sale is declared valid that the mortgagor may recover the surplus in an action specifically brought for
that purpose. However, to avoid multiplicity of suits, the better recourse is for the mortgagor to file a case for
annulment of foreclosure with an alternative cause of action for the return of the surplus, if any.
181. ECHEGARAY V. SECRETARY OF JUSTICE (1999)
The courts have jurisdiction over the execution phase of its decisions.
Leo Echegaray was sentenced to death. However, the Court afterward issued a Resolution temporarily restraining
the execution of Leo Echegaray. The Secretary of Justice assails this resolution contending that since the case had
already become final and executory, the execution of the decision enters the exclusive ambit of authority of the
executive. In other words, the Secretary is contending that since the case is already final and executor, the Court no
longer has jurisdiction over the execution of its judgment and cannot therefore suspend the execution of Echegaray.
In answering this issue, the Court cited constitutional provisions strengthening the independence of the judiciary.
Section 5(5), Article VIII provides: (5) Promulgate rules concerning the protection and enforcement of
constitutional rights, pleading, practice and procedure in all courts, the admission to the practice of law, the
Integrated Bar, and legal assistance to the underprivileged. Such rules shall provide a simplified and inexpensive
procedure for the speedy disposition of cases, shall be uniform for all courts of the same grade, and shall not
diminish, increase, or modify substantive rights. Rules of procedure of special courts and quasi-judicial bodies
shall remain effective unless disapproved by the Supreme Court. If the manifest intent of the 1987 Constitution is
to strengthen the independence of the judiciary, it is inutile to urge, as the Secretary does, that the Court has
no jurisdiction to control the process of execution of its decisions, a power conceded to it and which it has
exercised since time immemorial.

RULE 40

14. CRUZ v. MANILA INTERNATIONAL AIRPORT AUTHORITY (2013)


An appellee who has not himself appealed cannot obtain from the appellate court any affirmative relief other
than the ones granted in the decision of the court below.
A landlord-lessee dispute broke out between Cruz (lessee) and the MIAA. This dispute arose from the fact that
MIAA allegedly breached its lease contract with Cruz, rendering the latter unable to pay rent. Cruz then filed an
annulment of contract case in Pasig. MIAA countered, alleging improper venue, since in the complaint for
annulment of contract, it is indicated that Cruz is a resident of Manila. RTC-Pasig rendered a decision against Cruz,
prompting her to file a notice of appeal. In the CA, MIAA reiterated its argument regarding improper venue, which
was previously denied by the RTC. CA eventually said that the case is dismissible based on improper venue. Cruz
was flustered, so she went to the SC, arguing that in ruling on the issue of improper venue, the CA practically
allowed MIAA to pursue a lost appeal, although the latter did not file a notice of appeal within the proper
reglementary period nor pay the prescribed docket fees.
SC said that the CA erred in sustaining the dismissal of the Pasig case on the ground of improper venue
because the same was not an error raised by Cruz who was the appellant before it. The CA cannot take

REM Digests ALS 2014B Justice Gesmundo


cognizance of MIAAs position that the venue was improperly laid since, being the appellee, MIAAs
participation was confined to the refutation of the appellants assignment of errors.
76. DOLORES ADORA MACASLANG VS. RENATO AND MELBA ZAMORA G.R. No. 156375, May 30,
2011; Rule 51, Section 8- Appeal; Ejectment; Rule 40, Sec. 7
The Regional Trial Court (RTC) is not limited in its review of the decision of the Municipal Trial Court
(MTC) to the issues assigned by the appellant, but can decide on the basis of the entire records of the
proceedings of the trial court and such memoranda or briefs as may be submitted by the parties or required
by the RTC.
Respondents filed a complain for unlawful detainer in the MTCC. Despite the service of summons, the petitioner
never answered, thus she was declared in default. The petitioner appealed to the RTC and prayed for the dismissal
of complaint for failing to state a cause of action and The petitioner appealed to the RTC, averring the following as
reversible errors, namely:1. Extrinsic Fraud was practiced upon defendant-appellant 2. Defendant-Appellant
has a meritorious defense in that there was no actual sale. The RTC resolved the case in favor of the petitioner and
dismissed the case for failure to state a cause of action. The respondents appealed to the CA, assailing the RTCs
decision for disregarding the allegations in the complaint in determining the existence or non-existence of a cause
of action. On July 3, 2002, the CA reversed and set aside the RTCs decision and reinstated the MTCCs decision in
favor of the respondents. One of the issues assailed in the appeal is Whether or not the Regional Trial Court in
the exercise of its Appellate Jurisdiction is limited to the assigned errors in the Memorandum or brief filed
before it or whether it can decide the case based on the entire records of the case, as provided for in Rule 40,
Sec. 7. The Court ruled that as an appellate court, RTC may rule upon an issue not raised on appeal. The
Court ruled that the CA committed an error in disallowing the RTC as an appellate court to rule on the failure to
state a cause of action and the lack of demand to vacate. The CA might have been correct had the appeal been a
first appeal from the RTC to the CA or another proper superior court, in which instance Section 8 of Rule 51, which
applies to appeals from the RTC to the CA, imposes he express limitation of the review to only those specified in
the assignment of errors or closely related to or dependent on an assigned error and properly argued in the
appellants brief.
But the petitioners appeal herein, being taken from the decision of the MTCC to the RTC, was governed by a
different rule, specifically Section 18 of Rule 70 of the Rules of Court, to wit:
Section 18. xxx
xxx
The judgment or final order shall be appealable to the appropriate Regional Trial Court which
shall decide the same on the basis of the entire record of the proceedings had in the court of
origin and such memoranda and/or briefs as may be submitted by the parties or required by
the Regional Trial Court.
As a result, the RTC presently decides all appeals from the MTC based on the entire record of the proceedings had
in the court of origin and such memoranda or briefs as are filed in the RTC.
Yet, even without the differentiation in the procedures of deciding appeals, the limitation of the review to only the
errors assigned and properly argued in the appeal brief or memorandum and the errors necessarily related to such
assigned error sought not to have obstructed the CA from resolving the unassigned issues by virtue of their coming
under one or several of the following recognized exceptions to the limitation, namely:
(a) When the question affects jurisdiction over the subject matter;
(b) Matters that are evidently plain or clerical errors within contemplation of law;
(c) Matters whose consideration is necessary in arriving at a just decision and complete
resolution of the case or in serving the interests of justice or avoiding dispensing
piecemeal justice;
(d) Matters raised in the trial court and are of record having some bearing on the issue
submitted that the parties failed to raise or that the lower court ignored;
(e) Matters closely related to an error assigned; and
REM Digests ALS 2014B Justice Gesmundo
(f) Matters upon which the determination of a question properly assigned is dependent.[13]
Consequently, the CA improperly disallowed the consideration and resolution of the two errors despite their being:
(a)necessary in arriving at a just decision and acomplete resolution of the case; and (b) matters of record having
some bearing on the issues submitted that the lower court ignored.

RULE 41

48. ARTURO DELA CRUZ, SR. v. MARTIN AND FLORA FANKHOUSER (2012)
WHEN THE WRIT OF EXECUTION VARIES THE JUDGMENT = there is an allowable appeal to an
order of execution under Rule 41 (exception to general rule)
There is a case filed before the RTC. The RTC decides and later issues an ORDER OF EXECUTION, after the
winning party filed a motion for execution. The aggrieved party elevated the RTC Order of Execution to the CA by
notice of appeal. He claimed that the order of execution issued by the RTC varied the judgment of the CA.
Held: There are exceptions to the general rule that an order of execution is not appealable, one of which is
when the writ of execution varies the judgment. Where such order of execution in the opinion of the defeated
party varies the terms of the judgment and does not conform to the essence thereof, or when the terms of the
judgment are not clear and there is room for interpretation and the interpretation given by the trial court as
contained in its order of execution is wrong in the opinion of the defeated party, the appellate court may entertain
an appeal to the order of execution.
72. DM Wenceslao v. City of Paranaque (August 31, 2011. Sec. 4, Rule 41, Appeal-docket fees)
A court acquires jurisdiction over the subject matter of the action only upon the payment of the correct
amount of docket fees regardless of the actual date of filing of the case in court. The payment of appellate
docket fees is not a mere technicality of law or procedure. It is an essential requirement, without which the
decision or final order appealed from becomes final and executory as if no appeal was filed. Evidently, where
the appellate docket fee is not paid in full within the reglementary period, the decision of the trial court
becomes final and no longer susceptible to an appeal. For once a decision becomes final, the appellate court
is without jurisdiction to entertain the appeal. The right to appeal is not a natural right, nor part of due
processit is merely a statutory privilege and may be exercised only in the manner and in accordance with
the provisions of law.
Petitioner D.M. Wenceslao and Associates, Inc. is a domestic corporation engaged in the construction business. In
1996, the City of Paraaque passed Ordinance No. 96-16, providing for the market values of the properties within
its jurisdiction as basis for assessment and real property taxation. The ordinance also provided for a discount of
70% of the base value of the developed lots in the area, for low, sunken and undeveloped parcels of land, such as
the lots reclaimed and owned by petitioner. The City Assessor of Paraaque, however, assessed petitioners lots
based on the rates applicable to Barangay Baclaran, which rates were higher than those applicable to properties in
Barangay Tambo. Subsequently, the City Treasurer declared petitioners properties delinquent and included them in
the auction sale which DM Wenceslawo tried to stop by filing a complaint with a prayer for issuance of a TRO. The
City of Paranaque filed a MTD on the ground that the action is barred by prior judgment/ prescribed and that the
court has no jurisdiction. The City of Paranaque won in the RTC, and after appealing to the CA, the appeal of DM
Wenceslao was dismissed for late payment of docket fees.
The rule that appellate court docket and other lawful fees must be paid within the period for taking an
appeal is stated in Section 4, Rule 41 of the 1997 Rules of Civil Procedure, as amended:
SEC. 4. Appellate court docket and other lawful fees.Within the period for taking an appeal, the
appellant shall pay to the clerk of the court which rendered the judgment or final order appealed from, the
full amount of the appellate court docket and other lawful fees. Proof of payment of said fees shall be
transmitted to the appellate court together with the original record or the record on appeal.

REM Digests ALS 2014B Justice Gesmundo


It bears stressing that payment of docket and other fees within this period is mandatory for the perfection of
the appeal. Otherwise, the right to appeal is lost. This is so because a court acquires jurisdiction over the
subject matter of the action only upon the payment of the correct amount of docket fees regardless of the
actual date of filing of the case in court. The payment of appellate docket fees is not a mere technicality of
law or procedure. It is an essential requirement, without which the decision or final order appealed from
becomes final and executory as if no appeal was filed. Evidently, where the appellate docket fee is not paid in
full within the reglementary period, the decision of the trial court becomes final and no longer susceptible to
an appeal. For once a decision becomes final, the appellate court is without jurisdiction to entertain the
appeal. The explanation advanced by petitioners counsel that the failure to pay the appellate docket and
other legal fees within the prescribed period was due to his extremely heavy workload and by excusable
inadvertence does not convince.
95. VOLTAIRE RAVIRA v. HEIRS OF DELESTE (2010)
RULE 41 AND 16: A trial courts ruling on the matter of attorneys fees initiated through a motion, in a suit
for recovery of ownership and possession of land, may be appealed by a mere notice of appeal. Since the suit
is not one where multiple appeals are taken, a record on appeal is not necessary.
A suit for recovery of ownership and possession of land was instituted before the Court of First Instance of Lanao
del Norte and eventually was decided with finality. Later on, petitioner Atty. Voltaire Rovira filed as an incident
to said case a motion to resolve his claim for attorney's fees for services rendered.TC granted the case and
respondents filed a notice of appeal. The TC granted the appeal but subsequenlty dismissed it due to new
developments. CA found TC committed grave abuse of discretion.
Perfection of Appeal
Multiple appeals are allowed in special proceedings, in actions for partition of property with accounting, in the
special civil actions of eminent domain and foreclosure of mortgage. The rationale behind allowing more than one
appeal in the same case is to enable the rest of the case to proceed in the event that a separate and distinct issue is
resolved by the court and held to be final. In such a case, the filing of a record on appeal becomes indispensable
since only a particular incident of the case is brought to the appellate court for resolution with the rest of the
proceedings remaining within the jurisdiction of the trial court.
The main action involved herein, being a suit for recovery of ownership and possession, is not one where multiple
appeals can be taken or are necessary. Since the case has not been made out for multiple appeals, a record on appeal
is unnecessary to perfect the appeal. The only requirement to perfect the appeal in the present case is the filing of a
notice of appeal in due time. This the respondents did. Concededly, the respondents did not strictly follow Rule
13, Sec. 11 on priorities on modes of service. However, since rules of procedure are mere tools designed to
facilitate the attainment of justice, their strict and rigid application which would result in technicalities that tend to
frustrate rather than promote substantial justice must be avoided. The relaxation of the rules on service is all the
more proper in the present case, where petitioner had already received his copy of the notice of appeal by registered
mail.
Jurisdiction of the CA over the Petition for Certiorari
The discretion on initially determining the sufficiency of a petition for certiorari lies with the court before which the
petition was filed. In this matter, the CA determined the petition filed before it to be sufficient. First, the failure to
comply with the rule on a statement of material dates in the petition may be excused since the dates are evident
from the records.Second, the Rules do not specify the precise documents, pleadings or parts of the records that
should be appended to the petition other than the judgment, final order, or resolution being assailed. The Rules
only state that such documents, pleadings or records should be relevant or pertinent to the assailed resolution,
judgment or orders; as such, the initial determination of which pleading, document or parts of the records are
relevant to the assailed order, resolution or judgment, falls upon the petitioner. The CA will ultimately determine if
the supporting documents are sufficient to even make out a prima facie case. The CA, having given due course to
the petition, must have found the documents sufficient.Third, the caption of the petition filed with the CA may not
have specified the individual names of the heirs of Dr. Deleste but the verification contained all the names and
signatures of the four heirs. The petition sufficiently contains the full names of the petitioners therein, thus
substantially complying with the requirement of the Rules of Court.

REM Digests ALS 2014B Justice Gesmundo


186. La Sallette College v. Pilote (December 11, 2003) (Rule 41, Section 4, Docket fees)
Pilote is a student of La Sallette College. He was denied re-enrollment. He asked the court for a mandatory
injunction to compel the school to re-admit him. It was granted, but the school still refused to admit him. So Pilote
asked for damages also. The RTC ruled in favor of Pilote. The School filed a Notice of Appeal which the RTC
approved. Pilote moved for a reconsideration on the ground that the school failed to pay the docket fees within the
reglementary period. The cases revolves around the timeliness of the payment of the docket fees. The fees are
necessary to defray court expenses in the handling of cases. For this reason, and to secure a just and speedy
disposition of every action and proceeding, the Rules mandates the payment of docket and other lawful fees
within the prescribed period. Otherwise, the jurisdiction of the proper court to handle a case is adversely
affected. To perfect an appeal from a decision rendered by the RTC in the exercise of its original
jurisdiction, the following must be complied with: (1) within 15 days, a notice of appeal must be filed with
the court that rendered the judgment or final order sought to be appealed; (2) such notice must be served on
the adverse party; and (3) within the same 15-day period, the full amount of appellate court docket and other
legal fees must be paid to the clerk of the court that rendered the judgment or final order. Full payment of the
appellate docket fees within the prescribed period is mandatory, even jurisdictional, for the perfection of the
appeal. Otherwise, the appellate court would not be able to act on the subject matter of the action.
Notwithstanding the mandatory nature of the payment of docket fees, its strict application is qualified by the
following: (1) failure to pay those fees within the reglementary period allows only discretionary, not
automatic dismissal; (2) such power should be used by the court in conjunction with its exercise of sound
discretion in accordance with the tenets of justice and fair play. The School had paid after almost 7 months
from the mandated last day for payment. They have not shown any good reason to warrant the relaxation of the
Rules.

RULE 42

41-43. Mindanao Terminal and Brokerage Service, Inc. v. CA (2012)


Doctrine:
1. The service of judgment serves as the reckoning point to determine whether a decision had been
appealed within the reglementary period or has already become final;
2. Once a judgment has become final, the prevailing party is entitled as a matter of right to a writ of
execution;
3. RTC lost its jurisdiction from the time MINTERBRO perfected its appeal to the CA. (Rule 42 Sec. 8)
PPA filed a case against the Mindanao Terminal and Brokerage Services, Inc. (MINTERBRO), a private domestic
corporation. RTC ordered MINTERBRO to pay PPA governments share in MINTERBROs gross income from
its port-related services. MINTERBRO appealed before the CA (November 21, 2002). CA affirmed. Copies of the
decision were sent via registered mail to parties respective counsels along with the Notice of the Decision. PPA
filed Compliance, manifesting its receipt. MINTERBRO did not. CA Clerk of Court sent a letter-tracer to the
Postmaster requesting the registered letter to be mailed to Atty. Rafael Dizon. Postmaster advised the CA that the
registered letter was received by Cabrera on December 4, 2002. The CA decision became final and executor on
December 20, 2002, or 15 days after Cabreras receipt of the decision. Copies of Entries of Judgment were sent to
counsels, MINTERBROs copy was addressed to Atty. Rafael Dizon. Atty. Dizon claimed he did not receive the
Decision. PPA, by virtue of the Entry of Judgment, filed a Motion for the Issuance of a Write of Execution, which
was later held in abeyance per motion of MINTERBRO. PPA filed a petition for certiorari under Rule 65.
The service of judgment serves as the reckoning point to determine whether a decision had been appealed within
the reglementary period or has already become final. The fact that it was received by Cabrera does not persuade the
Court because, as certified by the Postmaster, Cabrera received the letter on 4 December 2002, or a year before
Atty. Dizons change of address, and while his office address was the old one. On that day, his office in his new
address was yet non-existent. Assuming it were true that he already moved to his new address, Atty. Dizon was

REM Digests ALS 2014B Justice Gesmundo


negligent because he failed to inform the court of a change of address. There was proper service of judgment on
MINTERBROs counsel.
Once a judgment has become final, the prevailing party is entitled as a matter of right to a writ of execution. It
becomes mandatory or the ministerial duty of the court to issue a write of execution to enforce the judgment which
has become executory. RTC abused its discretion when it held in abeyance the issuance of the writ of execution of
the judgment.
51. FENEQUITO ET. AL. V. VERGARA, JR., G.R. NO. 172829 (2012)
A final order is one that disposes of the whole subject matter or terminates a particular proceeding or
action, leaving nothing to be done but to enforce by execution what has been determined.
Vergara filed a criminal complaint for falsification of public documents against Fenequito, Hernandez and
Rodriguez before the City Prosecutor of Manila. When the corresponding Information was filed by the prosecutor,
petitioners filed a Motion to Dismiss on the ground of lack of probable cause. The MeTC granted the motion.
Vergara appealed to the RTC, which directed the MeTC to proceed with trial. This prompted the petitioners to
elevate the case to the CA via a petition for review. CA dismissed the petition, holding that the decision of the RTC
directing the MeTC to proceed with the trial is interlocutory in nature, hence, not appealable. Petitioners Motion
for Reconsideration was denied, hence this petition for review on certiorari under Rule 45 before the SC.
The order of the RTC directing the MeTC to proceed with the trial is interlocutory, hence, not appealable.
Under Section 1, Rule 42 of the 1997 Rules of Civil Procedure, a party desiring to appeal from a decision of
the RTC rendered in the exercise of its appellate jurisdiction may file a verified petition for review with the
CA. This provision contemplates a final order or decision of the RTC. A final order is one that disposes of
the whole subject matter or terminates a particular proceeding or action, leaving nothing to be done but to
enforce by execution what has been determined. The assailed order in this case clearly does not dispose of the
case in its entirety and thus, not final. Petition denied.
130. Brgy. Sangalang v. Brgy. Maguihan (2009) Rule 42
In appealing cases decided by the RTC in the exercise of its appellate jurisdiction, the proper mode of
appeal is by petition for review under Rule 42, not by way of Ordinary Appeal under Rule 41.
A barangay jurisdiction dispute arose between Brgy. Sangalang and Brgy. Maguihan. The Sangguniang Bayan
affirmed the hearing committees recommendation that the jurisdiction is with Brgy. Sangalang. Brgy. Maguihan
appealed to the RTC, which decided for Brgy. Maguihan. Brgy. Sangalang filed an MR, which was denied by the
RTC. Brgy. Sangalang filed a Notice of Appeal, which CA dismissed saying that the appellant availed of the wrong
remedy in filing a notice of appeal instead of filing a petition for review under Rule 42. Sec. 22 of BP 129 states
that decisions of the RTC in the exercise of its appellate jurisdiction, shall be appealable to the Court of Appeals by
way of petitions for review under Rule 42.
Sec. 118 of the Local Government Code provides that the jurisdiction over settlement of boundary disputes is
to be lodged before the Sangguniang Panlungsod or Sangguniang Bayan. Under Sec. 119 of the LGC, the
decision of the said Sangguinians may be appealed to the RTC having jurisdiction over the area in dispute.
In this case, when the case was appealed, the RTC took cognizance of the case in the exercise of its appellate
jurisdiction. The modes of appeal under Sec. 2, Rule 41 are Ordinary Appeal to the CA in cases decided by
the RTC in the exercise of its original jurisdiction and Petition for Review in accordance with Rule 42 in
cases decided by the RTC in the exercise of its appellate jurisdiction. Brgy. Sangalang availed itself of the
wrong remedy. It should have filed a petition for review under Rule 42, not an ordinary appeal under Rule
41. In any case, the Court recognizes the trend toward liberal construction of the Rules. Although the court
has the discretion to dismiss or not to dismiss an appeal, the CA should not have easily dismissed the petition
considering the parties are LGUs and the issue is for the determination of territorial jurisdiction. Ideally, the
case should be remanded to the CA, but since the case has been pending for almost 13 years, the SC ruled on
the issue stating that in weighing the evidence presented, Brgy. Sangalang had jurisdiction over the disputed
land.

REM Digests ALS 2014B Justice Gesmundo


RULE 43

25. CITY GOVERNMENT OF MAKATI v. ODEA (2013)


Rule 43 and 65 are available as against a writ of execution that varies the judgment or that has been issued
inequitably in light of a significant change in the situation of the parties e.g. payment.
Odea was a public school teacher in Makati. She was dropped from the payrolls. CSC issued a resolution to the
effect that her dismissal was illegal and ordered Makati to pay her salaries from separation to reinstatement. CA
modified that backwages may only be awarded to a maximum of 5 years. SC affirmed.
Odea went back to CSC question the 5 year maximum period. CSC issued a resolution ordering Makati to pay
more than 7 years' worth of backwages. Makati went to CA via Rule 43 and said CSC resolution is barred by res
judicata. CA dismissed and said 43 cannot be used as a remedy against a writ of execution and that 65 is the proper
remedy.
Held that first, 43 covers not only final orders and judgments of CSC but its awards resolutions as well.
Second, as a general rule writs of execution are not appealable subject to the following exceptions: 1) the writ
of execution varies the judgment, 2) there has been a change in the situation, 3) the writ was inequitably
issued, or defective, or issued to the wrong party (Limpin v. IAC, Banaga v. Majaducon). Here the writ
clearly varied the order.
56. DIMAGIBA v. ESPARTERO (2012)
rule 43 is the proper mode of appeal from an administrative decision of the Ombudsman
Rule 43
Dimagiba, Mendoza, and Rasco were officers of LIVECOR, a GOCC. LIVECOR entered into a management
agreement with SIDCOR, another GOCC, wherein LIVECOR would manage, administer, dispose, and liquidate all
SIDCOR assets, accounts, and projects. To do this, LIVECOR assigned its officers to hold concurrent positions in
SIDCOR, and Dimagiba, Mendoza, and Rasco were so assigned.
LIVECOR underwent reorganization, and as a result, Dimagiba, et al. were dismissed. SIDCOR offered gratuity
pay for its former employees. Dimagiba, et al. were claiming the gratuity pay, since they worked for SIDCOR,
while certain officers (respondents) opined that allowing such payment would amount to double compensation
violative of the Constitution. Gratuity pay was disallowed, so Dimagiba, et al. filed a complaint with the
Ombudsman against respondents, alleging Gross Neglect of Duty, Oppression, etc.
The Ombudsman ruled in favor of Dimagiba, et al. and ordered the dismissal of respondents. Respondents thus
filed a petition for review under rule 43 with the CA. Dimagiba, et al. claimed that the petition was filed out of
time, and that the gratuity pay did not amount to double compensation.
The Court held that the petition was filed in time (respondents filed on the 15th day from judgment). The issue here
is whether the reglementary period for filing an appeal with the CA is 10 days, as provided for in the Ombudsman
Act for appeals from administrative decisions of the Ombudsman, or 15 days, as provided for under Rule 43 of the
Rules of Court. In the earlier case of Fabian v. Desierto, the Court ruled that Section 27 of the Ombudsman Act
was unconstitutional for increasing the appellate jurisdiction of the SC without its concurrence, and appeals from
the Ombudsman for administrative cases should follow Rule 43 of the RoC. The 10 day reglementary period is
provided for under Section 27 of the Ombudsman Act. Since it was declared unconstitutional, the reglementary
period to be followed should be that provided for under Rule 43, which is 15 days. Moreover, dismissal on purely
technical grounds is frowned upon. Thus, the appeal was filed on time.
The SC also held that the gratuity pay amounted to double compensation proscribed by the Constitution, so the
respondents were reinstated.

REM Digests ALS 2014B Justice Gesmundo


58. Metropolitan Cebu Water District v. Mactan Rock Industries, Inc. (2012)
CIAC jurisdiction over disputes v. CA jurisdiction when an administrative agency or body is conferred
quasi-judicial functions, all controversies relating to the subject matter pertaining to its specialization are
deemed to be included within its jurisdiction since the law does not sanction a split of jurisdiction
Metrpolitan Cebu Water District (MCWD) entered into a Water Supply Contract with Metro Rock Industries, Inc.
(MRII) wherein it was agreed that the latter would supply MCWD with potable water. MRII filed a complaint
against MCWD with the Construction Industry Arbitration Commission (CIAC). MRII sought the reformation of a
certain clause of the contract. MCWD, in its answer, alleged that the CIAC had no jurisdiction over the case. The
issue is whether the CIAC had jurisdiction over a complaint praying for a reformation of a water supply contract.
The Supreme Court held that the CIAC has jurisdiction over the dispute.
The jurisdiction of courts and quasi-judicial bodies is determined by the Constitution and the law. It cannot
be fixed by the will of the parties to the dispute, nor can it be expanded or diminished by stipulation or
agreement. The text of Section 4 of E.O. No. 1008, which provides for the jurisdiction of the CIAC, is broad
enough to cover any dispute arising from, or connected with, construction contracts, whether these involve mere
contractual money claims or execution of the works. Thus, unless specifically excluded, all incidents and matters
relating to construction contracts are deemed to be within the jurisdiction of the CIAC.
Section 4 provides that (t)he jurisdiction of the CIAC may include but is not limited to x x x, underscoring the
expansive character of the CIACs jurisdiction. Very clearly, the CIAC has jurisdiction over a broad range of
issues and claims arising from construction disputes, including but not limited to claims for unrealized
profits and opportunity or business losses. What E.O. No. 1008 emphatically excludes is only disputes arising
from employer-employee relationships.
Neither the provisions of the Civil Code on reformation of contracts nor the law creating the CIAC exclude
the reformation of contracts from its jurisdiction. Jurisprudence further dictates that the grant of
jurisdiction over related and incidental matters is implied by law. Therefore, because the CIAC has been held
to have jurisdiction over the contract, it follows that it has jurisdiction to order the reformation of the contract
as well.
83.) Natl Water Resources Board (NWRB) v A.L. Ang Network, Inc., GR. 186459 (2010) (Rule 43; Appeal;
Jurisdiction )
Certiorari and appellate jurisdiction over adjudications of the National Water Resources Board now belongs to
the Court of Appeals in pursuant to BP 129. In this case, Respondent filed a petition for certiorari with the RTC
challenging the decision of NWRB in a proceeding before it (the proceeding concerns the allowance of a certain
private applicant to operate a water service system). The jurisdiction of RTC over such appeals is now in
question. The Supreme Court held that Art 89 of PD 1067 did specifically state that the appeals for NWRC
decisions lie with the RTC, BUT the passing of BP 129 has already repealed this clause (Sec 47, BP 129 repealed
all other statutes, letter of instructionsinconsistent with the provision of this Act.) Thus, Under BP 129, Sec 9,
the Court of Appeals has original jurisdiction to issue writs of mandamus, prohibition, certiorari, hebeas corpus,and
quo warranto.
This has been reinforced by the subsequent formulation of Rules of Court (rule 43) where CA has exclusive
appellate jurisdiction on petitions for writs of certiorari, prohibition or mandamus against the acts and omissions of
quasi-judicial agencies, like petitioners. Supreme Court also held that even though NWRB is not mentioned in the
rule, it still covered as it is among the quasi-judicial agencies. The list provided in the Rule is not exclusive.

RULE 44

REM Digests ALS 2014B Justice Gesmundo


RULE 45

19. MALAYANG MANGGAGAWA NG STAYFAST PHILS. V. NLRC (2013)


A petition for certiorari under Rule 65 of the ROC is a special civil action that may be resorted to only in the
absence of appeal or any plain, speedy and adequate remedy in the ordinary course of law. The existence and
availability of appeal prohibits the resort to certiorari. Moreover, certiorari is not and cannot be made a
substitute for an appeal where the latter remedy is available but was lost through fault or negligence.
Petitioner sought to be the exclusive bargaining agent of the employees of Stayfast Philppines Inc (SPI). After
losing the certification election, petitioner appealed the Order declaring competing union (NLMS-Olalia) as the
companys EBU. SPI refused to bargain with NLMS-Olalia, thus the latter went on strike. Subsequently, petitioner
union filed its own notice of strike and later on went a sit-down strike. Union members who participated in the
strike were terminated. Thus petitioner filed a complaint for unfair labor practice against SPI. The CA affirmed the
decision of the LA (as affirmed by the NLRC) and further ruled that there was no grave abuse of discretion on the
part of the NLRC. A petition for certiorari is a wrong remedy in this case. The CAs decision disposed of the
case in a manner that left nothing more to be done by the latter. Thus, petitioner should have filed an appeal
by petition for certiorari under Rule 45. The proper remedy to obtain a reversal of judgment on the merits,
final order or resolution is appeal.
57. Bethel Realty and Development Corporation v. Housing and Land Use Regulatory Board and Spouses
Visaya (2012)
HLURB Rules of Procedure in relation to Rules of Court (Rules 65 and 45) certiorari cannot be a
substitute for lost appeal or any plain, speedy and adequate remedy for that matter
Bethel Realty sold a parcel of lot to the Visaya spouses. Upon the full payment of the purchase price by the
spouses, the parties executed a Deed of Absolute Sale. However, despite several demands, Bethel Realty failed to
deliver the TCT covering the lot. The spouses filed a complaint with the Housing and Land Use Regulatory Board
(HLURB). Bethel was declared in default for failure to file an answer. The HLURB rendered a decision in favor of
the spouses. Bethel sought to nullify the proceedings in the HLURB by filing a petition for certiorari with
injunction with the Court of Appeals. The CA dismissed the petition.
The Court ruled that Bethel failed to exhaust all other remedies available to it. Section, Rule V of the 1996
Rules of Procedure of the HLURB provides that If the party declared in default who for good cause was unable to
file a motion to lift the order of default, and a judgment by default was consequently rendered, he may still file a
petition for review of the judgment by default with the Board in accordance with Rule XII of these Rules and
whatever defenses he has against the complainant may still be raised in said petition. Rule XVIII also provides that
the aggrieved party may also file a motion for reconsideration of the decision of the Board of Commissioners
and eventually appeal the same to the Office of the President.
Accordingly, inasmuch as certiorari cannot be a substitute for lost appeal or any plain, speedy, and adequate
remedy for that matter, especially if ones own negligence or error in ones choice of remedy occasioned
such loss or lapse, Bethel Realtys petition before the CA must fail.

RULE 46

RULE 47

62. REMEDIOS ANTONINO vs. THE REGISTER OF DEEDS OF MAKATI CITY AND TAN TIAN SU
(2012) [Rule 47 - Annulment of Judgment]
A petition for annulment of judgment cannot serve as a substitute for the lost remedy of an appeal. Grave
abuse of discretion is not a ground to annul a final and executory judgment.

REM Digests ALS 2014B Justice Gesmundo


Antonino filed a complaint against Su with the RTC of Makati City to enforce an Undertaking Agreement and
compel Su to sell to her a property in Makati City, pursuant to a right of first refusal in a prior lease agreement
between the parties. Order 1 (Dec. 8, 2004): RTC dismissed Antoninos complaint on the ground of improper
venue and non-payment of the appropriate docket fees. The RTC concluded that since the complaint is based on a
personal action, it should have been filed in the place of residence of either Antonino (Muntinlupa) or Su (Manila).
Order 2 (Jan. 6, 2005): MR was denied. Order 3 (Feb. 24, 2005): 2nd MR was denied. On Apr. 1, 2005, Antonino
filed with the CA a petition for annulment of judgment praying for the nullification of Orders 1, 2, & 3, alleging
that the RTC committed grave abuse of discretion amounting to lack of jurisdiction. CA dismissed the petition.
Held: Petition for annulment of judgment is denied. The remedy of annulment of judgment is only available under
certain exceptional circumstances as this is adverse to the concept of immutability of final judgments. Sec. 2 of
Rule 47 of the Rules of Court explicitly provides only two grounds for annulment of judgment, i.e., extrinsic
fraud and lack of jurisdiction. Additionally, one who seeks to avail of this remedy must show that the
ordinary and other appropriate remedies, such as a motion for new trial, an appeal, or a petition for relief
from judgment, are no longer available for causes not attributable to him. If he failed to avail of those cited
remedies without sufficient justification, he cannot resort to the action for annulment of judgment, for otherwise he
would benefit from his own inaction or negligence. A petition for annulment of judgment cannot serve as a
substitute for the lost remedy of an appeal. In this case, an appeal was already available from the issuance of
Order 1, as this is a final order. Antonino could have also appealed Order 2 (denying his MR) instead of filing a 2nd
MR. The denial of an MR of an order of dismissal is a final order, and is therefore, appealable. Moreover, a 2nd MR
from a final judgment or order is prohibited; hence, it can never interrupt the period to perfect an appeal.
Antoninos unjustified failure to appeal any of the issuances of the RTC bars him from filing a petition for
annulment of judgment.
Grave abuse of discretion is not a ground to annul a final and executory judgment. A petition for annulment
of judgment can only be based on extrinsic fraud and lack of jurisdiction and cannot prosper on the basis of
grave abuse of discretion. Lack of jurisdiction as a ground for the annulment of judgments pertains to lack of
jurisdiction over the person of the defending party or over the subject matter of the claim. It does not contemplate
grave abuse of discretion considering that jurisdiction is different from the exercise thereof. In fact, the RTC
did not gravely abuse its discretion or err in dismissing Antoninos complaint. The RTC was correct in classifying
Antoninos cause of action as personal and in holding that it was instituted in the wrong venue.
189. Biaco v. Philippine Countryside (February 8, 2007) (Rule 47; Annulment of Judgment)
Ernest Biaco is the husband of petitioner Teresa. Ernest obtained loans from Philippine Countryside Rural Bank,
evidenced by promissory notes. The Bank filed a complaint for foreclosure of mortgage against the spouses.
Summons was served to the spouses. But for unknown reasons, Ernesto failed to file an answer. They were declared
in default. Thereafter, they were ordered by the court to pay. The Sheriff personally served the Ernesto with a writ
of execution, personally received by Ernesto. The property was sold at a public auction. Since the amount was not
enough, the bank sought a writ of execution against the other properties of the spouses. The sheriff executed 2
notices of levy against properties registered under the name of Teresa.
Teresa thereafter sought the annulment of the RTC contending that extrinsic fraud prevented her from participating
in the judicial foreclosure proceedings. Apparently, the sheriff failed to personally serve her with summons and that
her husband concealed the proceedings from her.
Annulment of judgment is a recourse equitable in character, allowed only in exceptional cases as where there
is no available or other adequate remedy. Judgments may be annulled only on grounds of extrinsic fraud
and lack of jurisdiction or denial of due process. Extrinsic fraud exists when there is a fraudulent act
committed by the prevailing party outside of the trial of the case, whereby the defeated party was prevented
from presenting fully his side of the case by fraud or deception practice on him by the prevailing party. In this
case, there was no fraud especially since the spouses were codefendants in the case and shared the same interest.
Whatever fact conceleaed by the husband from the wife cannot be attributed to the bank. Moreover, petitioners
allegation that her signature on the promissory notes was forged does not evince extrinsic fraud. It is well-settled
that the use of forged instruments during trial is not extrinsic fraud because such evidence does not preclude
the participation of any party in the proceedings. But the RTC case was set aside because the SC found that the
trial court failed to acquire jurisdiction over the person of the petitioner.

REM Digests ALS 2014B Justice Gesmundo


RULE 48

RULE 49

RULE 50

RULE 51

RULE 52

RULE 53

RULE 54

RULE 55

RULE 56

PROVISIONAL REMEDIES

28. CALAWAG v. UP (2013)


Rule 58 - 3 facts must be established before a writ of preliminary injunction may issue. First, there is a
material and substantial invasion of a right. Second, the right is clear and unmistakable. Third, the issuance
of the writ is necessary to prevent serious damage.
Calawag and others were students in the Master of Science in Fisheries Biology program of UP Visayas. They
sought the approval of their thesis proposals by Dean Baylon. Dean Baylon rejected their proposals. The students
filed a petition for mandamus with preliminary injunction before the RTC. RTC enjoined Dean Baylon to approve
the thesis titles while the suit was pending. UP VIsayas went to CA via 65 and asked for a TRO. TRO was issued.
Held: The students were not entitled to preliminary injunction. To be entitled to a writ of preliminary
injunction, x x x the petitioners must establish the following requisites: (a) the invasion of the right sought to
be protected is material and substantial; (b) the right of the complainant is clear and unmistakable; and (c)
there is an urgent and permanent necessity for the writ to prevent serious damage. Since a preliminary
mandatory injunction commands the performance of an act, it does not preserve the status quo and is thus
more cautiously regarded than a mere prohibitive injunction. Accordingly, the issuance of a writ of
preliminary mandatory injunction [presents a fourth requirement: it] is justified only in a clear case, free
from doubt or dispute. When the complainants right is thus doubtful or disputed, he does not have a clear
legal right and, therefore, the issuance of injunctive relief is improper (China Banking v. Co, Gateway
Electronics v. Land Bank).

REM Digests ALS 2014B Justice Gesmundo


29. Espia vs Baylon
Espia et al filed a petition for mandamus against Dean Baylon to compel him to approved their thesis topics. They
also applied for preliminary injunction which was granted by the RTC.
To be entitled to a writ of preliminary injunction, x x x the petitioners must establish the following requisites: (a)
the invasion of the right sought to be protected is material and substantial; (b) the right of the complainant is clear
and unmistakable; and (c) there is an urgent and permanent necessity for the writ to prevent serious damage. It is
justified only in a clear case, free from doubt or dispute. When the complainants right is thus doubtful or disputed,
he does not have a clear legal right and, therefore, the issuance of injunctive relief is improper.
Petitioners failed to show a clear and unmistakable right that needs the protection of a preliminary mandatory
injunction. The dean has the discretion to approve or disapprove the composition of a thesis committee, and, hence,
the petitioners had no right for an automatic approval and composition of their thesis committees.

SPECIAL CIVIL ACTIONS



3. Republic of the Philippines, et al v. Herminio Harry Roque, et al (September 24, 2013) (Rule 63,
Declaratory Relief)
Private respondents filed a Petition for declaratory relief before the RTC, assailing the constitutionality of some of
the provisions of the Human Security Act of 2007. Petitioners filed a motion to dismiss, contending that the private
respondents failed to satisfy the requisites for declaratory relief. RTC denied the motion to dismiss.
The RTC committed GADALEJ when it ruled that private respondents petition had met all the requisites
for an action for declaratory relief. The following are the requisites for an action for declaratory relief: (1) the
subject matter of the controversy must be a deed, will, contract or other written instrument, statute, executive order
or regulation, or ordinance; (2) the terms of said documents and the validity thereof are doubtful and require
judicial construction; (3) there must have been no breach of the documents in question; (4) there must be an actual
justiciable controversy or the ripening seeds of one between persons whose interests are adverse; (5) the issue
must be ripe for judicial determination; and (6) adequate relief is not available through other means or other forms
of action or proceeding. The fourth, fifth, and sixth requirements are absent in this case. A justiciable controversy
refers to an existing case or controversy that is appropriate or ripe for judicial determination, not one that is
conjectural or merely anticipatory. By ripening seeds it is meant, not that sufficient accrued facts may be
dispensed with, but that a dispute may be tried at its inception before it has accumulated the asperity,
distemper, animosity, passion, and violence of a full blown battle that looms ahead. There is no justiciable
controversy here especially since allegations of abuse must be anchored on real events before courts may step
in to settle actual controversies involving rights which are legally demandable and enforceable. The private
respondents also lack locus standi since they have not shown any direct and personal interest in the case, especially
since this case concerns penal legislation. As to the fifth requisite, neither can it be inferred that the
controversy at hand is ripe for adjudication since the possibility of abuse remain highly-speculative and
merely theorized. It is well-settled that a question is ripe for adjudication when the act being challenged has
had a direct adverse effect on the individual challenging it.
26. VIVAS V. MONETARY BOARD (2013)
Rule 65 Prohibition is a remedy against actions yet to be made. Direct filing of a petition to the Supreme
Court unless no other court is capable of resolving the issue or the case involves exceptional circumstances.
Vivas was a major stakeholder in EuroCredit Bank (formerly Rural Bank of Faire). After several examinations
made by the Bangko Sentral, the Monetary Board issued a resolution to prohibit the Bank from engaging in
business and to place its assets under receivership. Vivas filed a petition for prohibition before the Supreme Court.
Held: Wrong remedy. The action assailed had already been made i.e. the Resolution had already been issued.
Certiorari would have been proper but Vivas also failed to observe the heirarchy of courts principle. Besides,
under Sec. 4, Rule 65, cases involving acts and omissions of a quasi-judicial agency must be filed before the
CA. The Monetary Board is a quasi-judicial agency.
REM Digests ALS 2014B Justice Gesmundo
27. PNB v. ARCOBILLAS (2013)
Rule 65 As a rule, a motion for reconsideration is a prerequisite of the filing of a petition for certiorari.
In her capacity as PNB employee, Arcobillas erroneously credited around $5k to a client (should have credited
P5k). PNB incurred losses and terminated her employment. Arcobillas filed a complaint for illegal dismissal. LA
ordered reinstatement and NLRC affirmed. Without filing an MR, PNB filed certiorari before CA. CA dismissed.
An MR was filed by both parties and both were denied.
Held: A Motion for Reconsideration is necessary before an aggrieved party can file a special civil action for
certiorari. However, this rule admits of several exceptions as (a) where the order is a patent nullity, as
where the court a quo has no jurisdiction; (b) where the questions raised in the certiorari proceedings
have been duly raised and passed upon by the lower court, or are the same as those raised and
passed upon in the lower court; (c) where there is an urgent necessity for the resolution of the
question and any further delay would prejudice the interests of the Government or of the petitioner
or the subject matter of the action is perishable; (d) where, under the circumstances, a [M]otion for
[R]econsideration would be useless; (e) where petitioner was deprived of due process and there is
extreme urgency for relief; (f) where, in a criminal case, relief from an order of arrest is urgent
and the granting of such relied by the trial court is improbable; (g) where the proceedings in the
lower court are a nullity for lack of due process; (h) where the proceeding was ex parte or in
which the petitioner had no opportunity to object; and, (i) where the issue raised is one purely of
law or where public interest is involved (Abraham v. NLRC). In this case, the existence of any one of the
exceptions was not established.
31. Sps. Sabitsana vs Muertegui
This case is an action to quiet title which was caused by the fact that the lot was sold twice by the owner. It was
contended that the RTC had no jurisdiction because the land was worth only Php 1,230 while the jurisdiction for all
civil actions in the RTC was, at that time, at a minimum of 20,000/50,000.
RTC has jurisdiction. It is clear under the Rules that an action for quieting of title may be instituted in the RTCs,
regardless of the assessed value of the real property in dispute. Under Rule 63 of the Rules of Court an action to
quiet title to real property or remove clouds therefrom may be brought in the appropriate RTC.
97. EAGLE RIDGE GOLF & COUNTRY CLUB vs. CA (2010)
Certiorari is an extraordinary, prerogative remedy and is never issued as a matter of right. The party who
seeks to avail of it must strictly observe the rules laid down by law.
"EagleRidge Employees Union" (EREU) filed a petition for certification election. Eagle Ridge Golf & Country
Club opposed, ascribed misrepresentation, false statement, or fraud to EREU in connection with the adoption of its
constitution and by-laws, the numerical composition of the Union (registration stated 30 members, while minutes
showed only 26)(5 members wanted to be excluded from the union which would reduce the membership below
the mandatory minimum), and other discrepancies. DOLE and BLR found for Eagle Ridge, MR granted for EREU.
Eagle Ridge went to CA but dismissed petition for certiorari.
Main Issue: Whether there was fraud NO, records failed to show any misrepresentation, false statement or fraud
that would amount to cancellation of registration.
REM Issue: Petitions for certiorari under Rule 65 of the Rules of Court require a sworn certification of non-forum
shopping as provided in the third paragraph of Section 3, Rule 46. Sec. 3, paragraphs 4 and 6 of Rule 46 pertinently
provides: SEC. 3. Contents and filing of petition; effect of non-compliance with requirements. x x x x The
petitioner shall also submit together with the petition a sworn certification that he has not theretofore
commenced any action involving the same issues in the Supreme Court, the Court of Appeals x x x, or any other
tribunal or agency; if there is such other action or proceeding, he must state the status of the same x x x The failure
of the petitioner to comply with any of the foregoing requirements shall be sufficient ground for the dismissal
of the petition.
Evidently, the Rules require the petitioner, not his counsel, to sign under oath the requisite certification against non-
forum shopping. Such certification is a peculiar personal representation on the part of the principal party, an

REM Digests ALS 2014B Justice Gesmundo


assurance to the court that there are no other pending cases involving basically the same parties, issues, and cause
of action. The signature of Eagle Ridges counsel was without the requisite authority.
It is, thus, clear that the counsel is not the proper person to sign the certification against forum shopping. If, for any
reason, the principal party cannot sign the petition, the one signing on his behalf must have been duly authorized.
Nevertheless, the Court will explore the merits of the instant case to obviate the inequity that might result from the
outright denial of the petition.
110. Domdom vs. Third and Fifth Divisions of the Sandiganbayan, (February 24, 2010) (Rule 119; Rule 65
Section 4; Motion for Extension of Time)
That no mention is made in the amended Section 4 of Rule 65 of a motion for extension, unlike in the
previous formulation, does not make the filing of such pleading absolutely prohibited
Petitioner filled a motion for extension of time file a petition for certiorari, then subsequently filled the petition for
certiorari beyond the period allowed in Section 4 of Rule 65. Supreme Court allowed the petition for the following
reasons. That no mention is made in the above-quoted amended Section 4 of Rule 65 of a motion for extension,
unlike in the previous formulation, does not make the filing of such pleading absolutely prohibited. If such were the
intention, the deleted portion could just have simply been reworded to state that no extension of time to file the
petition shall be granted. Absent such a prohibition, motions for extension are allowed, subject to the Courts
sound discretion. The present petition may thus be allowed, having been filed within the extension sought and, at all
events, given its merits.
179. KLAVENESS MARITIME AGENCY, INC. and DENHOLM SHIP MANAGEMENT (HK), LTD., vs.
JOSE MARIUS F. PALMOS and NATIONAL LABOR RELATIONS COMMISSION, respondents, (1994)
Prior motion for reconsideration is not indispensible for commencement of certiorari proceeding
Private respondents were employees of petitioner. They were hired as Able Seaman by a local manning agent. The
manning contract stipulated a period of employment of twelve (12) months but they were repatriated to the
Philippines from the Port of Santos, Brazil.
As a result of their untimely repatriation, private respondents each filed a complaint for illegal dismissal, as well as
non-payment and underpayment of wages with the Philippine Overseas Employment Administration ("POEA"). In
response, petitioners Denholm and Klaveness jointly filed a complaint against Palmos and Sevilla for disciplinary
action and reimbursement of repatriation expenses (POEA Case No. [M] 89-08-742). On motion of Klaveness and
Denholm, the three (3) cases were consolidated.
In time, the POEA rendered a decision in the three (3) consolidated cases before it, in favor of Palmos and Sevilla.
The POEA's decision was affirmed by the National Labor Relations Commission ("NLRC") on appeal by
petitioners. Both the POEA and the NLRC held that petitioners companies had failed to discharge their burden of
establishing the existence of a just or authorized cause for the dismissal of private respondents, who were
accordingly considered as illegally dismissed and as entitled to an award of salaries corresponding to the unexpired
portion of their contracts of employment as seamen and unpaid and underpaid salaries. Petitioner companies were
held solidarily liable for the amount found to be due to Palmos and Sevilla. Upon the other hand, petitioners'
complaint for disciplinary action and reimbursement of repatriation expenses was dismissed for lack of merit.
In the present Petition for Certiorari, Denholm and Klaveness claim that the NLRC had committed grave
abuse of discretion in disregarding the evidence which petitioners had submitted to prove their case, and in
failing to find that petitioners had terminated the services of Palmos and Sevilla for a just or authorized
cause and with due process. The Supreme Court granted petitioners' prayer for a temporary restraining order
enjoining the NLRC from executing its Decision, in order to prevent the present Petition becoming in effect moot
and academic. Palmos and Sevilla ask the Court to dismiss the present Petition for having been prematurely filed,
petitioners having failed to file a motion for reconsideration with the NLRC before instituting the present Petition
for Certiorari.
The Court does not agree. A prior motion for reconsideration is not indispensable for commencement of
certiorari proceedings if the errors sought to be corrected in such proceedings had been duly heard and
passed upon, or were similar to the issues already resolved by the tribunal or agency below. Accordingly, the
Court has excused the non-filing of a motion for reconsideration when such a motion would be basically pro
REM Digests ALS 2014B Justice Gesmundo
forma in nature and content, and where, as in the present Petition, the questions raised are essentially legal
in nature. We do not consider that the present Petition was prematurely filed with this Court.

OTHERS

2. Spouses Sarmiento v. Spouses Magsino (October 16, 2013)


Courts will not decide issues which have become moot and academic, there being no more practical
value/use.
Spouses Magsino filed a Complaint for Specific Performance and Damages with respect to a delivery of some
properties before RTC Branch 93. RTC Branch 93 ruled in favor of the Magsinos. The Clerk of Court had issued a
writ of execution against the properties, which were then in the possession of the Sarmientos. The levied properties
were sold at a public auction and bought by the Magsinos. TCTs were issued in their favor. The Magsinos
thereafter filed for the issuance of a writ of possession directing the sheriff to place them in actual physical
possession of the properties. The Sarmientos opposed. The Magsinos were granted the writ by RTC Branch 93 and
subsequently got physical possession of the properties.
BUT prior to RTC Branch 93s order, the Sarmientos had already filed a separate complaint for recovery of
possession and ownership against the Magsinos before RTC Branch 31. The Magsinos opposed on the ground that
the acts sought to be restrained was already fait accompli there was nothing else to perform regarding the act
sought to be restrained because the sheriff, upon the order of RTC Branch 93 had already placed the Magsinos in
possession. RTC Branch 31 sided with the Sarmientos. The issue sought to be resolved by the SC in this case is
W/N RTC Branch 31 interfered with the judgment and order of RTC Branch 96, a co-equal court, when it issued an
order granting the preliminary injunction restraining respondent spouses from occupying the properties and
ordering them to vacate the same, which in effect enjoined the enforcement of the writs of execution and possession
issued by RTC Branch 93.
Yet, pending resolution of this SC case, RTC Branch 31 had already decided the case in favor of the Sarmientos
and ordered the the cancellation of the TCTs in favor of the Magsinos. Thus, the case has become moot and
academic. It is a rule of universal application that courts of justice constituted to pass upon substantial rights
will not consider questions where no actual interests are involved. And where the issues have become moot
and academic, there is no justiciable controversy, so that a declaration thereon would be of no practical use
or value. The Court will refrain from expressing its opinion in a case where no practical relief may be
granted in view of a supervening event.
15. MALVAR v. KRAFT FOODS, ET AL. (2013)
Although the practice of law is not a business, an attorney is entitled to be properly compensated for the
professional services rendered for the client, who is bound by her express agreement to duly compensate the
attorney.
Malvar was terminated by Kraft Foods amidst relatively spurious circumstances. Malvar filed a complaint with the
NLRC, a battle she fought until she entered into a compromise settlement with the employer. After signing the
compromise, she tried to terminate the case by withdrawing the appeal she took. However, before the Court could
act on the withdrawal by Malvar, the Court was flabbergasted by a so-called Motion for Intervention to Protect
Attorneys Rights filed by a law firm (counsel of Malvar), praying that its contingent fees be paid in accordance to
the firms contract with Malvar. It appeared that Malvar, to escape paying the firm, unceremoniously terminated the
services of the firm.
The SC held that the firms withdrawal from the case neither cancelled nor terminated the written
agreement on the contingent attorneys fees. Nor did the withdrawal constitute a waiver of the agreement. In
the end, the Court granted the firms Motion for Intervention to Protect Attorneys Rights as a measure of
protecting the firms right to its stipulated professional fees that would be denied under the compromise
agreement. Basically, the SC said that Malvar and Kraft Foods connived by settling behind the firms back
to escape having to pay the firms contingent fees.

REM Digests ALS 2014B Justice Gesmundo


40) UP v. DIZON (2012)
exception to the finality of judgments (post decision circumstance)
UP contracted respondent Stern Builders Inc. for modification of the CAS Building in UPLB. When UP failed to
pay due to COA disallowance, respondent filed a case against UP resulting in the decision to garnish public funds
amounting to P16M. The RTC gave a favorable decision to respondent. UP failed to appeal within the 15-day
period, but filed 7months thereafter arguing it only received the decision around that time. RTC Judge Dizon
ordered the release of the funds. UP files for Certiorari assailing that the finality of the RTC decision can be
challenged and that the fresh-period rule can be given retroactive application. The court granted the petition. As a
general rule, once a decision has become final and executory, the prevailing party should not be deprived of
reaping the fruits of victory. But an exception to this would be when circumstances transpire after the
finality of the decision to render the execution unjust and inequitable. In the present case, SC rules that the
non-acceptance of RTC to the appeal made by UP for the Nov 28 2001 decision was inequitable and was a
clear violation to UPs right to due process: The service of the denial for MR was defective since it was not
given to the counsel of record, the OLS, but to Atty Nolasco of UPLB Legal. Only by May 31 2002 did OLS
receive a copy
The filing of the notice of appeal on Jun 3 was well within the reglementary period (as per the fresh-period
rule) For equity, the fresh-period rule should and must apply in this case QED the finality of RTC decision is
set aside The retroactive effect of a procedural law does not come within the legal conception of
retroactivity or is not subject to the general rule prohibiting the retroactive operation of statutes (Sec4,
NCC) rather, its retroactivity is already given since, by the nature of rules of procedure, no vested right is
impinged in its application (Sec 2252, NCC)
66. Navia v. Pardico (2012)
Writ of Amparo
To fall within the ambit of A.M. No. 07-9-12-SC in relation to RA No. 9851, the disappearance must be
attended by some governmental involvement. This hallmark of State participation differentiates an enforced
disappearance case from an ordinary case of a missing person.
The relevant laws and rules for the issuance of a Writ of Amparo are:
[1] Section 1 of A.M. No. 07-9-12-SC, on the Rule on the Writ of Amparo, which took effect on October 24,
2007.] SECTION 1. Petition. The petition for a writ of amparo is a remedy available to any person whose
right to life, liberty and security is violated or threatened with violation by an unlawful act or omission of a
public official or employee, or of a private individual or entity. The writ shall cover extralegal killings and
enforced disappearances or threats thereof. (Emphasis ours.)
[2] Section 3(g), R.A. No. 9851, otherwise known as the Philippine Act On Crimes Against International
Humanitarian Law, Genocide and Other Crimes Against Humanity, define and identify the elements which
constitute enforced disappearance, which are:
(a) that there be an arrest, detention, abduction or any form of deprivation of liberty;
(b) that it be carried out by, or with the authorization, support or acquiescence of, the State or a political
organization;
(c) that it be followed by the State or political organizations refusal to acknowledge or give information on
the fate or whereabouts of the person subject of the amparo petition; and,
(d) that the intention for such refusal is to remove subject person from the protection of the law for a
prolonged period of time.
Navia, Bio and Busing, as security guards at the Asian Land security department, invite Ben and Bong, to their
office because they received a report from a certain Mrs. Emphasis, a resident of Grand Royale Subdivision, that
she saw Bong and Ben removing a lamp from a post in said subdivision. Ben was missing after the incident; his
wife, Virginia, filed a petition for Writ of Amparo before the RTC of Malolos City.

REM Digests ALS 2014B Justice Gesmundo


Petition of Writ of Amparo dismissed. Navia et al. are mere security guards at Grand Royale Subdivision in
Brgy. Lugam, Malolos City and their principal, the Asian Land, is a private entity. They do not work for the
government and nothing has been presented that would link or connect them to some covert police, military
or governmental operation.
100) IN RE: EXEMPTION OF THE NATIONAL POWER CORPORATION FROM PAYMENT OF
FILING/ DOCKET FEES (2010)
The 1987 Constitution took away the power of Congress to repeal, alter, or supplement rules concerning
pleading, practice and procedure.
The National Power Corporation (NPC) seeks clarification from the Court on whether or not it is exempt from the
payment of filing fees, appeal bonds and supersedeas bonds.
On December 6, 2005, NPC is exempt based on R.A. No. 6395 (An Act Revising the Charter of the National Power
Corporation). On October 27, 2009 their exemption was denied by P.D. No. 938 amending RA 6395 and Section
5(5), Art VIII of the Constitution. NPC is a national government-owned and controlled corporation (RA 9136).
Section 22 of Rule 141 (Rules of Court) reads: Sec. 22. Government exempt. The Republic of the Philippines, its
agencies and instrumentalities are exempt from paying the legal fees provided in this rule. Local government units
and government-owned or controlled corporations with or without independent charters are not exempt from
paying such fees.
The 1987 Constitution took away the power of Congress to repeal, alter or supplement rules concerning
pleading, practice, and procedure; and that the power to promulgate these rules is no longer shared by the
Court with Congress and the Executive.
Since the payment of legal fees is a vital component of the rules promulgated by this Court concerning
pleading, practice and procedure, it cannot be validly annulled, changed or modified by Congress. As one of
the safeguards of this Courts institutional independence, the power to promulgate rules of pleading, practice
and procedure is now the Courts exclusive domain. That power is no longer shared by this Court with
Congress, much less the Executive.
138. Quasha, Ancheta, Pena, et. al. v. The Special Sixth Division of the Court of Appeals (2009)
Quasha Law Office is the duly authorized counsel of LIRL. LIRL is a foreign corporation from Hong Kong and is
licensed to operate a resort casino hotel in Subic Bay. Petitioner LIRL filed a Complaint for Annulment of
Contract, Specific Performance with Damages and Application for Preliminary Injunction and Temporary
Restraining Order before the Regional Trial Court (RTC) of Olongapo City, Branch 72, docketed as Civil Case No.
219-0-2004, against PAGCOR and SBMA for amending the 19 March 1993 Agreement, notwithstanding the total
absence of any consideration supporting petitioner LIRLs additional obligations. The RTC ruled in favor of LIRL.
PAGCOR filed a Notice of Appeal Ad Cautelam with the Sixth Division of the Court of Appeals. Meanwhile, in
relation to the winding up of LIRL Companies Hong Kong, the Hong Kong Court of First Instance issued an Order
appointing Kelvin Edward Flynn and Cosimo Borelli as the joint and several liquidator of LIRL, granting them the
power to carry on and manage the business of petitioner LIRL, including its business in Subic, Philippines.
Pursuant to such order, Flynn sent a letter to Picazo Law Office notifying it that its legal services as counsel for
LIRL has been terminated and that petitioner Quasha Law Office is LIRLs new counsel. The Special Sixth
Division of the Court of Appeals refused to recognize Quasha Law Office, saying that because such order of the
HK court is considered a foreign judgment, our courts should not take judicial notice thereof. Final orders of
foreign tribunals could only be enforced in Philippine courts after appropriate proceedings are filed. W/N the HK
court order was a foreign judgment? NO. No enforcement of a foreign judgment was involved in this case. LIRLs
appointed liquidators had been duly authorized to manage petitioner LIRL. The authority of the said liquidators
extended to all of petitioner LIRLs branches, wherever situated, the branch in the Philippines included. The
act of terminating the legal services of private respondent Picazo Law Office and engaging in its place
petitioner Quasha Law Office was a mere exercise of petitioner LIRLs prerogative, through its appointed
liquidators, which was an internal affair that required no prior recognition in a separate action.
178. PHILSA INTERNATIONAL PLACEMENT and SERVICES CORPORATION vs. THE HON.
SECRETARY OF LABOR AND EMPLOYMENT, VIVENCIO DE MESA, RODRIGO MIKIN and
CEDRIC LEYSON (2001)
REM Digests ALS 2014B Justice Gesmundo
Private respondents, were recruited by petitioner for employment in Saudi Arabia. While in Saudi Arabia, private
respondents were allegedly made to sign a second contract which changed some of the provisions of their original
contract resulting in the reduction of some of their benefits and privileges. Their foreign employer allegedly forced
them to sign a third contract which increased their work hours from 48 hours to 60 hours a week without any
corresponding increase in their basic monthly salary. When they refused to sign this third contract, the services of
private respondents were terminated and they were repatriated to the Philippines.
Upon their arrival in the Philippines, private respondents demanded from petitioner Philsa the return of their
placement fees and for the payment of their salaries for the unexpired portion of their contract. When petitioner
refused, they filed a case before the POEA against petitioner with the following causes of action:
1. Illegal dismissal;
2. Payment of salary differentials;
3. Illegal deduction/withholding of salaries;
4. Illegal exactions/refund of placement fees; and
5. Contract substitution.
The case was docketed as POEA Case No. (L) 85-05 0370.
Under the rules of the POEA dated May 21, 1985, complaints involving employer-employee relations arising out of
or by virtue of any law or contract involving Filipino workers for overseas employment, including money claims,
are adjudicated by the Workers' Assistance and Adjudication Office (hereinafter the "WAAO") thru the POEA
Hearing Officers. On the other hand, complaints involving recruitment violations warranting suspension or
cancellation of the license of recruiting agencies are cognizable by the POEA thru its Licensing and Recruitment
Office (hereinafter the "LRO"). In the case at bench, the first two causes of action were in the nature of money
claims arising from the employer-employee relations and were properly cognizable by the WAAO. The last two
causes of action were in the nature of recruitment violations and may be investigated by the LRO. The third cause
of action, illegal deduction/withholding of salary, is both a money claim and a violation of recruitment regulations
and is thus under the investigatory jurisdiction of both the WAAO and the LRO.
On the aspects of the case involving money claims arising from the employer-employee relations and illegal
dismissal, the POEA rendered a decision which both the petitioner and private respondents felt aggrieved. Thus,
petitioner and private respondents filed separate appeals from the August 31, 1988 POEA Decision to the NLRC.
The NLRC modified the appealed decision of the POEA Adjudication Office by deleting the award of salary
deductions and differentials. These awards to private respondents were deleted by the NLRC considering that these
were not raised in the complaint filed by private respondents. The NLRC likewise stated that there was nothing in
the text of the decision which would justify the award.Private respondents filed a Motion for Reconsideration but
the same was denied by the NLRC in a Resolution dated October 25; 1989.
Private respondents the decision of the NLRC to the Supreme Court in a petition for review for certiorari which was
however dismissed outright for "insufficiency in form and substance, having failed to comply with the Rules of
Court and Circular No. 1-88 requiring submission of a certified true copy of the questioned resolution.
Almost simultaneous with the promulgation of the August 31, 1988 decision of the POEA on private respondents'
money claims, the POEA issued a separate Order dated August 29, 1988 resolving the recruitment violations
aspect of private respondents' complaint. In this Order, the POEA found petitioner guilty of illegal exaction,
contract substitution, and unlawful deduction. Hence, the instant Petition for Certiorari. ( Sabi ng petitioner,
POEA cannot make me liable anymore sa illegal exaction because nadismiss na ung private respondents dati ng
Supreme Court.
Petitioner argues that the public respondent committed grave abuse of discretion in holding petitioner liable
for illegal deductions/withholding of salaries considering that the Supreme Court itself has already absolved
petitioner from this charge. Petitioner premises its argument on the fact that the July 26, 1989 Decision of
the NLRC absolving it from private respondent de Mesa's claim for salary deduction has already attained
finality by reason of the dismissal of private respondents' petition for certiorari of the said NLRC decision by
the Supreme Court.

REM Digests ALS 2014B Justice Gesmundo


Petitioner is correct in stating that the July 26, 1989 Decision of the NLRC has attained finality by reason of
the dismissal of the petition for certiorari assailing the same. However, the said NLRC Decision dealt only
with the money claims of private respondents arising from employer-employee relations and illegal dismissal
and as such, it is only for the payment of the said money claims that petitioner is absolved. The
administrative sanctions, which are distinct and separate from the money claims of private respondents, may
still be properly imposed by the POEA. In fact, in the August 31, 1988 Decision of the POEA dealing with the
money claims of private respondents, the POEA Adjudication Office precisely declared that "respondent's
liability for said money claims is without prejudice to and independent of its liabilities for the recruitment
violations aspect of the case which is the subject of a separate Order."
The NLRC Decision absolving petitioner from paying private respondent de Mesa's claim for salary
deduction based its ruling on a finding that the said money claim was not raised in the complaint. While
there may be questions regarding such finding of the NLRC, the finality of the said NLRC Decision prevents
us from modifying or reviewing the same. But the fact that the claim for salary deduction was not raised by
private respondents in their complaint will not bar the POEA from holding petitioner liable for illegal
deduction or withholding of salaries as a ground for the suspension or cancellation of petitioner's license.
Under the POEA Rules and Regulations, the POEA, on its own initiative, may conduct the necessary
proceeding for the suspension or cancellation of the license of any private placement agency on any of the
grounds mentioned therein. As such, even without a written complaint from an aggrieved party, the POEA
can initiate proceedings against an erring private placement agency and, if the result of its investigation so
warrants, impose the corresponding administrative sanction thereof. Moreover, the POEA, in an
investigation of an employer-employee relationship case, may still hold a respondent liable for administrative
sanctions if, in the course of its investigation, violations of recruitment regulations are uncovered. It is thus
clear that even if recruitment violations were not included in a complaint for money claims initiated by a
private complainant, the POEA, under its rules, may still take cognizance of the same and impose
administrative sanctions if the evidence so warrants.
As such, the fact that petitioner has been absolved by final judgment for the payment of the money claim to
private respondent de Mesa does not mean that it is likewise absolved from the administrative sanctions
which may be imposed as a result of the unlawful deduction or withholding of private respondents' salary.
The POEA thus committed no grave abuse of discretion in finding petitioner administratively liable of one
count of unlawful deduction/withholding of salary.
182. REPUBLIC V. GINGOYON (2005)
Final decisions of the Court are to be followed even when Rules of Court allow for a different course of
action.
The NAIA 3 project between the Government and PIATCO was nullified in an earlier case (Agan case). It was held
that the Government must pay PIATCO for the construction of the airport before it can make use of the already
finished facilities. The Government eventually filed for expropriation. The issue in this case is whether the Rules of
Court of RA No. 8974 govern in the matter of determining the amount of initial payment and to whom this initial
payment is to be given before the government can be entitled to a writ of possession.[1] Rule 67 merely requires the
Government to deposit with a government depositary an amount equivalent to the assessed value. While allowed
under the rules, this deposit does not conform to the express order given by the Court in the Agan case which
directed the government to pay the amount due to PIATCO before it can make use of the facilities. RA. 8974,
which necessitates payment to the property owner is in conformity with the Agan decision. The Court ruled that
RA 8974 should be applied instead of the Rules of Court because the latter contravenes an earlier final
decision.

[1] Rule 67 merely requires the Government to deposit with an authorized government depositary the assessed
value of the property for expropriation for it to be entitled to a writ of possession. On the other hand, Rep. Act No.
8974 requires that the Government make a direct payment to the property owner before the writ may issue.

REM Digests ALS 2014B Justice Gesmundo

You might also like